IBPS-RRBS Officer (Pre.) Exam-2017 Held on 16-9-2017 Quantitative Aptitude Question Paper With Answer Key

IBPS-RRBS Officer (Pre.) Exam-2017 Held on 16-9-2017
IBPS-RRBS Officer (Pre.) Exam-2017 Held on 16-9-2017 Quantitative Aptitude Question Paper With Answer Key

IBPS-RRBS Officer (Pre.) Exam-2017 Held on 16-9-2017

Quantitative Aptitude

1. A mixture of milk and water in a jar comprises 12 litre of milk. If 5 litre of pure milk and 3 litre of pure water were added to this jar, the percentage of water in the new mixture would be 20%. What was the initial quantity of water in the jar? (in litre)

(A) 5

(B) 4

(C) 2

(D) 6

(E) 1

Answer: (E)

Directions- (Q. 2-6) Refer to the graph and answer the given questions-

2. If in August the number of members who registered for both the clubs together increased by 20% as compared to June, what is the number of members who registered for both the clubs together in August

(A) 1152

(B) 816

(C) 1128

(D) 1056

(E) 1028

Answer: (C)

3. The number of members who registered for Club A in April was what per cent less than that registered in June for the same club?

(A)   

(B)   

(C)   

(D)   

(E)   

Answer: (A)

4. A Club B, the number of female members who registered in March was three-fifth of the number of male members who registered in the same month. If the number of female members who registered for Club A in March was 50 more than that registered for Club B in the same month, what was the number of male members who registered for Club A in March?

(A) 140

(B) 130

(C) 110

(D) 100

(E) 120

Answer: (E)

5. What is the respective ratio between the total number of members who registered for both the clubs together in March and that in July?

(A) 9 : 13

(B) 6 : 13

(C) 7 : 17

(D) 9 : 11

(E) 6 : 11

Answer: (D)

6. What is the difference between the total number of members who registered for Club A in May and June together and that registered for Club B in the same months together?

(A) 240

(B) 180

(C) 220

(D) 160

(E) 300

Answer: (B)

7. 8 men can finish a piece of work in 25 days. 15 women can finish the same piece of work in 16 days. 4 men and 8 women started working together and worked for 10 days. After that 6 more men joined them. How many days will they now take to finish the remaining work?

(A)   

(B)  

(C)   

(D)   

(E)   

Answer: (D)

Directions- (Q. 8-12) Study the table and answer the given questions-

Number of items sold and percentage of items returned by the customers over the months in two different shops-

8. The total number of items returned by the customers to shop A in August and October together was approximately what per cent more than the total number of items returned by the customers to shop B in September and October together?

(A) 46

(B) 32

(C) 67

(D) 51

(E) 23

Answer: (D)

9. The number of items returned by the customers to shop B in August was what per cent of the number items returned by the customers to that shop in November?

(A) 64

(B) 50

(C) 44

(D) 32

(E) 26

Answer: (B)

10. What was the average number of items which were not returned by the customers to shop B in September, October and November ?

(A) 438

(B) 251

(C) 255

(D) 380

(E) 352

Answer: (D)

11. What is the respective ratio between the number of items not returned by the customers to shop A in October to the number of items returned by the customers to that shop in November?

(A) 15 : 6

(B) 21 : 5

(C) 19 : 7

(D) 11 : 3

(E) 17 : 8

Answer: (D)

12. All the items returned by the customer to shop A in September were found to be defective and the shop received complaints from some of the customers who did not return the items and about 1/12th of those items not returned by the customer were found to be defective. What was the total defective items sold by that shop in that month?

(A) 116

(B) 168

(C) 198

(D) 216

(E) 172

Answer: (A)

13. ?% of (813.97 ×08) – 754.08 + 467.06

(A) 50

(B) 46

(C) 44

(D) 52

(E) 45

Answer: (A)

14. (1356.07 − ?) ÷09 = 2196.11 ÷ 5.92

(A) 367

(B) 335

(C) 294

(D) 258

(E) 274

Answer: (D)

15. ?2 + 142.04 – 21.97×08 = 277.03

(A) 9

(B) 5

(C) 16

(D) 17

(E) 6

Answer: (D)

16. 

(A) 29

(B) 43

(C) 48

(D) 50

(E) 25

Answer: (B)

17. 

(A) 18

(B) 11

(C) 4

(D) 12

(E) 9

Answer: (C)

18. If 20% of a = b, then b% of 20 is the same as-

(A) 4% of a

(B) 5% of a

(C) 20% of a

(D) 25% of a

(E) None of these

Answer: (A)

Directions- (Q. 19-23) Refer to the pie chart and answer the given question-

19. What is the central angle corresponding to the number of brownies sold by bakery E?

(A) 104°

(B) 108°

(C) 106°

(D) 102°

(E) 105°

Answer: (B)

20. The respective ratio between the number of brownies sold by bakery D and that by bakery F is 2 : 3. If bakery F sold each brownie for Rs 50, what was the amount earned by bakery F?

(A) Rs 3200

(B) Rs 2400

(C) Rs 2000

(D) Rs 2800

(E) Rs 3000

Answer: (B)

21. What is the difference between the average of the number f brownies sold by bakeries A and B and the average of the number of brownies sold by bakeries C and E?

(A) 28

(B) 32

(C) 36

(D) 42

(E) 45

Answer: (B)

22. The respective ratio between the number of chocolate and that of vanilla sold by bakery B is 5 : 3 and the respective ratio between the number of chocolate and that of vanilla brownies sold by bakery C is 3 : 1. What is the total number of chocolate brownies sold by bakeries B and C together? (Bakeries B and C sell only chocolate and vanilla brownies)

(A) 130

(B) 120

(C) 100

(D) 150

(E) 90

Answer: (C)

23. What is the difference between the total number of brownies sold by bakeries A and D together and that sold by bakery E?

(A) 12

(B) 16

(C) 6

(D) 18

(E) 9

Answer: (B)

24. The respective ratio between the present age of A and B is 9 : 10. 8 years ago the respective ratio of 1/7th A’s age that time and 1/4th of B’s age that time was 1 : 2. What will be the respective ratio between A’s age and B’s age 8 years hence?

(A) 17 : 18

(B) 11 : 12

(C) 9 : 11

(D) 9 : 12

(E) 8 : 13

Answer: (B)

25. Area of rectangle is equal to area of the circle whose radius is 21 cm. If the length and breadth of the rectangle are in the ratio 14 : 11 respectively. What is its perimeter?

(A) 142 cm

(B) 140 cm

(C) 132 cm

(D) 150 cm

(E) 155 cm

Answer: (D)

26. A metallic cuboid measuring 12 cm × 9 cm × 2 cm is melted and cast into a cube. Find the length of each edge of the cube-

(A) 5 cm

(B) 6 cm

(C) 2 cm

(D) 8 cm

(E) 10 cm

Answer: (B)

27. A boat can travel 6.4 km downstream in 16 minutes and 9.9 km upstream in 33 minutes. What is the total time taken by the boat to travel 48 km upstream and the same distance downstream together?

(A) 4 hr 40 min

(B) 4 hr

(C) 5 hr

(D) 5 hr 20 min

(E) 6 hr

Answer: (A)

Directions- (Q. 28-32) What will come in place of question-mark (?) in the given number series?

28. 48 63  43  68  38  ?

(A) 76

(B) 73

(C) 79

(D) 66

(E) 75

Answer: (B)

29. 14 16  22  34  54  ?

(A) 76

(B) 84

(C) 86

(D) 75

(E) 95

Answer: (B)

30. 852 285  96  33  12  ?

(A) 6

(B) 8

(C) 5

(D) 3

(E) 1

Answer: (C)

31. 0.25 4  32  128  256  ?

(A) 264

(B) 256

(C) 234

(D) 274

(E) 284

Answer: (B)

32. 9 4  3  3  4  ?

(A) 11

(B) 7.5

(C) 8.5

(D) 9.5

(E) 10.5

Answer: (B)

Directions- (Q. 33-37) In this question, two equations I and II are given. You have to solve both the equation and answered as-

(A) x > y

(B) x ≥ y

(C) x < y

(D) x ≤ y

(E) relationship cannot be established or x = y

33. (I) 3x2 + 13x + 12 = 0

(II) 4y2 + 5y + 1 = 0

Answer: (C)

34. (I) x2 = 25

(II) y2 + 10y + 25 = 0

Answer: (B)

35. (I) 5x2 – 11x + 2 = 0

(II) 3y2 – 5y + 2 = 0

Answer: (E)

36. (I) 5x2 – 13x + 6 = 0

(II) 2y2 – 7y + 6 = 0

Answer: (E)

37. (I) 2x2 – 17x + 35 = 0

(II) 3y2 – 10y + 7 = 0

Answer: (A)

38. Ram invests a certain sum in Scheme A offering simple interest @ 5% p.a. for 4 years. He further invests the amount obtained from Scheme A into Scheme B offering compound interest @ 10% p.a. (compounded annually) for 2 years. If the interest obtained from Scheme B was Rs 378, what was the sum invested in scheme A ? (in Rs)

(A) 1200

(B) 1000

(C) 1600

(D) 1500

(E) 1400

Answer: (D)

39. Mohan kept 30% of his retirement fund for himself and distributed the remaining amount among his wife, his only son and his only daughter in the respective ratio of 7 : 3 : 4. If the difference between the amount that Mohan kept for himself and the amount he gave to his son is Rs 31500. What was the total retirement fund?

(A) Rs 2,40,000

(B) Rs 2,70,000

(C) Rs 2,10,000

(D) Rs 2,60,000

(E) Rs 3,00,000

Answer: (C)

40. A bag contains 2 red balls, 5 yellow balls and ‘X’ green balls. One ball is drawn at random and the probability of all being yellow is 2/9. What is the value of X?

(A) 3

(B) 12

(C) Other than those given as options

(D) 6

(E) 9

Answer: (C)

IBPS Specialist Officers Marketing (Pre.) Exam-2017 Held on 31-12-2017 Quantitative Aptitude Question Paper With Answer Key

IBPS Specialist Officers Marketing (Pre.) Exam-2017 Held on 31-12-2017
IBPS Specialist Officers Marketing (Pre.) Exam-2017 Held on 31-12-2017 Quantitative Aptitude Question Paper With Answer Key

IBPS Specialist Officers Marketing (Pre.) Exam-2017 Held on 31-12-2017

Quantitative Aptitude

Directions- (Q. 1 to 10) The question consisting of a question and two statements I and II given below. You have to decide whether the data given in the statements are sufficient to answer the question read both the statements and choose the most appropriate option.

(A) The data in statement I alone are sufficient to answer the question, while the data in statement II alone are not sufficient to answer the question

(B) The data statement II alone are sufficient to answer the question, while the data in statement I alone are not sufficient to answer the question

(C) The data either in statement I alone or statement II alone are sufficient to answer the question

(D) The data is even in both statements I and II together are not sufficient to answer the question

(E) The data in both statements I and II together necessary to answer the question

1. What is the area of the circle?

(I) Perimeter of the circle is 88 cm.

(II) Diameter of the circle is 28 cm.

Answer: (C)

2. What is the rate of interest?

(I) Simple interest accrued on an amount of Rs 25,000 in two years is less than the compound interest for the same period by Rs 250.

(II) Simple interest accrued in 10 years is equal to the principal.

Answer: (C)

3. What is the number of trees planted in the field in rows and columns?

(I) Number of columns is more than the number of rows by 4.

(II) Number of column is 20.

Answer: (D)

4. What is the speed of the current?

(I) A man can swim a distance of 9 km. in  downstream.

(II) While coming back upstream it takes him 3 hr to cover the same distance.

Answer: (E)

5. What is the minimum passing percentage in a test?

(I) Raman scored 25% marks in the test and Sunil scored 288 marks which is 128 more than Raman.

(II) Raman scored 64 marks less than the minimum passing marks.

Answer: (E)

6. What is the value of x2 + y + z?

(I) 4x + 3y + 5z = 60 and 2x = y, 2y = z

(II) 3x + 3y + 2z = 34 and 2x + 5y + 6z = 72

Answer: (A)

7. Whose body weight is second highest among the five boys Arun, Vinay, Suraj, Raju and Pratap?

(I) Average weight of Arun, Suraj and Vinay is 68 kg and average weight of Raju and Pratap is 72 kg. Also Suraj is 78 kg. Raju is 68 kg and Vinay is 46 kg.

(II) Average weight of Arun, Suraj, Vinay and Raju is 68 kg and also Suraj is 78 kg. Raju is 68 kg and Vinay is 46 kg. All of them have different weight.

Answer: (A)

8. How many marks did Subodh obtain in Phsics?

(I) The average marks to Subodh in History, Geography and Chemistry are 75

(II) His average marks in History, Geography and Physics are 78.

Answer: (D)

9. What is the population of the city A?

(I) The ratio of the population of males and females in city A is 27 : 23 and the difference between their population is 100000.

(II) The population of city A is 80% of that of city B. The difference of population of city A and city B is 312500.

Answer: (C)

10. How many students did participate in elocution?

(I) The students who participate in dancing were 150% more than that who  participated in elocution.

(II) 150 students participated in dancing.

Answer: (E)

Directions- (Q. 11 to 15) Study the following graph carefully and answer the question given below it.

11. Expenditure of company B in 2009 and 2010 are Rs 12 lakhs and Rs 14.5 lakhs respectively. What was the total income of company B in 2009 and 2010 together (in lakh rupees)?

(A) 39.75

(B) 37.95

(C) 38.75      

(D) 38.55

(E) None of these

Answer: (B)

12. Ratio of expenditure of company A and B in 2012 was 3 : 4 respectively. What was the respective ratio of their incomes in 2012?

(A) 21 : 26

(B) 13 : 14

(C) 14 : 13

(D) 26 : 21

(E) None of these

Answer: (E)

13. Total expenditure of company A in all the years together was 82.5 lakhs. What was the total income of the company A in all the years together?

(A) 1.23 crore

(B) 98.75 crore

(C) 99.85 crore

(D) Cannot be determined

(E) None of these

Answer: (D)

14. If the expenditure of company A and B in 2013 were equal and the total incomes of the two companies was Rs 5.7 lakhs. What was the total expenditure of the two companies in 2013?

(A) 4 lakhs

(B) 2 lakhs

(C) 4.2 lakhs

(D) Cannot be determined

(E) None of these

Answer: (B)

15. If the income of company B in 2010 and 2011 were in the ratio of 2 : 3 respectively. What was the respective ratio of expenditure of that company in these two years?

(A) 20 : 29

(B) 9 : 10

(C) 29 : 45

(D) 10 : 29

(E) None of these

Answer: (C)

Directions- (Q. 16 to 20) Study the pie-charts carefully to answer the given question.

16. Number of girls enrolled in dancing form are what per cent of total number of students enrolled in the school?

(rounded off to two digits after decimal)

(A) 12.35

(B) 14.12

(C) 11.67

(D) 10.08

(E) None of these

Answer: (E)

17. How many boys are enrolled in Singing and Craft together?

(A) 505

(B) 610

(C) 485

(D) 420

(E) None of these

Answer: (A)

18. What is the respective ratio of number of girls enrolled in swimming to the number of boys enrolled in swimming?

(A) 47 : 49

(B) 23 : 29

(C) 29 : 23

(D) 49 : 47

(E) None of these

Answer: (D)

19. What is the total number of girls enrolled in Swimming and Drawing together?

(A) 480

(B) 525

(C) 505

(D) 495

(E) None of these

Answer: (B)

20. What is the approximate percentage of boys in the school?

(A) 34

(B) 56

(C) 28

(D) 50

(E) None of these

Answer: (E)

Directions- (Q. 21 to 30) In each question, two equation numbered I and II are given. You have to solve both the equations and mark an appropriate answer.

(A) x < y

(B) x > y

(C) x ≥ y

(D) x ≤ y

(E) relationship between x and y cannot be established

21. (I) 6x2 + 5x + 1 = 0

(II) 15y2 + 8y + 1 = 0

Answer: (D)

22. (I) x2 + 5x + 6 = 0

(II) 4y2 + 24y + 35 = 0

Answer: (E)

23. (I) 2x2 + 5x + 3 = 0

(II) y2 + 9y + 14 = 0

Answer: (B)

24. (I) 88x2 – 19x + 1 = 0

(II) 132y2 – 23y + 1 = 0

Answer: (C)

25. (I) 6x2 – 7x + 2 = 0

(II) 20y2 – 31y + 12 = 0

Answer: (A)

26. (I) 6x2 + 23x + 20 = 0

(II) 6y2 + 31y + 35 = 0

Answer: (B)

27. (I) x2 = 81

(II) y2 – 18y + 81 = 0

Answer: (D)

28. (I) 4x2 + 20x + 21 = 0

(II) 2y2 + 17y + 35 = 0

Answer: (C)

29. (I) x2 – 14x + 48 = 0

(II) y2 + 6 = 5y

Answer: (B)

30. (I) 38x2 – 3x – 11 = 0

(II) 28y2 + 32y + 9 = 0

Answer: (D)

31. Two men P and Q start a journey from same place speed at a speed of 3 km/hr and  If they move in the same direction then what is the distance between them after 4 hours?

(A) 3 km

(B)  

(C) 2 km

(D)  

(E) None of these

Answer: (C)

Directions-(Q. 32 to 35) What will come in place of question mark (?) in the given question?

32. 

(A) 5√5

(B) (125)3

(C) 25

(D) 5

(E) None of these

Answer: (D)

33. 

(A) 2

(B) 8

(C) 512

(D) 324

(E) None of these

Answer: (E)

34. 

(A) 36 + 44√(7)

(B) 6

(C) 216

(D) 36

(E) None of these

Answer: (C)

35. 

(A) 6320

(B) 6400

(C) 6351.82

(D) 6431.82

(E) 6491.82

Answer: (C)

Directions- (Q. 36 to 40) What will come in place of question mark (?) in the given number series?

36. 28 39 63 102 158 ?

(A) 232

(B) 242

(C) 233

(D) 244

(E) None of these

Answer: (C)

37. 7 16 141 190 919 ?

(A) 1029

(B) 1019

(C) 1020

(D) 1030

(E) None of these

Answer: (E)

38. 12 17 32 57 92 ?

(A) 198

(B) 195

(C) 137

(D) 205

(E) None of these   

Answer: (C)

39. 19 25 45 87 159 ?

(A) 254

(B) 279

(C) 284

(D) 269

(E) None of these

Answer: (D)

40. 83 124 206 370 698 ?

(A) 1344

(B) 1324

(C) 1364

(D) 1334

(E) None of these

Answer: (E)

Directions- (Q.41 to 47) Study the table carefully and answer the given question.

41. What is the difference between the number of academic books published by publishing house M and P?

(A) 450

(B) 640

(C) 540

(D) 504

(E) None of these

Answer: (C)

42. How many books were given to each distributor by publisher Q if each distributor gets equal number of books?

(A) 1806

(B) 1068

(C) 1608

(D) 1308

(E) None of these

Answer: (B)

43. What is the average number of non-academic books published by publishers R and S?

(A) 18750

(B) 18850

(C) 19950

(D) 18950

(E) 19990

Answer: (C)

44. If the total number of books published by P, Q and R is increased by 30% and the total number of books published by remaining publishers be decreased by 20%, what will be the new average of books published by all the publishers?

(A) 33418

(B) 33318

(C) 32518

(D) 33618

(E) None of these

Answer: (B)

45. What is the total number of books distributed by publishers O and Q?

(A) 26702

(B) 27324

(C) 55028

(D) 54026

(E) None of these

Answer: (D)

46. Meena Kumar goes to a shop and buys a saree, costing Rs 5.225, including sales tax of 12%. The shopkeeper gives her a discount. so that the price is decreased by an amount equivalent to sales tax. The price is decreased by (nearest value)

(A) Rs 615

(B) Rs 650

(C) Rs 560

(D) Rs 580

(E) Rs 680

Answer: (C)

47. Phanse invests an amount of Rs 24,200 at the rate of 4 p.c.p.a for 6 years to obtain a simple interest later he invests the principal amount as well as the amount obtained as simple interest for another 4 years at the same rate of interest. What amount of simple interest will be obtain at the end of the last 4 years?

(A) Rs 4800

(B) Rs 4850.32

(C) Rs 4801.28

(D) Rs 4700

(E) Rs 4870.32

Answer: (C)

Directions- (Q. 48 to 50) The questions are based on the following information-

  There are three different cable channels namely ahead, luck and bang. In a survey, it was found that 85% of viewers, respond to bang, 20% to luck and 30% of ahead. 20% of viewers respond to exactly two channels and 5% to none.

48. What percentage of the viewers responded to all three?

(A) 10

(B) 12

(C) 14

(D) 16

(E) 11

Answer: (A)

49. Assuming 20% respond ahead and bang, and 16% respond to bang and luck. What is the percentage of viewers who watch only luck?

(A) 20

(B) 0

(C) 16

(D) 18

(E) 14

Answer: (B)

50. A milkman mixes 20 L of water with 80 L of milk. After selling one-fourth of this mixture, he adds water to replenish the quantity that he has sold. What is the current proportion of water to milk?

(A) 2 : 3

(B) 1 : 2

(C) 1 : 3

(D) 2 : 1

(E) 3 : 4

Answer: (A)

IBPS-R.R.B Officers (Pre.) Exam-2016 Held on 5-11-2016 Quantitative Aptitude Question Paper With Answer Key

IBPS R.R.B. Officers (Pre.) Exam, 2016
IBPS-R.R.B Officers (Pre.) Exam-2016 Held on 5-11-2016 Quantitative Aptitude Question Paper With Answer Key

IBPS-R.R.B Officers (Pre.) Exam-2016 Held on 5-11-2016

Quantitative Aptitude

1. B is 1.5 times as efficient as A. If A can complete 6/7th of a given task in 12 days, what fraction of the same task would remain incomplete if B works on it independently for 6 days only?

(A)  2/5

(B) 3/5

(C)  4/10

(D)  5/14

(E)  3/7

Answer: (D)

Directions- (Q. 2-6) Based on the following table, answer the given question.

Note- The faculty Members include Assistant Professors, Associate Professors and Professors only.

2. In University L, 5/12 of the Assistant Professors are males and in University M, 5/11 of the Assistant professors are males. What is the respective ratio between male Assistant Professors in University L and that in University M?

(A)  4 : 9

(B) 8 : 9

(C)  5 : 7

(D)  5 : 9

(E)  3 : 5

Answer: (B)

3. In University K, 80% faculty members are females. If three fourth of the total Assistant Professors are females, what per cent females are either Associate Professors or Professors?

(A)  61

(B) 64.5

(C)  62.5

(D)  65

(E)  64

Answer: (C)

4. What is the difference between total number of Associate Professors in Universities L and M together and the total number of Professors in the same Universities together?

(A)  48

(B) 45

(C)  40

(D)  46

(E)  41

Answer: (B)

5. The number of Professors in Universities J and K together is approximately what per cent more than the number of Assistant Professors in University L?

(A)  22

(B) 8

(C)  35

(D)  15

(E)  18

Answer: (D)

6. What is the average number of Assistant professors in Universities J, L and M?

(A)  139       

(B) 138

(C)  135

(D)  137

(E)  132

Answer: (D)

Directions- (Q. 7-11) What approximate value will come in place of question-mark (?) in the given question ? (You are not expected to calculate the exact value)

7. 344 ÷99 + 144.08 ÷ 8.89 = ?

(A)  119

(B) 85

(C)  43

(D)  54

(E)  70

Answer: (B)

8. √? ×88 ÷ 12.01 = 289 – 109.992

(A)  4  

(B) 16

(C)  64

(D)  36

(E)  1

Answer: (D)

9. 43.99 × 20.001 – 1439 ÷ 6 = ?

(A)  500

(B) 640

(C)  540

(D)  600

(E)  680

Answer: (B)

10. 459.85 + 519.82 = ?% of 1399.92

(A)  90

(B) 70

(C)  75

(D)  50

(E)  80

Answer: (B)

11. 40% of 249 ÷ 4 + ? = 6.9992

(A)  24

(B) 12

(C)  42

(D)  56

(E)  34

Answer: (A)

12. A starts small business with Rs 3600. At the end of few months from the start of business, B joined the business with Rs 4000. If the annual profit between A and B was divided between them in the respective ratio of 6 : 5, then B joined the business after how many months from the start of the business?

(A)  Four

(B) Two

(C)  Six

(D)  Five

(E)  Three

Answer: (E)

13. The sum of the dimensions of a room (i.e., length, breadth and height) is 24 metres and its length, breadth and height are in the ratio of 8 : 7 : 5 respectively. If the room to be painted at the rate of Rs 12 per m2, what would be the total cost incurred on paint-ing only the four walls of the room (in Rs)

(A)  2592

(B) 2648

(C)  2848

(D)  2120

(E)  1956

Answer: (A)

Directions – (Q. 14-18) Refer to the graph and answer the given question.

14. If the respective ratio between total number of scarves sold b stores M and N together in 2003 and that in 2008 is 7 : 11, what is the total number of scarves sold by stores M and N together in 2008 ?

(A)  880

(B) 1100

(C)  660

(D)  770

(E)  990

Answer: (E)

15. If the total number of scarves sold by stores M and N together in 2010 is 105% of that in 2004, what is the total number of scarves sold by stores M and N together in 2010 ?

(A)  508

(B) 524

(C)  520

(D)  504

(E)  512

Answer: (D)

16. Number of scarves sold by store M decreased by what per cent from 2005 to 2006?

(A) 

(B)   

(C)    

(D)    

(E)    

Answer: (B)

17. What is the difference between total number of scarves sold by store M in 2003 and 2004 together and total number of scarves sold b y store N in 2006 and 2007 together?

(A)  150

(B) 130

(C)  90

(D)  100

(E)  110

Answer: (E)

18. What is the average number of scarves sold by store N in 2005, 2006 and 2007?

(A)  310

(B) 280

(C)  220

(D)  290

(E)  300

Answer: (E)

19. A boat takes a total time of eight hours of travel 63 kms upstream and the same distance downstream. The speed of the current is 1/8th of the speed of the boat in still water. What is the speed of the boat in still water ? (in km/hr)

(A)  32

(B) 24

(C)  16

(D)  8

(E)  36

Answer: (C)

20. P, Q and R have a certain amount of money with themselves. Q has 25% more than what P has, and R has 1/5th of what Q has. If P, Q and R together have Rs 150, then how much money does P alone have ? (in Rs)

(A)  40

(B) 70

(C)  80

(D)  60

(E)  50

Answer: (D)

21. In a class, the average weight of 40 boys is 65 kg and that of 50 girls is 60 kg. After a few days, 40% of the girls and 50% of the boys leave. What would be the new average weight of the class (in kg) ? Assume that the average weight of the boys and the girls remains constant throughout.

(A)  65

(B) 62

(C)  68

(D)  55

(E)  58

Answer: (B)

Directions- (Q. 22-26) In this question, two equations numbered I and II are given. You have to solve both the equations and choose the appropriate option and answer as

(A) x < y

(B) x ≤ y

(C) x > y

(D) x ≥ y

(E) x = y or cannot be determined

22. (I) x2 – x – 12 = 0

(II) y2 + 4y + 4 = 0

Answer: (E)

23. (I) 2x2 – 15x + 27 = 0

(II) 2y2 – 23y + 63= 0

Answer: (B)

24. (I) x2 + 11x + 28 = 0

(II) 5y2 + 27y + 28 = 0

Answer: (B)

25. (I) x2 – 11x + 30 = 0

(II) y2 – 15y + 56 = 0

Answer: (A)

26. (I) 3x2 + 16x + 21 = 0

(II) 2y2 + 15y + 25 = 0

Answer: (E)

27. At its usual speed, a train of length L metres crosses platform 300 metres long in 25 seconds. At 50% of its usual speed, the train crosses a vertical pole in 20 seconds. What is the value of L?

(A)  160

(B) 260

(C)  200

(D)  310

(E)  350

Answer: (C)

28. Jar A has 36 lts of mixture of milk and water in the respective ratio of 5 : 4. Jar B which had 20 lts of mixture of milk and water, was emptied into Jar A, and as a result in Jar A, the respective ratio of milk and water becomes 5 : 3. What was quantity of water in Jar B?

(A)  5 lts

(B) 3 lts

(C)  8 lts

(D)  2 lts

(E)  1 lts

Answer: (A)

29. Three years ago, the respective ratio between A’s age at that time and B’s age at that time was 9 : 5. A’s age two years hence will be 17 years more than B’s age five years hence, what is B’s present age?

(A)  26 years

(B) 27 years

(C)  28 years

(D)  24 years

(E)  23 years

Answer: (C)

Directions-(Q. 30-34) Refer to the pie chart and answer the given question :

30. In November 1/12 of the available bags in store Q remained unsold and 3/16 of the available bags in store T remained unsold. How many bags were sold by stores Q and T together in November?

(A)  246

(B) 254

(C)  248

(D)  252

(E)  268

Answer: (B)

31. What is the difference between the average number of bags available in stores P and Q together and the average number of bags available in stores R and S together?

(A)  16

(B) 15

(C)  18

(D)  21

(E)  12

Answer: (A)

32. The respective ratio between the number of bags available in store R in December and that available in the same store in the November was 7 : 6. How many bags were more available in Store R in December as compared to November?

(A)  32

(B) 28

(C)  40

(D)  12

(E)  80

Answer: (C)

33. In January 2012, the total number of bags available in all the stores together was 40 more than that available in November. What was the percentage increase in the total number of bags available in all the stores together from November to January?

(A)    

(B) 5

(C)   

(D)  4

(E)   

Answer: (B)

34. What is the central angle corresponding to the number of bags available in store Q? (in degrees)

(A)  83.2

(B) 86.4

(C)  82.5

(D)  88.6

(E)  84.2

Answer: (B)

Directions-(Q. 35-39) What will come in place of question mark (?) in the given number series?

35. 455 212 131 104 95 ?

(A)  84

(B) 92

(C)  45

(D)  61

(E)  49

Answer: (B)

36. 2 3 8 27 112 ?

(A)  565

(B) 650

(C)  316

(D)  290

(E)  430

Answer: (A)

37. 45 57 67 75 81 ?

(A)  89

(B) 85

(C)  105

(D)  91

(E)  78

Answer: (B)

38. 36 37 33 42 26 ?

(A)  51

(B) 41

(C)  61

(D)  45

(E)  49

Answer: (A)

39. 5 5  15  37.5  ?  393.75

(A)  80

(B) 112.5

(C)  160

(D)  48

(E)  72

Answer: (B)

40. The sum of a series of 5 consecutive odd numbers is 225. The second number of this series is 15 less than the second lowest number of another series of 5 consecutive even numbers. What is 60% of the highest number of this series of consecutive even numbers?

(A)  36.0

(B) 34.6

(C)  34.8

(D)  40.8

(E)  39.2

Answer: (*)

IBPS Bank Specialist Officers (Personnel) (Pre.) Examination Held on 31-12-2017 Reasoning Question Paper With Answer Key

IBPS Bank Specialist Officers (Personnel) (Pre.) Examination Held on 31-12-2017 Reasoning
IBPS Bank Specialist Officers (Personnel) (Pre.) Examination Held on 31-12-2017 Reasoning Question Paper With Answer Key

IBPS Bank Specialist Officers (Personnel) (Pre.) Examination Held on 31-12-2017

Reasoning

Directions- (Q. 1-5) Study the given information carefully to answer the given question.

  Eight erasers of different colours viz. White, Yellow, Pink, Green, Orange, Blue, Red and Violet are stacked one above the other but not necessarily in the same order.

  The green eraser is kept third from the bottom. Only one eraser is kept between the green and the red erasers. Only three erasers are kept between the blue and the violet erasers. The blue erasers is neither the topmost eraser nor kept immediately above or immediately below the green eraser. As many erasers are kept between the violet and the red erasers as between the pink and the white erasers. The pink eraser is kept at one of the positions above white as well as the blue eraser. More than three erasers are kept between the pink and the yellow erasers.

1. Which of the following erasers is kept at the topmost position?

(A) Violet

(B) Cannot be determined

(C) Pink

(D) Orange

(E) Yellow

Answer: (D)

2. Which of the following is the correct position of the Blue eraser in the stack?

(A) Fourth from the bottom

(B) Immediately below the orange eraser

(C) Third from the top

(D) Immediately above the yellow eraser

(E) Second from the bottom

Answer: (C)

3. Which of the following erasers is kept immediately below the pink eraser?

(A) Cannot be determined

(B) The orange eraser

(C) The blue eraser

(D) The red eraser

(E) The white eraser

Answer: (C)

4. Which of the following statements is TRUE with respect to the given information?

(A) No eraser is kept between the pink and the white erasers

(B) The yellow eraser is kept exactly between the green and the red erasers

(C) The blue eraser is kept at one of the positions below the violet eraser

(D) Both violet and white erasers are kept below the green eraser

(E) None of the given statements is true      

Answer: (E)

5. How many erasers are kept between the yellow and the white erasers?

(A) Two

(B) Four

(C) None

(D) One

(E) Three

Answer: (A)

Directions- (Q. 6-8) In this question a statement is given followed by two courses of action numbered I and II. A course of action is a practicable and feasible step or administrative decision to be taken for follow-up, improvement of further action in regard to the problem, policy etc. One the basis of the information given in the statement, you have to assume everything in the statement to be true, and decide which of the suggested courses of action logically follow(s) for pursuing.

6. Statement :According to latest census, the number of homeless people in urban areas is much more than that in rural areas of State of A whereas this trend is reverse in all other states of the country.

Course of Action I : Homeless people in urban areas of A should be relocated to the rural areas of State A.

Course of Action II : State A should cut public spending as well as subsidies on housing for the rural poor.

(A) Either I or II follows

(B) Only I follows

(C) Neither I nor II follows

(D) Only II follows

(E) Both I and II follow

Answer: (C)

7. Statement : Despite various appeal and orders by the municipal authorities since last one year, many housing complexes have not been segregating recyclable from non-recyclabe waste, leading to losses worth several crores every month.

Course of Action : Strict penalty should be levied by the authorizes on all the housing complexes which have not been following the protocol.

Course of Action II : The municipal authority should not collec the garbage from such housing complexes unless it is segregated accordingly.

(A) Either I or II follows

(B) Only I follows

(C) Neither I nor II follows

(D) Only II follows

(E) Both I and II follow

Answer: (E)

8. Statement : The electricity bill of Company X’s office building increased by 35% after the number of employees as well as the working hours were increased two months ago.

Course of Action I : A part of salary should be deducted from all employees towards the increased electricity bills.

Course of Action II : Electricity saving devices such as LED lighting and solar powered appliances should be used in the office building in place of traditional appliances.

(A) Either I or II follows

(B) Only I follows

(C) Neither I nor II follows

(D) Only II follows

(E) Both I and II follow

Answer: (D)

Directions – (Q. 9-13) Study the following information to answer the given question.

  Eight people P, Q, R, S, T, U, V and W are sitting around a circular table, facing the centre. Each one of them has different number of books with them viz. 2, 4, 5, 6, 8, 11, 13 and 16.

  Only three people sit between R and the person having 11 books. Only two people sit between the person having 11 books and W (Either from left or right). T sits second to left of the person having 6 books. The person having 6 books in neither an immediate neighbour of R nor the person having 11 books. R does not have 6 books. Only three people sit between V and the person having 2 books. V is not an immediate neighbour of T. V does not have 6 books. Sum of the total number of books with T and Q is 19. Q is not an immediate neighbour of T. Both the immediate neighbours of Q have more books than Q. Both V and R have even number of books with them. Sum of the total number of books with immediate neighbours of S is less than 10. Us is not an immediate neighbour of S.

9. Who amongst the following has 6 books?

(A) P

(B) W

(C) S

(D) Q

(E) U

Answer: (C)

10. How many people are sitting between W and the person having 8 books when counted from the right of W?

(A) Five

(B) One

(C) Three

(D) None

(E) Two

Answer: (E)

11. Who amongst the following has 13 books?

(A) U

(B) S   

(C) T

(D) P

(E) W

Answer: (A)

12. Which of the following represents the person sitting to immediate right of T?

(A) The person having 13 books

(B) P

(C) S

(D) W

(E) The person having 4 books

Answer: (B)

13. Which of the following represents the position of U with respect of S?

(A) Fourth to the right

(B) Fourth to the left

(C) Second to the right

(D) Third to the left

(E) Third to the right

Answer: (D)

Directions- (Q. 14-17) In this question three statements followed by two conclusions numbered I and II have been given. You have to take the given statements to be true even if seem to be at variance from commonly known facts and then decide which of the given conclusions logically follows from the given statements and answered as-

(A) Only conclusion I follows

(B) Neither conclusion I nor II follows

(C) Only conclusion II follows

(D) Either conclusion I or II follows

(E) Both conclusion I and II follow

14. Statements : Some profits are losses. No loss is a return. All wages are returns.

Conclusion I : No loss in a wage.

Conclusion II : All profits can never be returns.

Answer: (E)

15. Statements : All markets are shops. Some shops are arcades. All arcades are fairs.

Conclusion I : Atleast some fairs are shops.

Conclusion II : Atleast some arcades are markets.

Answer: (A)

16. Statements : No peak is a hill. All hills are slopes. All slopes are valleys.

Conclusion I : No peak is a slope.

Conclusion II : Atleast some valleys are peaks.

Answer: (A)

17. Statements : All markets are shops. Some shops are arcades. All arcades are fairs.

Conclusion I : No arcade is a market.

Conclusion II : All markets being fairs is a possibility.

Answer: (A)

Directions- (Q. 18 and 19) This question consists of a question and two statements numbered I and II given below it. You have to decide whether the data given in the statements are sufficient to answer the question. Read both the statements and choose the most appropriate option. as-

(A) Statement I alone sufficient to answer the question while statement II is not.

(B) Statement II alone sufficient to answer the question while statement I is not.

(C) Either statement I alone or statement II alone sufficient to answer the question.

(D) Both statement I and statement II together are necessary to answer the question.

(E) Both statement I and II together are not sufficient to answer the question.

18. How far is Point A from Point C?

(I)  Ameena starts walking from point A. She walks 5 m towards north, takes a right turn and walks 7 m. She takes a right turn, walks 11 m and stops at point B. Point C is 6 m away from Point B.

(II) Raju starts walking from point A. He walks 6 m towards west, takes a left turn, and walks 22 m. He then turns left again, walks 13 m and stops at Point Q. Point C is to the north of both Point Q as well as B.

Answer: (D)

19. How is ‘treat’ coded in the given code language ? (All the given codes are two letter codes only)

(I) ‘treat them same’ is coded as ‘cu ti su’ in the code language. In the same code language, ‘same by them’ is coded as ‘ti py cu’.

(II) ‘a treat for eyes’ is coded as ‘su jo vi la’ in the code language. In the same code language, ‘eyes for a cause’ is coded as ‘vi ko la jo’.

Answer: (C)

Directions- (Q. 20 and 21) Study the following information and answer the question.

  A certain number of people are sitting in a straight, horizontal line facing north. Only three people sit between Manas and Ravi. Ravi sits at one of the positions to the right of Manas. Only two people sit between Manas and Charu. Priya sits third to the right of Charu. Only three people sit between Priya and Fazila. Less than eight people sit in the line.

20. How many people sit between Manas and Fazila?

(A) Six

(B) One

(C) Three

(D) Two

(E) Four

Answer: (B)

21. If Anya is an immediate neighbour of Ravi, what is Anay’s position with respect to Priya?

(A) Immediate left

(B) Immediate right

(C) Second to the left

(D) Third to the left

(E) Second to the right

Answer: (A)

22. Four of the following five are alike in a certain way based on the English alphabetical order and so form a group. Which is the one that does not belong to that group?

(A) CGEF

(B) VZXW

(C) LPNM

(D) QUSR

(E) GKIH

Answer: (A)

Directions- (Q. 23-25) Read the following information to answer the given question.

  Each of the six cities viz. A, B, C, D, E and F have different number of lakes. Only one city has more lakes than A. D has more lakes than E and C but less than B. B does not have the highest number of lakes. The city having third highest number of lakes has 9 lakes. E has 4 lakes.

23. If C has 6 lakes, which of the following is true?

(I) C has second lowest number of lakes

(II) City D possibly has 11 lakes

(III) Only two cities have more number of lakes than E.

(A) Both I and II

(B) Only II

(C) Only III

(D) Only I

(E) All I, II and III

Answer: (D)

24. How many lakes does A possibly have ?

(A) 10

(B) 3

(C) 7

(D) 5

(E) 8

Answer: (A)

25. How many cities have more lakes than D?

(A) Four

(B) Two

(C) One

(D) Three

(E) Cannot be determined

Answer: (D)

Directions- (Q. 26-30) Study the following information and answer the question.

  Eight boxes marked as F, G, H, I, J, K, L and M are kept on eight different floors of a building. The lower mot floor of the building is numbered one, the one above that is numbered two and so on till the topmost floor is numbered eight. Each box has a different shape viz. Cuboid, Spherical, Oval, Conical, Cylindrical, Pyramidal, heart-shaped and Star-shaped.

• F is kept on an odd numbered floor below floor number 5. Only three boxes are kept between F and the cuboid box.

• Only two boxes are kept between L and the oval box. L is kept on floor above the oval box. L is kept on an odd numbered floor. L is not cuboid is shape.

• As many boxes are kept between the cuboid and the oval boxes as between the cylindrical box and K. K is kept at one of the floors below the oval box. K is not kept on floor number 2. Only three boxes are kept between the cylindrical box and M.

• The spherical box is kept immediately below H. H is neither kept on floor number 4 nor 8. Only two boxes are kept between H and the heart-shaped box.

• As many boxes are kept between M and the heart-shaped box as between I and the star-shaped box. G is kept on one of the floors above I.

• The pyramidal box is kept at one of the floors above the conical box.

26. Which of the following is true based on the given information?

(A) Box F is kept on the lowermost floor.

(B) Box L is spherical in shape.

(C) Box L is kept on floor number 7.

(D) None of the given options is true

(E) No box is kept between the cuboid and the oval boxes.

Answer: (B)

27. How many floors are there between the floors on which J and F are kept?

(A) Two

(B) Four

(C) Five

(D) None

(E) One

Answer: (D)

28. I is kept on which of the following floor numbers?

(A) 7

(B) 1

(C) 4

(D) 2

(E) Other than the those given as options

Answer: (A)

29. Which of the following boxes is conical?

(A) J

(B) M

(C) G

(D) I

(E) K

Answer: (E)

30. What is the shape of box J?

(A) Oval

(B) Star-shaped

(C) Cylindrical

(D) Pyramidal

(E) Conical

Answer: (D)

31. Which of the following will come in the given series in the place of question mark?

p q r s 4 q r s t 4 5 r s t u 4 5 6 s t  u v 4 5 6 7 ?

(A) w

(B) r

(C) t

(D) u

(E) 8

Answer: (C)

Directions-(Q. 32-35) In this question, relationship between different elements is shown in the statement. The statements are followed by conclusions. Study the conclusions based on the given statement and select the appropriate answer.

(A) Only conclusion I follows

(B) Only conclusion II follows

(C) Both conclusion I and II follow

(D) Either conclusion I or II follows

(E) Neither conclusion I nor II follows

32. Statements : J ≥ L > W ≤ R = T; B ≤ W < A

Conclusion I : B < J

Conclusion II : T ≥ A

Answer: (A)

33. Statements : J ≥ L > W ≤ R = T; B ≤ W < A

Conclusion I : B < T

Conclusion II : B = T

Answer: (D)

34. Statements : T < R ≤ U ≥ C > M; R ≥ S; C < Z

Conclusion I : S > Z

Conclusion II : Z > M

Answer: (B)

35. Statements : T < R ≤ U ≥ C > M; R ≥ S; C < Z

Conclusion I: Z > T

Conclusion II : M < S

Answer: (E)

Directions- (Q. 36-40) Study the following information to answer the given question.

  Each of the seven people, J, K, L, M, N, O and P are seated in a straight line facing north with equal distance between each other. Each one of them owns are car from a different brands viz. Toyota, Hyundai, Tata, Tesla, Audi, BMW and Suzuki but not necessarily in the same order.

  M sits third to the right of Toyota owner. O sits second to the right of M. The Audi owner sits second to the left of P. P is neither an immediate neighbour of Toyota owner nor sits at any of the extreme ends of the line. As many people sit to the right of P as to the left of N. As many people sit between N and M as between K and the BMW owner. K neither owns as Audi nor a Toyota. Only two people sit between the BMW and the Hyundai owners. More than three people sit between J and the Tesla owner. More than one person sits between the Tesla and the Tata owners.

36. Which of the following is true with respect to L as per the given arrangement?

(A) L owns a Tesla

(B) Only one person sits between L and the BMW owner.

(C) None of the given options is true

(D) L sits at one of the extreme ends of the line

(E) L is an immediate neighbour of J

Answer: (B)

37. How many persons are seated between N and M?

(A) Two

(B) None

(C) One

(D) Three

(E) Four

Answer: (A)

38. What is the position of K with respect to the Tata owner?

(A) Third to the right

(B) Fourth of the right

(C) Fifth to the right

(D) Third to the left

(E) Second to the right

Answer: (E)

39. Car of which brand does K own?

(A) Tesla

(B) Suzuki

(C) Tata

(D) Hyundai

(E) Other than those given as options

Answer: (D)

40. Which of the following represents persons seated at the two extreme ends of the line?

(A) J and the Suzuki owner

(B) L and the Toyota owner

(C) The Audi owner and L

(D) K, O

(E) The Tesla and The Tata owners

Answer: (E)

41. Read the following information carefully and answer the question.

Most of the Japanese companies which invested almost 500 million dollars in country Zylland made losses in the last financial year to the extent of 250 million dollars.

“If all the Japanese companies makes losses in Zylland to such extent, it would deter any further investments in the near future.” An industry insider.

Which of the following statements weakens the industry insider’s view to some extent?

(A) More than 94% of the total investments by Japanese companies were made in Zylland’s e-retailer company ‘Netshop’which closed down last year.

(B) Companies from all other countries invested an average amount of 450 million dollar only in zylland.

(C) An economic downturn which hit the world economy two y ears ago had impacted Zylland adversely as well.

(D) The Japanese companies were well aware before investing in Zylland that average profit earned through investments in this country was only 12%.

(E) All Japanese companies which made losses in Zylland also made losses in the past in some of their endeavours in other countries as well.

Answer: (E)

42. Read the following information carefully and answer the question.

NetBart has agreed to provide free of cost, high-speed internet connection to all schools in Village A.

“This step would bring the computer literacy to 10% in Village A “-NetBart’s CEO.

Which of the following is an assumption made by the NetBart’s CEO in the given statement

(A) No school at present can afford a paid internet connection.

(B) NetBart may provide free internet connections to other villages as well.

(C) At present computer literacy in Village A is much lower than other village.

(D) NetBart will make substantial profits from this venture.

(E) Al schools in Village A can afford adequate number of computers.      

Answer: (E)

43. How many such pairs of letters are there in the word INVESTOR each of which has as many letters between them (in both forward and backward directions) as there are in English alphabetical order?

(A) One

(B) Two

(C) Three

(D) None

(E) More than three

Answer: (C)

Directions- (Q. 44-48) Study the given information carefully and answer the given question.

  Eight people A, B, C, D, L, M, N and O were born in one of the months-January, March, June and November of the same year but not necessarily in the same order. In each month these people were born on either 5th or 18th, with each person born on a different day.

  C has born on an even numbered date in a month having only 30 days. Only two of these eight people were born between C and N. L is younger than N. L was born on the 18th. Only three of these given people were born between B and O. B is older than O. D is older than B. Only one of the given persons has birthday between D and A.

44. Which of the following statements is true as per the given arrangement?

(A) M is the youngest amongst the given people.

(B) C was born in November.

(C) None of the given statements is true.

(D) N was born on 5th March.

(E) O has his birthday on one of the days between D and A.

Answer: (B)

45. Which of the following combinations is correct based on the given information?

(A) L-November

(B) L-March

(C) M-18th

(D) A-March

(E) D-5th

Answer: (D)

46. When was D born?

(A) 5th June

(B) 18th January

(C) 5th March

(D) 18th March

(E) 5th November

Answer: (B)

47. How many people have birthday between M and L?

(A) Cannot be determined

(B) Four

(C) Six

(D) Three

(E) Five

Answer: (B)

48. How many people are younger than O?

(A) Six

(B) None

(C) Three

(D) One

(E) Four

Answer: (D)

49. In a certain code language, COULD is coded as BPTMC. Following the same coding pattern, ‘FLOWS’ is written as ‘EMNXR’. How will ‘PRICE’ be written in the same code language?

(A) OSHBF

(B) OSHDD

(C) OSHDF

(D) QQJBF

(E) QQHDD

Answer: (B)

50. The positions of first and the sixth letters of the word NORMAL are interchanged; similarly, the positions of second and fifth letters and third and fourth letters are interchanged. In the new arrangement. Thus formed, how many letters are there between the letter which is third from the left and the letter which is second from the right, in the English alphabetical series?

(A) Three

(B) None

(C) One

(D) Two

(E) More than three

Answer: (C)

IBPS Bank Pos-MTs (Pre.) Examination Held on 14-7-2017 Quantitative Aptitude Question Paper With Answer Key

IBPS Bank Pos-MTs (Pre.) Examination Held on 14-7-2017 Quantitative Aptitude
IBPS Bank Pos-MTs (Pre.) Examination Held on 14-7-2017 Quantitative Aptitude Question Paper With Answer Key

IBPS Bank Pos/MTs (Pre.) Examination Held on 14-7-2017

Quantitative Aptitude

Directions- (Q. 1 to 5) When a number arrangement machine is given an input line of numbers, it arranges them following a particular rule.

  The following is an illustration of input and rearrangement. All the numbers are two digit numbers :

Input : 81 63 79 42 15 24 86 37 96 19

Step I : 15 19 81 63 79 42 24 86 37 96

Step II : 96 86 15 19 81 63 79 42 24 37

Step III: 24 37 96 86 15 19 81 63 79 42

Step IV: 81 79 24 37 96 86 15 19 63 42

Step V: 42 63 81 79 24 37 96 86 15 19

Step V is the last step of the above arrangement as the intended output of arrangement is obtained.

  As per the rules followed in the given steps, find the appropriate steps for the given input.

Input : 26 69 13 82 55 21 71 34 93 47

1. Which is the fourth element to the left of the seventh element from the left end in Step III of the given arrangement?

(A)  93

(B)  82

(C)  13

(D)  26

(E)  55

Answer: (A)

2. What will be the resultant if the fifth element from right end in Step V is subtracted from the second element from left end in Step II as per the given arrangement?

(A)  59

(B)  79

(C)  56

(D)  48

(E)  67

Answer: (D)

3. In which of the following steps ’26 69 55’ found consecutively in the same order as the given arrangement?

(A)  Only II

(B)  Both I and II

(C)  Both II and III

(D)  There is no such step

(E)  Only III

Answer: (A)

4. As per the given arrangement, in Step I ‘21’ is related to ‘69’ in a certain pattern. Following the same pattern, ‘26’ is related to ‘93’ in Step IV. To which of the following is ‘47’ related to following the same pattern in Step V?

(A)  71

(B)  21

(C)  34

(D)  69

(E)  55

Answer: (A)

5. How many elements appear to the right of ‘26’ in Step IV of the given arrangement?

(A)  Six

(B)  None

(C)  Two

(D)  Five

(E)  Seven

Answer: (E)

Directions- (Q. 6 to 8) Study the given information carefully to answer the given questions-

  D is the father of only F and E. D has only one son. E is married to G. G is the son-in-law of M. H is the only son of G. K and J are the children of E. L is married to K.

6. How is M related to J?

(A)  Aunt

(B)  Grandmother

(C)  Mother

(D)  Niece

(E)  Granddaughter

Answer: (B)

7. How is H related to L?

(A)  Brother

(B)  Brother-in-law

(C)  Son-in-law

(D)  Nephew

(E)  Uncle

Answer: (B)

8. How is F related to K?

(A)  Grandfather

(B)  Either ‘brother’ or ‘sister’

(C)  Aunt

(D)  Either ‘nephew’ or ‘niece’

(E)  Uncle

Answer: (E)

Directions- (Q. 9 and 10)In this question, relationship between different elements is shown in the statements. The statements are followed by two conclusions. Study the conclusions based on the given statements and select the appropriate answer.
(A) Only conclusion I follows

(B) Either conclusion I or II follows

(C) Neither conclusion I nor II follows

(D) Only conclusion II follows

(E) Both conclusion I and II follows

9. Statements:

B ≤ O = L ≤ D; P ≥ C ≥ A = L

Conclusions: I. P = B       II. B < P

Answer: (B)

10. Statements:

P ≥ O ≤ L < E; S ≤ O ≥ A = K

Conclusions: I. E < S       II. K ≤ P

Answer: (D)

Directions-(Q. 11 to 15) Read the given information to answer the given questions-

  Eight people viz., A, B, C, D, E, F, G and H are sitting around a square table in such a way that four of them sit at four corners of the table while four sit in the middle of each of the four sides, but not necessarily in the same order. The ones sitting in the middle of the sides are facing the centre and the ones sitting at the corners of the table are facing outside (i.e., opposite to the centre).

  A sits in the middle of one of the sides. C is an immediate neighbour of A. E sits second to the left of C. Only three people sit between E and G. Only two people sit between G and D (either from left or right). D is not an immediate neighbour of C. B sits second to the right of D. F sits second to the right of B.

11. How many people sit between C and H when counted from the left of H?

(A)  None

(B)  Two

(C)  One

(D)  More than three

(E)  Three    

Answer: (E)

12. Which of the given statements is not true as per the given arrangement?

(A)  G sits at one of the corners of the table

(B)  All the given statements are true

(C)  Only three people sit between F and D 

(D)  B and F face the centre

(E)  E sits second to the right of F

Answer: (E)

13. Which of the following pairs represent the people sitting between H and F, when counted from the left of F?

(A)  G, B

(B)  C, G

(C)  C, E

(D)  A, D

(E)  B, D

Answer: (A)

14. As per the given arrangement, four of the following five are alike in a certain way and thus form a group. Which one of the following does not belong to that group?

(A)  C

(B)  D

(C)  H

(D)  G

(E)  E

Answer: (B)

15. Who sits third to the left of B?

(A)  G

(B)  A

(C)  E

(D)  H

(E)  C

Answer: (C)

Directions – (Q. 16 to 20) Read the given information to answer the given questions.

  Eight people viz. P, Q, R, S, T, U, V and W are sitting in a straight line facing north with equal distance between each other. Each of them teaches a different subject viz. Chemistry, History, Mathematics, English, Physics, Biology, Geography and Social Science.

(Note : None of the given information in necessarily in the same order.)

  P sits third from the left end of the line. Only one person sits between P and the one who teaches English. Only three people sit between the one who teaches English and R. As many people sit to the right of R as to the left of the one who teaches Social Science. Only three people sit between the one who teaches Social Science and W. No one sits between W and the one who teaches History. As many people sit to the left of the one who  teaches History as to the right to T. Only One person sits between T and V. The one who teaches Mathematics sits to the immediate right of the one who teaches Physics. The one who teaches Mathematics is an immediate neighbour of V. Only one person sits between Q and the one who teaches Mathematics. More than four people sit between Q and the one who teaches Chemistry. U sits to the immediate left of S. U does not teach Geography.

16. How many people sit to the right of U?

(A)  More than three

(B)  None

(C)  Two

(D)  Three

(E)  One

Answer: (C)

17. Four of the following five are alike in a certain way based on the given arrangement and thus form a group. Which one of the following does not belong to that group?

(A)  Q-Physics

(B)  T-English

(C)  S-Chemistry

(D)  W-History

(E)  P-Biology

Answer: (E)

18. Which of the following is true about the one who teaches Geography as per the given arrangement?

(A)  The one who teaches Geography sits second to the left of P

(B)  Only one person sits between T and the one who teaches Geography

(C)  V sits to the immediate left of the one who teaches Geography

(D)  None of the given statements is true

(E)  Less than two people sit between the one who teaches Geography and English

Answer: (C)

19. What is the position of the one who teaches Chemistry with respect to V?

(A)  Immediate right

(B)  Fourth to the right   

(C)  Third to the right

(D)  Third to the left

(E)  Fourth to the left

Answer: (B)

20. Who sits third to the left of S?

(A)  T

(B)  The one who teaches Mathematics

(C)  P

(D)  The one who teaches Biology

(E)  The one who teaches Social Science

Answer: (E)

Directions- (Q. 21 to 25) Study the given information to answer the given questions-

  In a certain code language,

  ‘with all caring friends’ is written as ‘bs up ck lq’

  ‘all the time dancing’ is written as ‘av tn lq og’

  ‘friends daning and singing’ is written as ‘tn dz up ry’

  ‘caring and loving people’ is written as ‘dz xn bs eh’

  (Note: All the codes are two letter codes only)

21. How many ‘all singing song’ be coded as in the given code language?

(A)  ry sf lq

(B)  ry bs up

(C)  av lq ry

(D)  sf tn ry

(E)  ry lq eh

Answer: (A)

22. What is the code for ‘time’ in the given code language?

(A)  Either ‘lq’ or ‘tn’

(B)  up

(C)  tn

(D)  Either ‘av’ or ‘og’

(E)  Either ‘xn’ or ‘lq’

Answer: (D)

23. If ‘people dancing together’ is coded as ‘tn gi xn’, then how will ‘loving friends together’ be coded as in the given code language?

(A)  av dz up

(B)  eh gi dz

(C)  gi up lq

(D)  eh dz up

(E)  gi up eh

Answer: (E)

24. What does the code ‘ck’ stand for in the given code language?

(A)  friends

(B)  all

(C)  caring

(D)  singing

(E)  with

Answer: (E)

25. What does ‘bs dz’ stand for in the given code language?

(A)  and caring

(B)  and with

(C)  caring with

(D)  with the

(E)  and the

Answer: (A)

Directions- (Q. 26 to 30) Read the given information to answer the given questions-

  Seven people viz., T, U, V, W, X, Y and Z studies in three branches of an engineering college viz. Aeronautical, Chemical and Electrical. Each of them also likes a different sport viz. Hockey, Cricket, Football, Volleyball, Boxing, Archery and Wrestling. Atleast two people study in each branch.

(Note: None of the given information is necessarily in the same order)

  T studies in Aeronautical with only the one who likes Archery. The one who likes Wrestling studies with the one who likes Hockey. U studies with W and the one who likes Cricket. Neither U nor W like either Wrestling or Hockey. Z studies with the one who likes Boxing. Neither T nor U like Boxing. Y studies with V. V neither studies Electrical engineering nor likes Wrestling. T does not like Volleyball.

26. Who amongst the following studies Chemical Engineering?

(A)  Y

(B)  U

(C)  The one who likes Volleyball

(D)  W

(E)  The one who likes Cricket

Answer: (A)

27. Which of the following is true as per the given arrangement?

(1) Y likes Cricket.

(2) X studies Aeronautical Engineering.

(3) The one who likes Boxing studies Electrical Engineering.

(A)  Only 3

(B)  Only 1

(C)  Both 1 and2

(D)  Only 2

(E)  Both 2 and 3

Answer: (E)

28. Which sport does X likes?

(A)  Wrestling

(B)  Archery

(C)  Boxing

(D)  Volleyball

(E)  Cricket

Answer: (B)

29. Four of the following five study in the same branch based on the given arrangement and thus form a group. Who amongst the following study in a different branch as per the given arrangement?

(A)  T-X

(B)  Football-Archery

(C)  Wrestling-Boxing

(D)  Y-Hockey

(E)  W-Volleyball

Answer: (C)

30. Who amongst the following likes Football?

(A)  T

(B)  U

(C)  V

(D)  W

(E)  Y

Answer: (A)

Directions- (Q. 31 to 35) Read the given information to answer the given questions-

  Seven people viz. J, K, L, M, N, O and P advertize for a brand viz. Fastrack, Biba, Nike, Puma, Marigo, Vivo and Zara in seven different months of the same year viz. February, April, May, June, August, September and November.

(Note: None of the given information is necessarily in the same order. No one advertises in any other month of the given year.)

  Only two people advertise between P and the one who advertises for Puma. P advertises in a month having 31 days. More than three people advertise between the one who advertises for Puma and the one who advertises for Biba. Only three people advertise between the one who advertises for Biba and K. No one advertises between K and the one who advertises for Marigo. Only three people advertise between O and L. L advertises in one of the months before O. The one who advertises for Fastrack advertises in a month immediately before the one who advertise for Nike. Only two people advertise between N and the one who advertises for Vivo. Only one person advertises between M and the one who advertises for Zara. M advertises in one of the months before June.

31. Who advertises in the month of April?

(A)  The one who advertises for Fastrack

(B)  N

(C)  The one who advertises for Puma

(D)  J

(E)  M

Answer: (C)

32. How many people advertise between M and the one who advertises for Nike?

(A)  Three

(B)  One

(C)  More than three

(D)  Two

(E)  None

Answer: (C)

33. Advertisement for Marigo is done in which month?

(A)  April

(B)  September

(C)  August

(D)  June

(E)  May

Answer: (D)

34. As per the given arrangement, L is related to the one who advertises for Zara following a certain pattern. In the same pattern, N is related to the one who advertises for fastrack. Following the same pattern to who amongst the following is O related to?

(A)  Vivo

(B)  Puma

(C)  Marigo

(D)  Biba

(E)  Nike

Answer: (D)

35. Which of the given combinations is correct as per the given arrangement?

(A)  J-Zara

(B)  L-May

(C)  N-Fastrack

(D)  M-Vivo

(E)  O-November

Answer: (D)

IBPS Specialist Officers IT (Pre.) Examination Held on 30-10-2017 Reasoning Question Paper With Answer Key

IBPS Specialist Officers IT (Pre.) Examination Held on 30-10-2017 Reasoning
IBPS Specialist Officers IT (Pre.) Examination Held on 30-10-2017 Reasoning Question Paper With Answer Key

IBPS Specialist Officers IT (Pre.) Examination Held on 30-10-2017

Reasoning

Directions- (Q. 1 to 3) This question consists of a question and two statements numbered I and Ii given below it. You have to decide whether the data given in the statements are sufficient to answer the question. Read both the statements and choose the most appropriate option and answered as-

(A) Only statement I is sufficient to answer the question while statement II is not

(B) Only statement II is sufficient to answer the question while statement I is not

(C) Statement I and statement II together are necessary to answer the question

(D) Either statement I alone are sufficient to answer the question or statement II alone are sufficient to answer the question

(E) Neither statement I nor statement II are sufficient to answer the question

1. How many students are there in class 8 in school K?

(I) The class teacher of class 8 correctly remembers that the number of students in her class is more than 21but less than 32. The number of students in her class is exactly divisible by 6.

(II) The principal of school K correctly remembers that the difference between the number of students in class 7 and class 8 is less than nine. There are 33 students in class 7.

Answer: (C)

2. Among A, B, C, D and E, seated around a circular table, facing the centre, who sits second to the right of C?

(I) C sits to the immediate right of A. Only two person sits between B and A (when counted from left to right).

(II) Both D and C are immediate neighbours of B. E sits second to the right of B.

Answer: (C)

3. Amongst five bags, L , M, N, O and P, each having a different weight, which is the lightest?

(I) Only two bags are heavier than O. O is heavier than P but lighter than M.

(II) N is lighter than O but heavier than P. L is heavier than O, but not the heaviest.

Answer: (B)

Directions  (Q. 4 to 8) In this questions, relationship between different elements is shown in the statements. The statements are followed by conclusions. Study the conclusions based on the given statements and select the appropriate answer.

(A) Only conclusions I follows

(B) Only conclusion II follows

(C) Either conclusion I or II follows

(D) Neither conclusion I nor II follows

(E) Both conclusion I and II follows

4. Statements : P ≥ R > O = S ≤ A = Q

Conclusions: I. P > S       II. O ≤ Q

Answer: (E)

5. Statements : R ≤ A < C = E ≤ D

Conclusions: I. R < D      II. R = D

Answer: (A)

6. Statements: V ≥ E = N ≥ I ≥ C > E

Conclusions: I. C ≤ V      II. N > E

Answer: (A)

7. Statements: C ≤ D < L ≤ F ≥ G; K ≥ L > M

Conclusions : I. C < M    II. K ≥ G

Answer: (D)

8. Statements: J ≤ L ≤ P ≤ E; K ≥ P > M

Conclusions: I. M > E      II. J ≤ K

Answer: (B)

9. Read the following information carefully and answer the question which follows.

‘A variety of duplicate products have snatched our market. Our profits are reducing by almost 10% each passing year. Despite much longer durability of our handmade products, people these days are settling for the second copies only because of their prices and easy availability”-statement by the owner of a handmade products store in City Y. Which of the following can be inferred from the owner’s statements?

(An inference is something by which you can logically deduce something to be true based).

(A)  The market of handmade products will dissolve completely in the coming few years

(B)  Manufacturing of second copies or duplicate articles is recent trend in City Y

(C)  Reducing the price of handmade articles by 10% will attract more customers

(D)  Making handmade products more widely available at reduced prices will help its market revive atleast to some extent

(E)  Durability is the least important factor that affects the choice of customers while buying such items

Answer: (B)

Directions-(Q. 10 to 14) Study the given information carefully to answer the given question.

In a certain code language.

  ‘give me your number’ is coded as ‘tm ct ar bp’

  ‘highest number in English’ is coded as ‘bp dg sk mi’

  ‘test your English skills’ is coded as ‘tm fo ve dg’

  ‘skills in pencil sketches’ is coded as ‘nu ve ky sk’

  (Note: All codes are two letter codes only)

10. What is the code for ‘English’ in the given code language?

(A)  ve

(B)  tm

(C)  nu

(D)  ky

(E)  dg

Answer: (E)

11. What does the code ‘ct’ stand for in the given code language?

(A)  test

(B)  Either ‘give’ or ‘me’

(C)  Either ‘your’ or ‘number’

(D)  number

(E)  highest

Answer: (B)

12. What may be the possible code for ‘your hidden skills’ in the given code language?

(A)  mi tm fo

(B)  ve fo tm

(C)  ve rb sk

(D)  ve rb tm

(E)  rb tm ky

Answer: (D)

13. If in the given code language, ‘colour of pencil’ is coded as ‘nu xg hf’, what will be the code for ‘number of sketches’?

(A)  bp xg ve

(B)  hf fo xg

(C)  fo tm hf

(D)  Cannot be determined

(E)  ky tm nu

Answer: (D)

14. What does the code ‘fo’ stand for in the given code language?

(A)  None of the given options

(B)  your

(C)  test

(D)  skills

(E)  English

Answer: (C)

Directions- (Q. 15 to 18) Study the given information carefully to answer the given question.

  A certain number of people are sitting in a straight line facing north. B sits fifth to the right of K. Only two people sit between K and S. M sits seventh to the left of S.R sits at one of the positions to the right of M. The number of people between S and B is one less than that between M and R. Q sits at one of the positions between S and R. Q is not an immediate neighbour of K. L sits seventh to the right of Q. L sits at an extreme end. As many people sit between L and B as to the left of M.

15. Who sits second to the left of B?

(A)  S

(B)  R

(C)  L

(D)  M

(E)  Other than those given as options

Answer: (A)

16. Which of the following represents the positions of R in the given line?

(A)  Exactly between Q and K

(B)  Second to the right of S

(C)  Eighth from the right end of the line

(D)  Sixth to the left of L

(E)  Seventh from the left end of the line

Answer: (E)

17. Which of the following statements is true as per the given information?

(A)  K is an immediate neighbour of L

(B)  None of the given statements is due

(C)  Only six people sit between B and L

(D)  Only one person sits to the left of M

(E)  More than three people sits between M and R

Answer: (B)

18. How many people are sitting in the given line?

(A)  17

(B)  20

(C)  15

(D)  22

(E)  Cannot be determined

Answer: (A)

Directions- (Q. 19 to 23) Study the given information carefully to answer the given question.

  Twelve people are sitting in two parallel rows containing six people each, in such a way that there is equal distance between adjacent persons. In row-1 A, B, C, D, E and F are seated (not necessarily in the same order) and all of them are facing north. In row-2 P, Q, R, S, T and U are seated (not necessarily in the same order) and all of them are facing south. Therefore, in the given seating arrangement, each member seated in a row faces another member of the other now.

  B sits third to the right of A. The one who faces A sits second to the right of P. R sits second to the left of P. Only two people sit between C and the one who faces R. As many people sit to the right of E as to the left of C. D faces U. Only one person sits between U and T. The one who faces F sits second to the left of Q.

19. Which of the given statements is TRUE with respect to the given information?

(i) C faces G

(ii) Its at an extreme end

(iii) D faces an immediate neighbour of R

(A)  Only (iii)

(B)  All (i), (ii) and (iii)

(C)  Both (ii) and (iii)

(D)  Both (i) and (ii)        

(E)  Only (ii)

Answer: (A)

20. Who amongst the following faces T?

(A)  B

(B)  D

(C)  E

(D)  C

(E)  A

Answer: (A)

21. If all the people of Row-1 are made to sit in alphabetical order from left to right, then who amongst the following will face P as per the new arrangement?

(A)  F

(B)  A

(C)  D

(D)  C

(E)  B

Answer: (D)

22. As per the given arrangement, B is related to Q in the same way as P is related to D. Following the same pattern, to whom is E related?

(A)  U

(B)  The one who faces A

(C)  S

(D)  The one who faces D

(E)  T

Answer: (C)

23. Who among the following does not sit at any of the extreme end of the rows?

(A)  The one who faces A

(B)  The one who sits to the immediate left of C

(C)  U

(D)  R

(E)  D

Answer: (D)

Directions- (Q. 24 to 28) Study the given information carefully to answer the given question.

  Eight people D, E, F, G, R, S, T and U are sitting around a square table facing the centre. Four people are sitting at the corners and four in the middle of the sides of the table. The ages of the ones sitting in the middle of the sides are multiples of 3 and that of those sitting at the corners are multiples of 7.

  D sits second to the right of the 36 year old. One of the immediate neighbours of D is 28 years old. Only three people sit between E and the 28 year old. The age of the one who sits second to the left of E is less than 15 and also a multiple of 2. G is an immediate neighbour of the 51 year old. G is neither 14 nor 28 year old.Only one person sits between G and U (when counted from left or right). F is 27 years old. Only three people sit between F and T. The one who sits to the immediate left of S is 49 years old. G’s age is less than 10 years. The difference between ages of G and D is 11.

24. How many people sit between T and the 51 year old, when counted from the left of T?

(A)  One

(B)  None

(C)  More than three

(D)  Two

(E)  Three

Answer: (A)

25. How many people are younger than D?

(A)  Two

(B)  More than four

(C)  Four

(D)  Three

(E)  One

Answer: (A)

26. Which of the following statements is true as per the given arrangement?

(A)  T is 51 years old

(B)  None of the given statements is true

(C)  F is an immediate neighbour of U

(D)  The differences of the ages of E and F is 8

(E)  T sits to the immediate right of the 7 year old

Answer: (E)

27. Who amongst the following is 14 years old?

(A)  None of the given options

(B)  E

(C)  T

(D)  U

(E)  R

Answer: (E)

28. What is the age of D (in years)?

(A)  48

(B)  15

(C)  18

(D)  27

(E)  9

Answer: (C)

29. Study the given information carefully to answer the given question.

Bus no. 218 is the only bus which plies on the only route between localities A and G of city Haroya. There are three huge factories between these areas. Though the bus has always been very punctual and well maintained, last week, it was decided to discontinue the bus for certain reason.

Which of the following cannot be reason behind discontinuing Bus no. 218?

(A)  Soon AC buses will start playing between localities A and G, whose fare will be just 2% more than that of Bus no. 218. It is anticipated that Bus 218 will not make much profit thereafter

(B)  All the given options can be possible reasons for discontinuing the bus

(C)  It has been decided to increase the number of e-ricksh was playing between areas A and G by 40% hence making them more easily and frequently available to the customers as compared to the bus. Also, as a result, the additional pollution caused by buses in these areas can brought down to the great extent

(D)  Under the recently completed metro rails project of Haroya, there are more frequent train between localities A and G as compared to the frequency of Bus no. 218. The metro fare is also lower than that of the bus.

(E)  The number of chain snatching and pick-pocketing cases near locality G of Haroya have drastically increased. As a result, Bus no. 218 has considerably lesser passengers now.

Answer: (B)

30. This consist of a decision and two statements numbered I and II given below it. You have to decide which of the given statements weaken/s or strengthen/s the decision and mark the appropriate answer.

Decision : ‘Health-Wise’ is a chain of stores of healthcare products in country K. It has decided to keep products on only 20 companies in its store from now onwards.

(I) These 20 companies have been associated with ‘Health-Wise’ for almost a decade and hence the time between the supply and demand of products is very less.

(II) 90% of the 20 listed companies have been awarded as the best healthcare products providing companies of the country.

(A)  Both statement I and II are neutral statements

(B)  Statements I weakens the decision and while statement II strengthens the decision

(C)  Both statement I and II weaken the decision

(D)  Statement I strengthens the decision while statement II weakens the decision

(E)  Both statement I and II strengthen the decision

Answer: (E)

31. Study the given information carefully to answer the given question.

Despite giving repeated waning to the science students of School, Z, it has been found in this academic year that students who have not opted for Biology frequently enter the Biology Lab without permission. A few of such students were also found to have damaged the microscopes in the laboratory.

Which of the given two options can be a feasible course of action for the given situation?

(I) Biology students must be given additional ID cards specifically to enter the laboratory. Any student without the card should strictly not be allowed inside the lab unless specified by the teacher.

(II) Only the teacher should be allowed to touch or use the microscopes.

(A)  Only Ii is a feasible course of action

(B)  Both I and II are feasible course of actions

(C)  Either I or II is a feasible course of action

(D)  Neither I nor II is a feasible course of action

(E)  Only I is a feasible course of action

Answer: (E)

Directions- (Q. 32 to 35) In this question three statements followed by two conclusions numbered I and II given below it. You have to take the given statements to b e true even if they seem to be at variance from the commonly known facts and then decide which of the given conclusions logically follows from the given statements and answered as-

(A) Only conclusion I is true

(B) Only conclusion II is true

(C) Both conclusion I and II are true

(D) Neither conclusion I nor conclusion II is true

(E) Either conclusion I or II is true

32. Statements : Some juices are fruits. No fruit is a potato. Some potatoes are cakes.

Conclusions:

(I) Some cakes are fruits.

(II) No fruit is a cake.

Answer: (E)

33. Statements : Some juices are fruits. No fruit is a potato. Some potatoes are cakes.

Conclusions:

(I) All cakes being juices is a possibility.

(II) All juices can never be potatoes.

Answer: (B)

34. Statements : All sketches are paintings. Some paintings are drawings. All drawings are letters.

Conclusions:

(I) All paintings can never be letters.

(II) Atleast some sketches are drawings.

Answer: (D)

35. Statements: All cubs are kittens. No kitten is a zebra. All zebras are bears.

Conclusions:

(I) All zebras being cubs is a possibility.

(II) No kitten is a bear.

Answer: (B)

36. What will come in place of the question mark (?) in the given series?

AZ-4 BY-6 DW-10 GT-16 ?

(A)  KP-20

(B)  JO-20

(C)  KN-24

(D)  KP-24

(E)  JO-22

Answer: (D)

37. In a straight line of six people (all facing north), Q sits at an extreme end of the line. Only two people sit between Q and G. R sits second to the right of G. L sits third to the right of A. Only one person sits between Q and B. Who sits to the immediate right of B?

(A)  No one as B sits at the extreme right end

(B)  L

(C)  G

(D)  A

(E)  Cannot be determined

Answer: (C)

38. Five boxes A, B, C, D and E are stacked above one another but not necessarily in the same order. Only one box is kept between A and B. C is kept at one of the positions above B but not at the top of the stack. Only two boxes are kept between C and D. Which of the following box is kept second from the bottom of the stack?

(A)  D

(B)  B

(C)  A

(D)  C

(E)  Cannot be determined

Answer: (D)

39. In the word PATRONISE, the letter immediately before each vowel is replaced with the next alphabet (as per the English alphabetical order) and all others remain unchanged. Which of the following letters will appear twice in the word thus formed?

(A)  Only T

(B)  Both T and O

(C)  Both S and O

(D)  Only Q

(E)  Only E

Answer: (B)

40. If it is possible to make only one meaningful English word with the fourth, the eighth, the ninth and the eleventh letters from the left of the word EXAMINATION using all the letter but each letter only once, which would be the second letter of the world from the left end ? (If more than one such word can be formed, given ‘Z’ as your answer. If no such word can be formed, ,given ‘Y’ as your answer.

(A)  Z

(B)  T

(C)  Y

(D)  M

(E)  I

Answer: (E)

Directions- (Q. 41 to 45) Study the given information carefully to answer the given question.

  Seven people A, B, F, G, L, M and X were born on seven different days of the same week starting from Monday and ending on Sunday. Each one of them works in different companies-Dell, Capegemini, Convergys and IBM. Atleast one person works in each of the given companies.

  A was born on Thursday. Only two people were born between F and A. F works for Capegemini. One of the persons who works for IBM was born on Tuesday. Only two people were born between L and the one born on Tuesday. X works for Dell, and was born on one of the days after L. The person born on one of the days after L. The person born immediately after X works for Capegemini. One of the persons who works for Covergys was born immediately before M. Only two people were born between B and one of the persons who work for the same company. One of the persons born after G works for IBM.

41. Who amongst the following was born on Monday?

(A)  L

(B)  F

(C)  None of the given options

(D)  One of the persons who work for Capegeminin

(E)  One of the persons who work for IBM

Answer: (C)

42. In which of the given companies, does only one person work?

(A)  None

(B)  Only Convergys       

(C)  Both Capegemini and Convergys

(D)  Both Dell and Capegemini

(E)  Only Dell

Answer: (D)

43. X was born on which of the following days?

(A)  Friday

(B)  Wednesday

(C)  Tuesday

(D)  Other than those given as options

(E)  Saturday

Answer: (E)

44. Which of the following represents the people who work or IBM?

(A)  L, B, A

(B)  G, M

(C)  A, M

(D)  G, L, M

(E)  L, G

Answer: (D)

45. How many people were born between M and the one who works for Capegemini?

(A)  Two

(B)  Three

(C)  One

(D)  More than three

(E)  None

Answer: (D)

46. Study the given information to answer the given question.

A very famous play ‘Jhankaar’ was set to be staged in one of the biggest theatres of city X on 27th of December. The theatre usually closes booking of its tickets two days before the play. Despite all promotions and large scale advertisements, the theater sold only 70% of his tickets for the play by 25th December.

Which of the following can be an effect of given situation?

(I) The theatre will not give its premises to the director of ‘Jhankaar’ to stage any of his future plays.

(II) The theatre can decide to remain open for booking tickets till a few hours before the play.

(A)  Either I or II can be an effect

(B)  Only II can be an effect

(C)  Neither I nor II can be an effect

(D)  Both I and II can be effects

(E)  Only I can be an effect

Answer: (B)

Directions- (Q. 47 to 50) Study the given information carefully to answer the given question.

  Point A is 12 m to the north of Point B. Point C is 8 m to the east of Point B. Point D is 4 m south of Point C.

  Madhav who is standing at Point A, walks 10 m towards west, takes left turn and walks for 14 m to reach Point  S. He takes a left turn again, walks for 5 m and stops at Point Q. Aditya who is standing at Point D walks 6 m towards west, takes a right turn, walks for 2 m and stops at Point Y. Point Z is 7 m away from Points S.

47. In which direction is Point S with respect to Point D?

(A)  North-West

(B)  West

(C)  South-West

(D)  North-East

(E)  East

Answer: (A)

48. If point L is to the north of Point B such that point Z and Point L form a horizontal straight line, then which of the following will be true?

(A)  Distance between points A and L is 7 m

(B)  Point L is to the north west of Point D

(C)  Point Y is to the south east of Point Z

(D)  Point Z is to the north of Point S

(E)  All the given statements are true

Answer: (E)

49. What is the distance between Point Q and Point Y?

(A)  5 m

(B)  13 m

(C)  7 m

(D)  Cannot be determined

(E)  9 m

Answer: (C)

50. If 2 is added to all odd digits of the number 4738261 and 1 is subtracted from all the even digits, then how many digits will be greater than 5 in the new number thus formed?

(A)  Five

(B)  Two

(C)  Four

(D)  Three

(E)  One

Answer: (B)

IBPS Specialist Officers IT (Pre.) Examination Held on 30-10-2017 Quantitative Aptitude Question Paper With Answer Key

IBPS Specialist Officers IT (Pre.) Examination Held on 30-10-2017 Quantitative Aptitude
IBPS Specialist Officers IT (Pre.) Examination Held on 30-10-2017 Quantitative Aptitude Question Paper With Answer Key

IBPS Specialist Officers IT (Pre.) Examination Held on 30-10-2017

Quantitative Aptitude

Directions- (Q. 1-5) Refer to the graph and answer the given questions.

1. The total number of people who travelled by both the given trains together on Tuesday is what per cent more than the total number of people who travelled by both the given trains together on Thursday?

(A)  29

(B)  25.5

(C)  31

(D)  27.5

(E)  28.4

Answer: (D)

2. What is the average number of people who travelled by train P on Sunday, Tuesday, Wednesday and Thursday?

(A)  225

(B)  235

(C)  215

(D)  195

(E)  205

Answer: (A)

3. The number of people who travelled by Train Q on Friday is 20% more than the number of people who travelled by the same train on Thursday. What is the respective ratio between the number of people who travelled on Friday and those who travel led on Saturday by the same train?

(A)  6 : 11

(B)  6 : 7

(C)  5 : 7

(D)  3 : 5

(E)  5 : 9

Answer: (B)

4. The number of people who travelled by Train P decreased by what per cent from Monday to Wednesday?

(A)   

(B) 

(C)   

(D)   

(E)   

Answer: (C)

5. What is the difference between the total number of people who travelled by Train P on Monday and Sunday together and the total number of people who travelled by Train Q on the same days together?

(A)  10

(B)  20

(C)  30

(D)  50

(E)  40

Answer: (E)

6. The respective ratio of curved surface area and total surface area of a right circular cylinder is 3 : 5. If the curved surface area of the right circular cylinder is 1848 cm2, what is its height? (in m)

(A)  24

(B)  14

(C)  21

(D)  28

(E)  18

Answer: (C)

7. Jar A and Jar B both contain mixture of milk and water. Jar A has 80 litres of mixture out of which 20% is water. The mixture in Jar B has 40% of water. The mixture from both the jars is poured in an empty Jar C. The resultant respective ratio between milk and water in the Jar C is 5 : 2. What is the quantity of milk in Jar C? (in litres)

(A)  105

(B)  120

(C)  110

(D)  100

(E)  130

Answer: (D)

8. What is the difference between total number of female students in institutions C and D together and number of students (both male and female) in institute E?

(A)  415

(B)  414

(C)  424

(D)  418

(E)  294

Answer: (E)

9. Number of male students in institute E is approximately what per cent more than the number of female students in institute A?

(A)  80

(B)  68

(C)  60

(D)  75

(E)  55

Answer: (B)

10. Number of female students in institute B is what per cent of the number of male students in Institute D?

(A)   

(B)   

(C)   

(D)   

(E)   

Answer: (E)

11. What is the central angle corresponding to number of students (both male and female) in institute A?

(A)  55.2°

(B)  51.2°

(C)  57.6°

(D)  63.4°

(E)  61.6°

Answer: (C)

12. 2/3rd of the number of students (both male and female) in institute C are science graduate. If the number of female science graduate students in institute C is 68, what percentage of male students in institute C are science graduates?

(A)  30

(B)  55

(C)  45

(D)  35

(E)  57

Answer: (E)

13. What is the average number of male students in institute A, B and E?

(A)  382

(B)  390

(C)  394

(D)  388

(E)  396

Answer: (B)

14. Number of students (both male and female) in institute E increased by 25% from 2011 to 2012, if the respective ratio of number of male and female students in 2012 in institute E is 5 : 6, what is the number of female students institute E in 2012?

(A)  300

(B)  324

(C)  420

(D)  380

(E)  405

Answer: (E)

Directions – (Q. 15-19) Study the given table carefully to answer the questions that follow-

15. What is the total number of men and children staying in locality I together?

(A)  4115

(B)  4551

(C)  4515

(D)  4155

(E)  None of these

Answer: (C)

16. The number of women staying in which locality is the highest?

(A)  H

(B)  I

(C)  F

(D)  G

(E)  None of these

Answer: (D)

17. What is the total number of children staying in localities H and I together-

(A)  1287

(B)  1278

(C)  1827

(D)  1728

(E)  None of these

Answer: (B)

18. What is the respective ratio of number of men staying in locality F to the number of men staying in locality H?

(A)  517 : 416

(B)  403 : 522

(C)  416 : 517

(D)  522 : 403

(E)  None of these

Answer: (A)

19. Total number of people staying in locality J forms approximately what per cent of the total number of people staying in locality F?

(A)  81

(B)  72

(C)  78

(D)  93

(E)  87

Answer: (E)

20. 8 8  11  19  34  ?

(A)  49

(B)  73

(C)  51

(D)  58

(E)  62

Answer: (D)

21. 80 1  86.3  92.6  ?   111.5

(A)  102.3

(B)  101

(C)  99

(D)  98.2

(E)  100.4

Answer: (B)

22. 16 7  6  8  15  ?

(A)  42

(B)  38

(C)  55.5

(D)  44.8

(E)  38.5

Answer: (E)

23. 3 4  9  28  113  ?

(A)  462

(B)  566

(C)  585

(D)  558

(E)  36.5

Answer: (B)

24. 126 62  30   14  ?

(A)  3

(B)  4

(C)  6

(D)  8

(E)  7

Answer: (C)

Direction- (Q. 25-27) Each of the questions below consists of a question and two statements numbered I and II given below it. You have to decide whether the data provided in the statements are sufficient to answer the question. Read both the statements and Given Answers :

(A) If the data in Statement I alone are sufficient to answer the question, while the data in Statement II alone are not sufficient to answer the question.

(B) If the data in Statement II alone are sufficient to answer the question, while the data in Statement I alone are not sufficient to answer the question.

(C) If the data in Statement I alone or in Statement II alone are sufficient to answer the question.

(D) If the data in both the Statements I and II are not sufficient to answer the question.

(E) If the data in both the Statements I and II together are necessary to answer the question.

25. Equal sums have been invested in schemes A and B for two years. Scheme A offers simple interest while scheme B offers compound interest (compounded annually. Both the schemes have equal rate of interest (p.c.p.a.). What is the rate of interest (p.c.p.a.) offered by each of the schemes?

(I) The interest earned from scheme A per annum on the given sum is Rs 545.

(II) The interest earned on scheme B after two years is 65.40 more than the interest earned from scheme A after two years.

Answer: (E)

26. What is the cost price of the table?

(I) The profit earned when the table is sold for Rs 560 is double the loss incurred when the same table is sold for Rs 320.

(II) The market price of the table is 40% more than the cost price. If a discount of 30% is given on the marked price, loss incurred is Rs 8.

Answer: (C)

27. How much time will Train M take to cross Train N (from the moment they meet) running in opposite direction (towards each other)?

(I)  Train M can cross a signal pole in 10 seconds it can cross 420 m long station in 25 seconds.

(II) The respective ratio of speeds of Train M and Train N is 5 : 3. The sum of the lengths of Train M and Train N is 575 m.

Answer: (E)

28. The circumference of a circular field is 20 m less than the perimeter of square field. If the radius of the circular field is 9 m less than the side of the square field, what is the cost of graveling the circular field @ Rs 50 per m sq.?

(A)  Rs 10,200

(B)  Rs 7,700

(C)  Rs 8,342

(D)  Rs 6,500

(E)  Rs 12,920

Answer: (B)

29. Present age of Ram is equal to Shyam’s age 8 years ago, the respective 6 years hence, the respective ratio between Shyam’s age and Ram’s age will be 6 : 5 at that time. What is Ram’s present age?

(A)  40 years

(B)  24 years

(C)  28 years

(D)  38 years

(E)  34 years

Answer: (E)

30. 600.16 × √? + 60.05 × √63 = 2280

(A)  9

(B)  49

(C)  81

(D)  121

(E)  25

Answer: (A)

31. 59.99% of 500.18 + 60.97 = ?2

(A)  11

(B)  9

(C)  19

(D)  21

(E)  7

Answer: (C)

32. 18.013 ÷ ? × 19.95 × 3.01 = 350.15

(A)  22

(B)  56

(C)  40

(D)  34

(E)  15

Answer: (B)

33. 45012 ÷85 ÷ √146 × 4.98 = ?

(A)  125

(B)  110

(C)  250

(D)  50

(E)  75

Answer: (A)

34. (1280.14 + 519.85) ÷99 = ?

(A)  250

(B)  100

(C)  200

(D)  150

(E)  125

Answer: (D)

35. A boat travels from A to B upstream and then from B to C downstream taking the same time. The respective ratio between the distance from A to B and the distance from B to C is 5 : 7. If the boat takes 2 hours 30 minutes to travel a distance of 35 km downstream what is the speed of the stream? (in km/h)

(A)  2 km/h

(B)  3 km/h

(C)  12 km/h

(D)  10 km/h

(E)  14 km/h

Answer: (A)

36. Rs 7,350 was partly invested in Scheme A at 10% p.a. compound interest (compounded annually) for 2 years and partly in Scheme B at 7% p.a. simple interest for 4 years. Both the schemes earn equal interests. How much was invested in Scheme A?

(A)  Rs 5,000

(B)  Rs 4,200

(C)  Rs 4,500

(D)  Rs 3,150

(E)  Rs 6,000

Answer: (B)

37. The height of cylinder is 14 cm and its curved surface area is 264 sqm. The volume of the cylinder is-

(A)  308 cm3

(B)  396 cm3

(C)  148 cm3

(D)  1232 cm3

(E)  529 cm3

Answer: (B)

38. A certain sum is divided among A, B and C in such a way that A gets 220 more than 1/3th of the sum, B gets Rs 40 less than 2/5th of the sum and C gets Rs 300. What is the total sum invested?

(A)  Rs 1,300

(B)  Rs 1,400

(C)  Rs 1,800

(D)  Rs 1,500

(E)  Rs 1,250

Answer: (C)

Direction – (Q. 39-43) In the following questions two equation numbered I and II are given you have to solve both the questions and given answers-

(A) If x < y

(B) If x ≤ y

(C) x > y

(D) If x ≥ y

(E) x = y or the relations cannot be established.

39. (I) x2 – 8x + 15 = 0

(II) y2 – 7x + 10 = 0

Answer: (D)

40. (I) 2x2 + 9x + 9 = 0

(II) 2y2 + 15y + 28 = 0

Answer: (C)

41. (I) 2x2 – 13x + 18 = 0

(II) 2y2 – 19y + 45 = 0

Answer: (B)

42. (I) 2x2 + 19x + 44 = 0

(II) 2y2 + 7y + 3 = 0

Answer: (A)

43. (I) x2 + 7x + 12 = 0

(II) 2y2 + 17y + 35 = 0

Answer: (E)

44. The cost of 2 TV sets and a radio is 7,000, while 2 radios and one TV set together cost 4,250 the cost of a TV set is-

(A)  Rs 3,000

(B)  Rs 3,160

(C)  Rs 3,240

(D)  Rs 4,160

(E)  Rs 3,250

Answer: (E)

Direction- Study the following information carefully to answer the given question.

  In a college, there were 700 students in 2012 the college offers Engineering in five specialization’s Computer Science, Civil, Electrical, Mechanical and Bio-technology. Out of the total number of male students in the college, 30% study computer science, 15% study Civil, 18% study Electrical, 12% study Mechanical and the remaining students study biotechnology.

  Out of the total number of female students in the college, 35% study computer science, 5% study civil, 20% study electrical and 15% study Mechanical. There are 75 female students in bio-technology.

45. The number of male students in bio-technology is what per cent more than the number of female students studying in the same course?

(A)   

(B) 

(C)    

(D)   

(E)   

Answer: (C)

46. What is the respective ratio between the male students in computer science and the number of female students studying in the same course?

(A)  15 : 8

(B)  13 : 9

(C)  8 : 7

(D)  10 : 9

(E)  12 : 11

Answer: (C)

47. What is the difference between the total number of female students in Electrical and Mechanical together and total number of male studying in the same courses together?

(A)  12

(B)  19

(C)  15

(D)  11

(E)  17

Answer: (C)

48. The total number of male students in Civil and Computer Science together are what per cent of the total number of students (male and female) studying in these two courses together?

(A)  66

(B)  62

(C)  64

(D)  65

(E)  60

Answer: (E)

49. What is the average number of students (male and female) in Mechanical and Bio-technology?

(A)  132

(B)  124

(C)  134

(D)  128

(E)  138

Answer: (C)

50. A started a business. After 4 months from the start of the respective ratio between the amounts invested by A, B and C was 6 : 11 : 12. If the C’s share in annual profit was 720 more than A’s share. What was the total annual profit earned?

(A)  Rs 7,680

(B)  Rs 7,860

(C)  Rs 10,500

(D)  Rs 8,680

(E)  Rs 9,000

Answer: (A)

IBPS PO (Pre.) Examination Held on 15-10-2017 Reasoning Question Paper With Answer Key

IBPS PO (Pre.) Examination Held on 15-10-2017 Reasoning
IBPS PO (Pre.) Examination Held on 15-10-2017 Reasoning Question Paper With Answer Key

IBPS PO (Pre.) Examination Held on 15-10-2017

Reasoning

Directions – (Q. 1-5) Read the following information and answer the given question-

  L, M, N, O, P, Q and R are seven employees who are working in the same company. They attend meeting in different department viz., Administrative, Security, Finance and HR department on different days from Monday to Sunday but not necessarily in the same order. One employee attends only one meeting and only one meeting is held on each day. There are two employees who attend meeting in administrative, security HR department and only one employee attends meeting in Finance department.

  L attends meeting on Thursday. There are two persons who attend meeting between L and the person who attends meeting in HR department. There are three persons who attend meeting between the persons who attend meeting in Administrative department and the one who attends meeting in Finance department. The one who attends meeting in administrative department attends before the one who attends in Finance department. The one who attends meeting in finance department does not attend on Saturday. The number of persons who attend meeting between L and the one who attend meeting in finance department is same as the number of persons who attend meeting between O and the one who attends meeting in security department. The one who attends meeting in security department attend before O. O does not attend meeting on the day immediately before the day on which L attends meeting. O does not attend meeting on the day just after the day on which L attends meeting. The number of persons who attend meeting between L and P is same as the number of persons who attend meeting between L and R. P attends meeting in one of the day before the day on which R attend meeting. N attends meeting in administrative department. R does not attend meeting in security department.

1. Who among the following person attend meeting on Friday?

(A)  P

(B)  M

(C)  P

(D)  N

(E)  R

Answer: (D)

2. Which of the following combinations of ‘Person-Day’ is true with respect to the given arrangement?

(A)  R-Friday

(B)  M-Saturday

(C)  Q-Thursday    

(D)  P-Friday

(E)  P-Tuesday

Answer: (E)

3. L attends meeting in which of the following department?

(A)  Security

(B)  HR

(C)  Administrative

(D)  Finance

(E)  Either (A) or (B)

Answer: (A)

4. In this arrangement, Q is related on Monday, L is related to Security then N is related to?

(A)  Thursday

(B)  Wednesday

(C)  HR

(D)  None of those given as options

(E)  Sunday

Answer: (B)

5. How many persons attend meeting between P and O?

(A)  3

(B)  None

(C)  2

(D)  1

(E)  More than three

Answer: (E)

Directions –(Q. 6-10) In these questions, relationship between different elements is shown in the statements. These statements are followed by two conclusions. Mark answer:

(A) If only conclusion I follows

(B) If only conclusion II follows

(C) If either conclusion I or II follows

(D) If neither conclusion I nor II follows

(E) If both conclusions I and II follow

6. Statements : L > I = N > P;

Conclusions: (I) E > P     (II) R < L

Answer: (E)

7. Statements : S > A = N ≥ D;

A ≥ L > E; M ≤ L ≤ D

Conclusions: (I) S > E     (II) L < S

Answer: (E)

8. Statements : L > I = N > P;

I ≥ R > K; N ≤ E < Z

Conclusions: (I) K > N    (II) I < Z

Answer: (B)

9. Statements: P ≥ V ≥ R ≤ E < Y;

G ≥ E > N

Conclusions: (I) P > N     (II) G ≥ Y

Answer: (D)

10. Statements: S > A = N ≥ D;

A ≥ L > E; M ≤ L ≤ D

Conclusions: (I) A > M    (II) A = M

Answer: (C)

Directions (Q. 11-15) Read the following information and answer the given question-

  A, B, C, D, E, F, G & H are eight friends and sitting around a circular table but not necessarily in same order. Some of them are facing inside and some of them are facing outside. A sits third to right of H. There is two people sits between H and B. C sits second to left of B. There is three people sits between B and E. D is second to left of F, who is not immediate neighbour of A. Immediate neighbours of H faces same direction as H. F sits third to left of A, who faces centre. The immediate neighbours of A face opposite to the direction of A.

11. Who is sitting third to the right of F?

(A)  C

(B)  B

(C)  A

(D)  E

(E)  D

Answer: (D)

12. Who is facing the centre?

(A)  AD

(B)  AGH

(C)  AB

(D)  ADC

(E)  None of those given as options

Answer: (C)

13. Who sits opposite to H?

(A)  A

(B)  D

(C)  F

(D)  E

(E)  G

Answer: (B)

14. How many people are sitting between C and B, when counted from left of C?

(A)  Two

(B)  Three

(C)  One

(D)  Four

(E)  Five

Answer: (C)

15. Who is sitting exactly between D and C when counted from right of D?

(A)  HE

(B)  CE

(C)  FH

(D)  BF

(E)  DF

Answer: (D)

Directions – (Q. 16-20) Study the following information carefully and answer the given question-

  Ten persons are sitting in 2 parallel rows containing 5 persons in each row. In 1st row M, N, O, P and Q are seated and are facing south. In 2nd row, U, V, X, Y and Z are seated and are facing north. Therefore in the given seating arrangement, each member seated in a row faces another member of the other row. They like different colours Red, Orange, Blue, Brown, Black, White, Yellow, Pink, Peach and Grey (not necessarily in same order).

  M doesn’t like brown and P likes black. Y sits third to the left of U, who likes yellow. M faces immediate neighbour of Y, who likes orange. The one who likes peach sits at extreme end. O sits second to the right of M. The one who likes red faces the one who likes pink but M doesn’t like pink. Only one person sits between N and P. V and Z are immediate neighbours. Z does not face M and N, who doesn’t like grey. The one who faces U likes white. The one who faces an immediate neighbour of Y likes brown.

16. How many persons are seated between N and the one who likes white?

(A)  None

(B)  One

(C)  Two

(D)  Three

(E)  More than three

Answer: (C)

17. Who amongst the following faces P?

(A)  U

(B)  The one who likes Pink

(C)  X

(D)  N

(E)  The one who like grey

Answer: (E)

18. Which of the following is true regarding M?

(A)  N and X are immediate neighbours of M

(B)  M sits at one of the extreme ends of the line

(C)  M likes black

(D)  P sits to the immediate left of M

(E)  None of the above

Answer: (D)

19. Who amongst the following pair sits exactly in the middle of the rows?

(A)  M, Z

(B)  P, Y

(C)  U, N

(D)  M, V

(E)  None of those given as options

Answer: (D)

20. V likes which of the following colour?

(A)  Brown

(B)  Pink

(C)  Black

(D)  White

(E)  Other than those given as options

Answer: (B)

Directions – (Q. 21-25) Study the following information and answer the given question-

  In a certain code,

  ‘detail even year this’ is written as ‘bi g voc st’,

  ‘event revised of awaited’ is written as ‘tm oc da pu’,

  ‘of detail results first’ is written as ‘nh mk tm gv’.

  ‘awaited great year of’ is written as ‘da st rx tm’

  (All codes are two letter code only)

21. In the given code language, what does the code ‘mk’ stand for?

(A)  either ‘detail’ or ‘year’

(B)  this

(C)  of

(D)  Either ‘first’ or ‘results’

(E)  great

Answer: (D)

22. What is the code for ‘awaited’ in the given code language?

(A)  tm

(B)  rx

(C)  st

(D)  da

(E)  Other than those given as options

Answer: (D)

23. What is the code for ‘event’ in the given code language?

(A)  nh

(B)  oc

(C)  gv

(D)  pu

(E)  rx

Answer: (B)

24. What may be the possible code for ‘awaited revised’ in the given code language?

(A)  ve be

(B)  da nh

(C)  nh ve

(D)  pu da

(E)  bi da

Answer: (D)

25. If ‘of year home’ is written as ‘aj tm st’ in the given code language, then what is the code for ‘home revised this’?

(A)  aj gv nh

(B)  pu aj bi

(C)  nh bi pu

(D)  bi da aj

(E)  aj oc pu

Answer: (B)

26. How many such pairs of letters are there in the word ‘TRANSFER’, each of which has as many letters between them in the word as they have between them in the English alphabet?

(A)  None

(B)  One

(C)  Three

(D)  More than three

(E)  Two

Answer: (C)

27. What should come in place of question mark (?) in the following series based on the given arrangement?

BED    EIG   HMJ  KQM ?

(A)  PUN

(B)  OUQ

(C)  NUQ

(D)  NUP

(E)  NUR

Answer: (D)

Directions – (Q. 28 and 29) Read the following information and answer the given question-

  R is the sister of Q. M is the father of R. V is the son of Q. C is the maternal grandfather of V. M does not have married daughter.

28. How is R related to V?

(A)  Uncle

(B)  Aunt

(C)  Mother

(D)  Cannot be determined

(E)  Nephew

Answer: (B)

29. If B is married to Q, then how is B related to M?

(A)  Grandson

(B)  Son-in-law

(C)  Son

(D)  Daughter-in-law

(E)  Cannot be determined

Answer: (D)

Directions- (Q. 30-32) Read the following information and answer the given question-

  A certain number of persons are seated in a row. The Row is arranged in a vertical manner and all are facing to north direction. Ranjan sits fourth from the left end of the row. There are two person sit between Ranjan and Seema. Puja sits immediate right of Seema. There are as many persons sit between Puja and Seema as sits between Dinesh and Puja. Dinesh does not sit to the left of Ranjan.

30. How many person sits between Ranjan and Dinesh?

(A)  3

(B)  5

(C)  None

(D)  4

(E)  1

Answer: (D)

31. What is the position of Puja with respect to Ranjan?

(A)  Immediate right

(B)  Third to the left

(C)  Other than those given as options

(D)  Second to the left

(E)  Fourth to the right

Answer: (E)

32. How many persons are seated in a row?

(A)  4

(B)  6

(C)  10

(D)  11

(E)  9

Answer: (E)

Directions – (Q. 33-35) Read the following information and answer the given question-

  The are six persons S, T, U, V, W and X, who got different marks in the examination. S got more marks than only U and X. T got less marks than W, who did not get the highest marks in the examination. The Second highest person got 92 marks.

33. How many persons got more marks than U?

(A)  Four

(B)  Two

(C)  Five

(D)  Cannot be determined

(E)  One

Answer: (D)

34. If S got 69 marks and U got 68 marks, then which of the following statement is true?

(A)  X got the lowest marks

(B)  U got the fifth highest marks

(C)  Five persons got more marks than X

(D)  All are true

(E)  X got 66 marks is a possibility

Answer: (D)

35. V got which of the following possible score?

(A)  85

(B)  66

(C)  92

(D)  89

(E)  94

Answer: (E)

IBPS Clerk Pre. Online Examination Held on December 3, 2017 Question Paper With Answer Key

IBPS Clerk Pre. Online Examination Held on December 3, 2017
IBPS Clerk Pre. Online Examination Held on December 3, 2017 Question Paper With Answer Key

IBPS Clerk Pre. Online Examination Held on December 3, 2017

Part I English Language

Direction (Q. Nos. 1-6) In the given sentence, a blank is given indicating that something is missing. From the given four options (a), (b) (c) and (d), a combination of words would independently fit and one combination would not. Mark that option that does not fit as the answer. If the given combinations fit perfectly, then mark ‘All fit’ as the answer. Both the words in the correct answer combination independently must not fit in the blank.

1. Games between England and Australia enjoy a popularity sadly ………… in others parts of the world.

(A) absent     (B) dearth

(C) lacking    (D) inadequately

(a)  AD

(b)  AB

(c)  BD

(d)  CD

(e)  All fit

Answer: (d)

2. A recent survey …… that light pollution is threatening darkness almost everywhere.

(A) highlighted        (B) revealed

(C) show                 (D) unveiling

(a)  AD

(b)  AB

(c)  BC

(d)  CD

(e)  All fit

Answer: (d)

3. The mobile wallet firm Mobikwik recently announced that the users will now be ……… to pay traffic challans on its application.

(A) able         (B) willing

(C) went        (D) allowed

(a)  AD

(b)  AB

(c)  BC

(d)  CD

(e)  All fit

Answer: (c)

4. The president has consistently been ……….. to take it on a bigger role in the world owing to reasons unknown.

(A) very                  (B) hesitant

(C) uncertainty       (D) reluctant

(a)  AC

(b)  AB

(c)  BC

(d)  CD

(e)  All fit

Answer: (a)

5. The two countries will ……. in the next phase of bilateral cooperation in the field of science and technology.

(A) merged              (B) clasps

(C) work together   (D) join hands

(a)  AD

(b)  AB

(c)  BC

(d)  CD

(e)  All fit

Answer: (b)

6. Passengers should brace themselves for a fare increase as the government is likely to …….. the passengers service’ fee by 30%.

(A) raise        (B) rise        (C) hike       (D) boast

(a)  BD

(b)  AB

(c)  BC

(d)  CD

(e)  All fit

Answer: (a)

Directions (Q. Nos. 7-12) Read the following passage and answer the given questions.

In a world in which nearly half the population is over the age of 50, we will have to start thinking of older people differently. The first thing we will have to recognize is that our very notion of old age is faulty. In the mid-to-late 1800s, medical theory suggested that the only healthy thing for older adults to do was rest. This gave birth to the first government pensions, corporate retirement policies, dedicated old-age homes. Surviving till today is this idea that older people are ‘supposed’ to be consumers of ideas, work, products and culture, but never producers.

Today, ask anyone over the age of 60 or so if she wants to sit in a rocking chair for the rest of her life and she’ll laugh you out of the room. Yet, this outmoded norm concerning the capabilities and goals of older people persists. It remains at the core of misguided products, policies, workplace expectations. Living longer and better may mean working longer. It may mean a lifetime of learning and growth. We may see older adults in roles to which we are not accustomed: retail, manufacturing, teaching and delivering healthcare (not just receiving it). The energy we draw from our older population may grow to become one of our most valuable resources. Businesses founded by people over the age of 50 are already major sources of employment across the nation. Products designed with older people in mind must evolve. An ageing society has given rise to a new multi-trillion-dollar market of consumers who want not just to be taken care of, but also live out aspirations once considered impossible in old age. These products, services and experiences will become a source of economic growth and innovation. Infrastructure from housing to transportation must be reengineered, Workplaces will have to learn to utilize the unique skills and institutions knowledge of older workers and resist the age-old urge to age-discriminate. We as individuals will have to rethink old age, both for the sake of those older than us and for our own future selves.

7. Choose the world which is most opposite in meaning to the word ‘Innovation’ printed in bold as used in the passage?

(a)  Modernity

(b)  Customary

(c)  Nascency

(d)  Novation

(e)  Upstart

Answer: (b)

8. Which of the following is the author referring to through the term ‘valuable resources’?

(a)  Embracing the latest technology

(b)  Economic growth

(c)  Infrastructure

(d)  Cut-throat corporate culture

(e)  None of the given options

Answer: (e)

9. Which of the following is/are the impact(s) of the medical theory mentioned in the passage?

(A) We have side-lined the older population.

(B) Better treatment of age related diseases.

(C) Creation of facilities such as pensions for the elderly.

(a)  Only A

(b)  Only B

(c)  A and C

(d)  Only C

(e)  All of these

Answer: (c)

10. Which of the following is the author’s advice regarding work?

(a)  Organizations should harness the skills of older workers.

(b)  Companies should utilize the experience of workers who are not yound.

(c)  We must embrace the idea of people working even at an older age.

(d)  We must keep in mind that older workers can be entirely productive.

(e)  All the given options

Answer: (e)

11. Which of the following is not true in the context of the passage?

(a)  We must revamp infrastructure to meet the needs of the elderly.

(b)  The elderly population is dropping.

(c)  The concept of ‘old age’ needs to be rethought.

(d)  The traditional idea of ‘old age homes’ is outdated.

(e)  All the given options are true.

Answer: (b)

12. Which of the following represent(s) the author’s view?

(A) Society needs to recognize the needs, desires and aspirations of older people.

(B) Older people are capable producers.

(C) Corporate retirement policies are very generous at present.

(a)  A and B

(b)  Only B

(c)  B and C

(d)  Only C

(e)  All of these

Answer: (a)

Directions (Q. Nos. 13-20) The sentence has been spilt into five parts denoted by A, B, C, D and E. Part A is fixed in the sentence and is to be taken as grammatically correct. The remaining four parts B, C, D and E may or may not have either (an) inappropriate word (s) or spelling errors in it. From these four, there will be only one part without any error. Identify that part and mark it as  your answer. If the sentence is correct as it is, then mark ‘No error’ as your answer.

13. Traditionally, people have/ associated excessive sweating/ with having a higher body fat/ percentage and less sweating/ with a higher level of acrobic activity.

(a)  B

(b)  C

(c)  D

(d)  E

(e)  No error

Answer: (c)

14. The country is among/the emerging global players in/research and innovation, thenks/to the excellent research centres/ and a considerable talent pool.

(a)  B

(b)  C

(c)  D

(d)  E

(e)  No error

Answer: (c)

15. After a gap of over eighty years,/ the black bear has returned/ to the northern region of the country/ through conservation, and their/population believed to be over 500.

(a)  B

(b)  C

(c)  D

(d)  E

(e)  No error

Answer: (b)

16.The government is likely/ to lounch a policy on electric vehicles/ proposing benefits for manufacturers/ and putting in place the necessary/ support system by the end of this year.

(a)  B

(b)  C

(c)  D

(d)  E

(e)  No error

Answer: (d)

17. ‘Writer Page’ is a unique platform/where budding writers/are encouraged to hone/ their writing skills/ with the help of establishment authors.

(a)  B

(b)  C

(c)  D

(d)  E

(e)  No error

Answer: (e)

18. Ferrying of coal in/uncowered vehicles and rail wagons is/said to won of the key/ reasons behind high pollution/ levels along the transportation root.

(a)  B

(b)  C

(c)  D

(d)  E

(e)  No error

Answer: (c)

19. Thanksgiving is that/festival when families/gather together, forget/ their differences and feast/ on turkey and pumpkin pie.

(a)  B

(b)  C

(c)  D

(d)  E

(e)  No error

Answer: (c)

20. There needs to be/a dipper exploration of/why parents actually have/ a problem with/their children reading books based on voilence.

(a)  B

(b)  C

(c)  D

(d)  E

(e)  No error

Answer: (b)

Directions (Q. Nos. 21-30) The sentence has  a blank which indicates something has been omitted. which of the following phrases given against the sentence, fits the blank in the given sentence both grammatically and meaningfully, If none of phrases fit, select ‘None of the given options’ as your answer.

21. She ……….. so badly in the last movie that she decided to withdraw from acting.

(a)  so hardly hit

(b)  got alive and kicking

(c)  got her fingers burnt

(d)  fall at life

(e)  None of the given options

Answer: (c)

22. Last night the cricket match was ………. as the home team could not stand their opponents.

(a)  a throwback on

(b)  epic fall

(c)  a major let down

(d)  loss of face

(e)  None of the given options

Answer: (c)

23. Since you were just discharged from the hospital, you need ………. so that you are able to recover fully.

(a)  plenty of down-time

(b)  play back required

(c)  take care of

(d)  recover from

(e)  None of the given options

Answer: (a)

24. He started working from home so that he would be able ………. with his family.

(a)  for the duration

(b)  to spend more time

(c)  to in the long run

(d)  spend a penny for

(e)  None of the given options

Answer: (b)

25. While I was searching the attic, I ………. some very interesting photographs.

(a)  find a way around

(b)  came across

(c)  looking forward to

(d)  came down with

(e)  None of the given options

Answer: (b)

26. Simone ……… deleted Ravi’s contact number but pretended to have lost it.

(a)  an honest mistake

(b)  accidentally on purpose

(c)  at cross purpose

(d)  all intention of purpose

(e)  None of the given options

Answer: (d)

27. The manager offered him a lucrative job but he ……….. .

(a)  fell for it

(b)  paid dearly for

(c)  turned it down

(d)  given it away

(e)  None of the given options

Answer: (c)

28. The head of the detective department asked the officers to …….. before reaching any conclusion in the case.

(a)  thinking over

(b)  do searching of

(c)  looking for clues

(d)  explore all the avenues

(e)  None of the given options

Answer: (d)

29. The teacher complained to his mother that he does not ……… the rule.

(a)  follow over

(b)  make the rounds

(c)  stick with

(d)  abide by

(e)  None of these

Answer: (d)

30. At the beginning of her performance on stage, she always has ……… .

(a)  butterflies in her stomach

(b)  get highly tensed

(c)  nervous brokedown

(d)  get all worked up

(e)  None of the given options

Answer: (a)

Part II Reasoning Ability

Directions (Q. Nos. 31-32) In each question, relationship between different elements is shown in the two statements. The statements are followed  by conclusions. Study the conclusions based on the given statement and select the appropriate answer.

Give answer

(a) if either conclusion I or II follows

(b) if neither conclusion I nor II follows

(c) if only conclusion II follows

(d) if both conclusions follow

(e) if only conclusion I follows

31. Statement M ≥ P ≥ F ≥ Q; R ≤ F

Conclusion I R ≤ M

Conclusion II Q < R

Answer: (e)

32. Statement G = E ≤ N < R ≥ L > T

Conclusion I T < N

Conclusion II E ≤L

Answer: (b)

Directions (Q. Nos. 33-37) Study the information to answer the given questions.

In a certain code,

‘lots of exams today’ is written as ‘po li sy ko’

‘today are final exams’ is written as ‘jp sy po fr’

‘are the lots ready’ is written as ‘fr dv ko mt’

‘get ready for exams’ is written as ‘sy ot gh dv’

(All the codes are two letter codes only)

33. What is the code for ‘ready today’ in the given code language?

(a)  mt po

(b)  po dv

(c)  dv fr

(d)  li ot

(e)  mt ko

Answer: (b)

34. Which of the following may represent ‘the routine exams’ in the given code language?

(a)  mt kk gh

(b)  dv op sy

(c)  mt ot sy

(d)  fr sy mt

(e)  sy ul mt

Answer: (e)

35. What is the code for ‘get’ in the given code language?

(a)  Either ‘sy’ or ‘dv’

(b)  sy

(c)  ko

(d)  Either ‘ot’ or ‘gh’

(e)  None of these

Answer: (d)

36. What is the code for ‘lots’ in the given code language?

(a)  po

(b)  ko

(c)  fr

(d)  li

(e)  mt

Answer: (b)

37. What is the code for ‘final’ in the given code language?

(a)  po

(b)  jp

(c)  ko

(d)  fr

(e)  sy

Answer: (b)

Directions (Q. Nos. 38-42) Study the following information carefully and answer the questions.

Eight people C, D, E, F, S, T, U and V are sitting around a circular table, facing the centre with equal distance between each other.

Only three people sit between E and U. Only two people sit between U and F. C sits third to the left of V. V is neither an immediate neighbour of U nor F. D sits to the immediate right of T.

38. Which of the following represents the position of D with respect to C?

(a)  Second to the right

(b)  Third to the left

(c)  Fourth to the right

(d)  Third to the right

(e)  Second to the left

Answer: (b)

39. Which of the following is true regarding S as per the given arrangement?

(a)  S sits to the immediate right of D.

(b)  None of the given options is true.

(c)  S is an immediate neighbour of F.

(d)  S sits second to the right of E.

(e)  Only three people sit between S and T.

Answer: (b)

40. Who sits second to the right of T?

(a)  F

(b)  V

(c)  E

(d)  U

(e)  C

Answer: (d)

41. How many people are sitting between V and F when counted from the right of F?

(a)  One

(b)  None

(c)  Four

(d)  three

(e)  Two

Answer: (a)

42. Who among the following are seated exactly between U and F when counted from the right of U?

(a)  C, D

(b)  C, S

(c)  E, S

(d)  E, T

(e)  S, T

Answer: (b)

Directions (Q. Nos. 43-45) Study the given information carefully to answer the given questions.

Sameer starts walking from Point A, He walks 15 m towards East and reaches Point B. He then takes a left turn, walks 5m and reaches Point C. From Point C he takes, a left turn. walks 6 m and reaches Point D. He takes a left turn, walks 10 m and reaches Point E. From Point E, he takes a right turn, walks 9 m and reaches point F.

43. How far and in which direction is Point A with respect to Point F?

(a)  9m towards South

(b)  9m towards North

(c)  8m towards North

(d)  5m towards North

(e)  6m towards South

Answer: (d)

44. Points B, C and Z form a straight line in such a way that B falls at a point between Points C and Z. If the distance between B and Z is double the distance between Points B and C, in which direction is Point Z with respect to Point E?

(a)  South-East

(b)  North-West

(c)  West

(d)  South-West

(e)  North-East

Answer: (a)

45. In which direction is Point F with respect to Point C?

(a)  North

(b)  East

(c)  South-West

(d)  West

(e)  South-East

Answer: (c)

Directions (Q. Nos. 46-50) Study the following information to answer the given questions.

Ten people are sitting in two parallel rows containing five people each, in such a way that there is an equal distance between adjacent persons. All the given persons in both the rows face North. In row-1 A, B, C, D and E area seated and in row-2, L, M, N, O and P are seated. Therefore, in the given seating arrangement, each member of row-2 is sitting right behind a member of row-1.

D sits second to the left A. A does not sit at any of the extreme ends of the line. The one sitting behind A sits to the immediate right of P. The one sitting in front of M sits third to the right of B. Only one person sits between B and C. E sits in front of L. More than two people sit between L and N.

46. Which of the following is true regarding O?

(a)  O sits second to the left of M.

(b)  O sits behind C.

(c)  O is an immediate neighbour of L

(d)  O sits at one of the extreme ends of the line.

(e)  None of the given option is true.

Answer: (a)

47. In which of the following pairs of people, are both of them sitting at extreme ends of the rows?

(a)  C, M

(b)  C, N

(c)  E, N

(d)  B, O

(e)  B, P

Answer: (c)

48. B is related to C in the same way as P is related to L based on the given seating arrangement. To who amongst the following is N related to following the same pattern?

(a)  N

(b)  The one sitting behind E

(c)  P

(d)  O

(e)  The one sitting behind D

Answer: (c)

49. How many persons are seated between E and D?

(a)  One

(b)  None

(c)  Three

(d)  Two

(e)  Cannot be determined

Answer: (d)

50. Who amongst the following is sitting in front of O?

(a)  B

(b)  The one sitting to immediate right of A

(c)  The one sitting to immediate right of C

(d)  C

(e)  D

Answer: (e)

Directions (Q. Nos. 51-53) Study the following arrangement carefully and answer the given questions.

D C U D B C B A C D E B U B A C

A D C A D B A E B U A D E B C E

51. How many alphabets are there in the English alphabetical series between the alphabet which is fourth from the left end and the alphabet which is third from the right end of the arrangement?

(a)  More than three

(b)  Two

(c)  One

(d)  None

(e)  Three

Answer: (c)

52. If all D’s are dropped from the given arrangement, which of the following will be eighth from the left end of the given arrangement?

(a)  U

(b)  A

(c)  E

(d)  B

(e)  C

Answer: (c)

53. How many such vowels are there in the given arrangement each of which is immediately preceded by a vowel and also immediately followed by a consonant?

(a)  Two

(b)  More than four

(c)  Three

(d)  One

(e)  Four

Answer: (a)

Directions (Q. Nos. 54-58) Study the following information and answer the given questions.

Each of eight people-L, M, N, O, P, Q, R and S was born in a different month viz. January, March, May, June, September, October, November and December of the same year.

S was born in one of the months before April. Only three people were born between the months in which S and Q were born. Only two people were born between L and N. L was born in one of the months before N. More than two people were born between the months in which L and Q were born. As many people were born before N as after O. P was born in one of the months having only 30 days. M has born in one of the months before R.

54. If X was born in one of the months between the months in which N and O were born, in which was X possibly born?

(a)  February

(b)  May

(c)  April

(d)  October

(e)  August

Answer: (e)

55. Who amongst the following was born in a month immediately after the month in which P was born?

(a)  M

(b)  L

(c)  R

(d)  Q

(e)  S

Answer: (c)

56. In which of the following months was M born?

(a)  March

(b)  December

(c)  January

(d)  May

(e)  November

Answer: (d)

57. Four of the following five are alike in a certain way based on the given information and thus form a group. Which is the one that dies not belong to that group?

(a)  L

(b)  m

(c)  R

(d)  S

(e)  O

Answer: (e)

58. How many people were born between the months is which S and O were born?

(a)  Two

(b)  Three

(c)  None

(d)  One

(e)  Four

Answer: (a)

Directions (Q. Nos. 59-62) Following questions are based on five words given below.

SHE   OLD   ANT   TIN   JUG

(The new words formed after performing the mentioned operations may or may not necessarily be meaningful English words)

59. If S is added before the first letter in each word, which of the following would form meaningful English words with the new arrangement?

(a)  Only SHE

(b)  ANT and JUG

(c)  Only OLD

(d)  TIN and JUG

(e)  Only TIN

Answer: (c)

60. If in each of the given words each of the consonants is changed to previous letter and each vowel is chaged to next letter in the English alphabetical series, in how many words thus formed will at least one vowel appear?

(a)  One

(b)  More than three

(c)  None

(d)  Three

(e)  Two

Answer: (a)

61. How many letters are there in the English alphabetical series between the second letter of the word which is second from the right and second letter of the word which is second from the left of the given words?

(a)  Two

(b)  Eight

(c)  Five

(d)  Six

(e)  Three

Answer: (a)

62. If second alphabet in each of the words is chaged to next alphabet in the English alphabetical order, how many words having more than one vowel will be formed?

(a)  Three

(b)  More than three

(c)  Two

(d)  One

(e)  None

Answer: (c)

Directions (Q. Nos. 63-65) In each question two/three statements followed by two conclusions numbered I and II have been given. You have to take the given statements to be true even if they seem to be at variance from the commonly known facts and then decide which of the given conclusions logically follows.

Give answer

(a) if either conclusion I or II follows

(b) if both conclusions follows

(c) if neither conclusion I nor II follows

(d) if only conclusion II follows

(e) if only conclusion I follows

63. Statements All alerts are mails. No mail is a post. Some posts are boxes.

Conclusion I No post is an alert.

Conclusion II No box is an alert.

Answer: (e)

64. Statements All alerts are nails. No mail is a post. Some posts are boxes.

Conclusion I Atleast some boxes are nails.

Conclusion II All mails can never be boxes.

Answer: (c)

65. Statements Some sauces are jams. All jams are pickles.

Conclusion I Some jams are definitely not sauces.

Conclusion II At least some pickles are sauces.

Answer: (d)

Part III Quantitative Aptitude

66. 120 kg and 150 kg of potatoes were stored in warehouse on Monday and Tuesday respectively. If ‘X’ kg of potatoes got rotten on each day and the respective ratio of number of potatoes that did not get rotten on Monday and Tuesday was 3 : 4, what is the value of X?

(a)  30

(b)  25

(c)  50

(d)  20

(e)  40

Answer: (a)

Directions (Q. Nos. 67-81) What will come in place of question marks (?) in the given questions?

67. (612 +184 – ?) ÷ (2 ÷04) = 15

(a)  50

(b)  46

(c)  42

(d)  54

(e)  49

Answer: (b)

68. 

(a)  16

(b)  8

(c)  4

(d)  0.5

(e)  2

Answer: (b)

69. (4.5)2 – (1.5)2 = ?2

(a)  9√2

(b)  9

(c)  3√2

(d)  18

(e)  4√3

Answer: (c)

70. 3432/3 * 2561/4 – ? = 122

(a)  44

(b)  56

(c)  52

(d)  42

(e)  54

Answer: (c)

71. 

(a)  3

(b)  5

(c)  8

(d)  7

(e)  6

Answer: (b)

72. 

(a)  14

(b)  15

(c)  12

(d)  26

(e)  28

Answer: (e)

73. 

(a)  25

(b)  15

(c)  10

(d)  20

(e)  45

Answer: (b)

74. 4/5th of 3/8th of ? = 112 + 5

(a)  450

(b)  390

(c)  360

(d)  420

(e)  440

Answer: (d)

75. 4 – ? ÷ 5 – 3.9 = 17.8

(a)  17.5

(b)  16.5

(c)  18.5

(d)  12.5

(e)  14.5

Answer: (c)

76. 

(a)  225

(b)  324

(c)  196

(d)  400

(e)  289

Answer: (a)

77. (18* 23 – 141) ÷5 = 2?

(a)  2

(b)  5

(c)  4

(d)  3

(e)  6

Answer: (c)

78. 

(a)  12

(b)  16

(c)  36

(d)  24

(e)  48

Answer: (d)

79. √? = 120 ÷ 48 + 29.5

(a)  900

(b)  1156

(c)  784

(d)  1024

(e)  676

Answer: (d)

80. 2/5 of (40 ÷25 + 2 ÷ 5) = ?

(a)  4/5

(b)  1/2

(c)  2/5

(d)  1/4

(e)  1/5

Answer: (a)

81. 14* 5 ÷ 8 + 7.25= ?2

(a)  6

(b)  12

(c)  8

(d)  4

(e)  2

Answer: (d)

82. Train A, 150 m long and running at 15 m/sec overtakes Train B running at 10 m/sec in a direction same as that of Train A in 72 sec (from the moment both trains meet). What is Train B’s length?

(a)  240 m

(b)  220 m

(c)  200 m

(d)  160 m

(e)  210 m

Answer: (e)

83. A vessel contains a mixture of X L milk and 25 L water in the respective ratio of 24 : 5. If 29 L mixture is taken out and 4 L water is added to it, what will be the resultant respective ratio between milk and water?

(a)  16 : 3

(b)  8 : 1

(c)  16 : 5

(d)  4 : 1

(e)  8 : 3

Answer: (e)

84. The sum of four consecutive even numbers is 236. Sum of four consecutive odd numbers is 140% more than the second highest even number. What is the average of the given four odd numbers?

(a)  32

(b)  38

(c)  34

(d)  28

(e)  36

Answer: (e)

Directions (Q. Nos. 85-89) What will come in place of question marks in the given number series?

85. 129 127  122  112  ?  69

(a)  105

(b)  65

(c)  115

(d)  75

(e)  95

Answer: (e)

86. 60 89  112  131  ?  161

(a)  158

(b)  148

(c)  142

(d)  152

(e)  144

Answer: (b)

87. 2 8  26  80   242  ?

(a)  718

(b)  716

(c)  728

(d)  722

(e)  724

Answer: (c)

88. 25 8   128  1024  ?  8192

(a)  4048

(b)  2024

(c)  4096

(d)  3072

(e)  2048

Answer: (c)

89. 5  6  10   17  30  ?

(a)  45

(b)  58

(c)  60

(d)  52

(e)  55

Answer: (e)

90. Marked price of article A is 40% more than its Cost Price. Profit earned when article A was sold after giving two successive discounts of 10% each is Rs 53.60. If article A is sold at 30% profit, what would be its selling price?

(a)  Rs 580

(b)  Rs 680

(c)  Rs 750

(d)  Rs 590

(e)  Rs 520

Answer: (e)

91. A and B started a business with investment of Rs 12800 and Rs 15600 respectively. 8 months after the start of the business, B left. If the total annual profit was Rs 14500, what was A’s share in profit?

(a)  Rs 8800

(b)  Rs 8000

(c)  Rs 7200

(d)  Rs 7680

(e)  Rs 9600

Answer: (b)

92. Harshal spends his monthly income on paying rent, buying groceries and investing in mutual funds in the respective ratio of 2 : 3 : 5. If the difference between the income spent on rent and that invested in mutual funds is Rs 12000, what is h is annual income?

(a)  Rs 4.8 lakh

(b)  Rs 4.0 lakh

(c)  Rs 5.4 lakh

(d)  Rs 3.6 lakh

(e)  Rs 6.0 lakh

Answer: (a)

93. Suresh invested some money for 8 years in scheme A offering simple interest at 12% p.a. The interest received from Scheme A was then invested in scheme B (which offers simple interest at 10% p.a.) for 5 years. If interest received from scheme B was Rs 4032, what was the sum invested in scheme A?

(a)  Rs 9600

(b)  Rs 7200

(c)  Rs 7600

(d)  Rs 8000

(e)  Rs 8400

Answer: (e)

94. A alone can finish a piece of work in 36 days. B is 20% more efficient than A. A alone started working and was replaced by B after 8 days. In how many days B alone can finish the remaining work?

(a)  21 1/3

(b)  19 1/3

(c)  25 2/3

(d)  20 2/3

(e)  23 1/3

Answer: (e)

Directions (Q. Nos. 95-99) Study the table and answer the given questions.

95. What was the respective ratio of total number of tickets sold from Location D on Tuesday and Wednesday together to number of tickets sold from Location A on Wednesday?

(a)  6 : 5

(b)  3 : 2

(c)  5: 4

(d)  2 : 1

(e)  4 : 3

Answer: (d)

96. What is the average number of tickets sold from Location C on Monday, Wednesday and Friday?

(a)  91

(b)  89

(c)  90

(d)  88

(e)  87

Answer: (c)

97. If number of tickets sold from Location E on Tuesday was 15/31 times the total number of tickets sold from locations A and B together on Tuesday, how many tickets were sold from Location E on Tuesday?

(a)  60

(b)  65

(c)  70

(d)  75

(e)  55

Answer: (d)

98. What was the difference between total number of tickets sold from Location B and D on Monday and those sold from the same locations together on Thursday?

(a)  9

(b)  12

(c)  11

(d)  14

(e)  8

Answer: (b)

99. Number of tickets sold from Location a on Monday was what percent more than those sold from Location A on Friday?

(a)  25%

(b)  22.5%

(c)  15%

(d)  12.5%

(e)  17.5%

Answer: (d)

100. If the length of the rectangle is increased by 25%, its perimeter would become 92 m. If the breadth of the original rectangle is increased by 50%, its perimeter would become 96 m. What is the breadth of the original rectangle?

(a)  14 m

(b)  12 m

(c)  16 m

(d)  20 m

(e)  18 m

Answer: (c)

IBPS Specialist Officer Pre. Online Examination Held on December 20, 2017Question Paper With Answer Key

IBPS Specialist Officer Pre. Online Examination Held on December 20, 2017
IBPS Specialist Officer Pre. Online Examination Held on December 20, 2017Question Paper With Answer Key

IBPS Specialist Officer Pre. Online Examination Held on December 20, 2017

 

Directions (Q. Nos. 1-5) Each sentence has two blanks, each blank indicating that something has been omitted. Choose the set of words for the blanks which best fits the meaning of the sentence as a whole.

1. A rift on a gigantic ice shall has grown more ………… in the past few months than it has …………. the past five years, and the consequences could be disastrous.

(a)  rapidly; over

(b)  slowly; from

(c)  fast; in

(d)  sluggishly; under

(e)  briskly; on

Answer: (a)

2. Contrary to the usual ………… of air travel, wherein July is the lean month, this year’s air travel data presents a ………… picture.

(a)  style; similar

(b)  trend; different

(c)  pattern; poor

(d)  Statistic; solid

(e)  fad; drastic

Answer: (b)

3. The new material, when incorporated ……. windows or sunroofs, can …….. the incoming heat and light from the Sun, thereby cutting electricity costs.

(a)  at; forfeit

(b)  to; manage

(c)  in; control

(d)  for; change

(e)  of; regulate

Answer: (c)

4. Initially preferred by many as a winter …………., velvet has, over the years, assumed great versatility and can now be worn …….. the year.

(a)  dress, along

(b)  clothing, across

(c)  thread, for

(d)  fabric, throughout

(e)  suit, out

Answer: (d)

5. Recently, the local municipal corporation of the city has made the ………. of yoga mandatory in all the schools that come ………… its purview.

(a)  outcome, within

(b)  duty, on

(c)  essence, for

(d)  custom, off

(e)  practice, under

Answer: (e)

Directions (Q. Nos. 6-15) Read the following passage carefully and answer the question given. Certain words/phrases have been given in bold to help you locate them while answering some of the questions.

Education in most of the developing world is shocking. Half the children in South Asia and a third of those in Africa who complete four years of schooling cannot read  properly. In India, 60% of six-to-14-year-olds cannot read at the level of a child who has finished two years of schooling.

Most government have proposed to provide universal primary education and to promote secondary education. But even when public schools exist, they often fail to do so.

Results of the survey conducted in this respect by the World Bank depicted worrisome results. In rural Indian schools, a quarter of teachers was absent. In Africa, the teacher-absenteeism rates ranged between15-25%. Pakistan recently discovered that it had over 8000 non-existent state schools, 17% of the total, Sierra Leone spotted 6000’ ghost’ teachers, nearly a fifth the number on the State payroll.

Powerful teachers’ union are part of the problem, They often see jobs as hereditary sinecures, the State-education budget as a revenue stream to be milked and any attempt to monitor the quality of education as an intrusion. The union can be fearsome enemies, so governments leave them to run schools in the interests of teachers rather than pupils.

The failure of State education, combined with the shift in emerging economies from farmin to jobs that need at least a modicum of education, has caused a private-school boom. According to the World Bank, across the developing world, a fifth of primary-school pupils are enrolled in private schools twice, as many as 20 years ago. So many private schools are unregistered that the real figure is likely to be much higher. A census in Lagos found 12000 private schools, four times as many as on government records. Across Nigeria, 26% of primary-age children were in private schools in 2010, up from 18% in 2004. In India in 2013, 29% were, up from 19% in 2006. In Liberia and Sierra Leone, around 60% and 50% respectively of secondary schools enrollments are private.

By and large, politicians and educationalists are unenthusiastic. Governments see education as the State’s job. Teacher’s union dislike private schools because they pay less and are harder to organize in NGOs tend to be ideologically opposed to the private sector. The UN special rapporteur on education has said that ‘for-profit education should not be allowed in order to safeguard the noble cause of education.’ This attitude harms those whom educationalists claim to serve children.

The boom in private education is excellent news for them and their countries, for three reasons. First, it is bringing in money-not just from parents, but also from investors, some in search of a profit. Most private schools in the developing world are single operators that charge a few dollars a month, but chains are now emerging.

Bridge International Academies, for instamce, has 400 nursery and primary schools in Kenya and Uganda which teach in standardized classrooms that look rather like stacked shipping containers. It plans to expand into Nigeria and India. Mark Zuckenberg, Facebook’s founder. Bill Gates and the International Finance Corporation, the World Bank’s private-sector arm, are among its investors, Chains are a healthy development, because they have reputation to guard. Second, private schools are often better value for money than State ones. Measuring this is hard, since the children who go to private schools tend to be better off and therefore likely to perform better.

But a rigorous four-year study of 6000 pupils in Andhra Pradesh, in southern India, suggested that private pupils perform better in English and Hindi than public-school pupils and at a similar level in mathematics and Telugu, the local language, The private schools achieved these results at a third of the cost of the public schools. Lastly, private schools are innovative. Since, technology has great (though as yet mostly unrealized) potential in education, this could be important. Bridge gives teachers tablets linked to a central system that provides teaching materials and monitors their work. Such robo-teaching may not be ideal, but it is better than lessons without either materials or monitoring.

Critics of the private sector are right that it has problems. Quality ranges from top-notch international standard to not much more than cheap child care. But the alternative is often a public school that is worse ….. or no school at all.

6. As mentioned in the passage, the boom in education sector, brought in by the private education, is welcomed because

(A) It provides a platform for bringing in innovative practices.

(B) If focuses on education children on a large scale.

(C) It encourages investment from all stakeholders.

(a)  Only A

(b)  Only B

(c)  Both A and C

(d)  Only C

(e)  Both B and C

Answer: (c)

7. Which of the following is true in the context of the passage?

(a)  Private schools in Africa function better as compared to those in Sierra Leone.

(b)  Keeping track of the number of mushrooming private schools is easy in the developing world.

(c)  The education sector in Liberia is  mainly dominated by public schools.

(d)  Standalone education institutions are better managed as compared to chains of schools.

(e)  None of the given statements is true.

Answer: (c)

8. Which of the following is most nearly the opposite in meaning as the world ‘Safeguard’ as used in the passage?

(a)  Threaten

(b)  Shield

(c)  Misfortune

(d)  Violent

(e)  Default

Answer: (a)

9. Which of the following is most nearly the opposite in meaning as the word ‘Unenthusiastic’ as used in the passage?

(a)  Illogical

(b)  Passionate

(c)  Apathetic

(d)  Kind

(e)  Distressed

Answer: (b)

10. The data with respect to the failure of public schools

(a)  depicted the real picture of the education sector in the developed world.

(b)  identified loopholes in the existing pedagogy followed by most public schools.

(c)  brought to the fore the fact that teachers must undergo constant training.

(d)  highlighted the fact that a good number of schools and teachers existed only on paper.

(e)  emphasized the point that absenteeism from school was a problem that existed only in the developing world.

Answer: (d)

11. Which of the following correctly explains the meaning of the phrase, ‘ hereditary sinecures’ as used in the passage for jobs?

(a)  These can be acquired only on the basis of merit.

(b)  These are inherited by female members of the family and passed on from generation to generation.

(c)  These are acquired with minimum effort and involve little or no work.

(d)  These are married by insecurities.

(e)  These are not at all lucrative.

Answer: (c)

12. Which of the following is most nearly the same in meaning as the word ‘Milked’ as used in the passage?

(a)  Saved

(b)  Exploited

(c)  Expressed

(d)  Outsourced

(e)  Managed

Answer: (b)

13. As mentioned in the passage, teachers’ unions pose a threat to the education system because

(A) they have a strong network of student volunteers.

(B) they always think about betterment of students at the cost interests of teachers.

(C) they follow controversial methods of teaching.

(a)  Only A

(b)  Only B

(c)  Both A and B

(d)  Only C

(e)  None of the given A, B & C

Answer: (e)

14. Which of the following is most nearly the same in meaning as the word ‘Modicum’ as used in the passage?

(a)  Minimum

(b)  Strategy

(c)  Medium

(d)  Abundance

(e)  Relief

Answer: (a)

15. Which of the following is the central theme of the passage?

(a)  Exploring methods to implement education models from the western world.

(b)  Provisions for making teacher unions an integral part of the education system.

(c)  Exploring the role played by the technology to better the education sector in the developed world.

(d)  Concerns surrounding the education system in the developing world.

(e)  Critically analyzing the loopholes in the private sector.

Answer: (d)

Directions (Q. Nos. 16-20) Rearrange the given six sentences A, B, C, D, E and F in a proper sequence so as to form a meaningful paragraph and then answer the given questions.

(A) Conversion of forests into agricultural land is weakening India’s monsoon.

(B) This is because forests are a source of transpiration, which adds to rainfall.

(C) It contributes to around 25% of the total precipitation in the later stages of the monsoon.

(D) The latter cannot do the same, thereby resulting in a weak monsoon.

(E) Now, transpiration is the process by which trees channel moisture from the soil into the air.

(F) However, deforestation replaces deep-rooted plants with shallow-rooted vegetation.

16. Which of the following should be the second sentence after the rearrangement?

(a)  A

(b)  B

(c)  C

(d)  D

(e)  E

Answer: (a)

17. Which of the following should be the third sentence after the rearrangement?

(a)  D

(b)  E

(c)  C

(d)  B

(e)  A

Answer: (d)

18. Which of the following should be the first sentence after the rearrangement?

(a)  B

(b)  F

(c)  D

(d)  C

(e)  A

Answer: (d)

19. Which of the following should be the sixth sentence after the rearrangement?

(a)  B

(b)  C

(c)  D

(d)  E

(e)  F

Answer: (e)

20. Which of the following should be the fifth sentence after the rearrangement?

(a)  B

(b)  C

(c)  D

(d)  E

(e)  F

Answer: (d)

Directions (Q. Nos. 21-30) Which of the phrases (a), (b), (c) and (d) given be low should replace the phrase given in bold in the following sentences to make the sentence grammatically correct. If the sentence is correct as it is and ‘No correction is required’, mark (e) as the answer.

21. The drought preceding winter has played spoilsport with regard to the sowing of winter crops such as wheat, pulses and oilseeds.

(a)  playing spoilsport

(b)  are  being playing spoilsports

(c)  has been spoilt

(d)  to p lay spoilsport

(e)  No correction required

Answer: (e)

22. Evidence indicates that businesses are holding off on investment as they await clearly about this country’s future relationship with the European Union.

(a)  they wait for clarity

(b)  clearly waiting

(c)  their waiting clearly

(d)  to wait for clarity

(e)  No correction required

Answer: (a)

23. Over the past three years, the government has been urging civil servants to open bank accounts to which they pay transferred directly.

(a)  they are paid

(b)  their paying to be

(c)  their paying can be

(d)  it has been paid

(e)  No correction required

Answer: (c)

24. Scientists at an American University have discovered a fabric that keeps the skin 2 degrees cooler which will be good for the wearer serve to reduce energy bills.

(a)  by serving reduced

(b)  and also serve to reduce

(c)  to serve reduce

(d)  by serving a reduction

(e)  No correction required

Answer: (b)

25. Newly created a vaccine from scratch is a huge undertaking, but in the case of Ebola, several vaccines were already on the shelf, thanks to the work of the army.

(a)  Creating a new

(b)  Creation from a new

(c)  To create newly

(d)  The new creation

(e)  No correction required

Answer: (a)

26. As per the World Giving index, India came down in the rankings from 93 in 2013 to 106 in 2014 demonstration an overall reduction in Indian philanthropy.

(a)  that reducing overall

(b)  how to reduce overall

(c)  that an overall reduction of

(d)  if there is overall a reduction

(e)  No correction required

Answer: (e)

27. The company has been testing self-driving cars on the roads of the capital for months and will soon begin offering customers request chances rides in one.

(a)  a chance request

(b)  to request chance

(c)  a chance to request

(d)  requesting a chance

(e)  No correction required

Answer: (c)

28. They became much involved in the rescue and relief work that they refused to be evacuated.

(a)  so involved with

(b)  very involved

(c)  more involved with

(d)  too involved that

(e)  No correction required

Answer: (a)

29. The appreciate the social transformations that took place in Europe between 1815 and World War I, it is importantly considered its towns and cities evolved.

(a)  considered of importance that

(b)  to be considered importantly

(c)  important to consider how

(d)  considered how important

(e)  No correction required

Answer: (c)

30. When a country became richer, they tend to move away from the use of cash on grounds of security, convenience and cost.

(a)  When rich countries

(b)  Until rich country becomes richer

(c)  A country which is rich

(d)  As countries become richer

(e)  No correction required

Answer: (d)

Directions (Q. Nos. 31-40) Read each sentence to find out whether there is any grammatical error in it. The error, if any, will be in one part of the sentence. Mark the part with the error as your answer. If there is no error, your answer is (e). (Ignore errors of punctuation, if any)

31. For almost three decades, central bankers/have agreed that one of the best way/ to stabilize an economy is/to aim for a specific ‘inflation target’.

(a)  For almost three decades, central bankers

(b)  have agreed that one of the best way

(c)  to stabilize an economy is

(d)  to aim for a specific ‘inflation target’

(e)  No error

Answer: (b)

32. Consulting firms estimates that about $1540 billion / of the world’s IT budget of $34 trillion will be spent/ on cloud computing this year alone/ a number which is expected to grow tenfold by next year.

(a)  Consulting firms estimates that about $1540 billion

(b)  of the world’s IT budget of $34 trillion will be spent

(c)  on cloud computing this year alone

(d)  a number which is expected to grow tenfold by next year

(e)  No error

Answer: (a)

33. In this state, a mere/ 50% of the food been consumed/ is grown locally, compared/ to 81% in the other States.

(a)  In this state, a mere

(b)  50% of the food been consumed

(c)  is grown locally, compared

(d)  to 81% in the other States

(e)  No error

Answer: (b)

34. After years of economic stagnation/ and questionable lending,/ bad loans at Italian banks/ have pile up.

(a)  After years of economic stagnation

(b)  and questionable lending

(c)  bad loans at Italian  banks

(d)  have pile up

(e)  No error

Answer: (d)

35. The main reason for Britain’s success/ at the Olympics was/ the efficient allocation of fund toward/ equipment, facilities and training for athletes.

(a)  The main reason for Britain’s success

(b)  at the Olympics was

(c)  the efficient allocation of fund toward

(d)  equipment, facilities and training for athletes

(e)  No error

Answer: (e)

36. Over thirty years ago, Chile created/ a pension system in which workers/ had to save for their own retirement by paying/ 10% of its earning into individual accounts.

(a)  Over thirty years ago, Chile created

(b)  a pension system in which workers

(c)  had to save for their own retirement by paying

(d)  10% of its earning into individual accounts

(e)  No error

Answer: (d)

37. The cement industry is one of/ the world’s most polluting industries/ and it accounts 5% of/ manmade carbon dioxide emissions each year.

(a)  The cement industry is one of

(b)  the world’s most polluting industries

(c)  and it accounts 5% of

(d)  manmade carbon dioxide emissions each year

(e)  No error

Answer: (c)

38. Under the ‘Dodd-Frank’ Act, a law aimed/ mainly at tightening the regulation of companies,/ firms in the United States had to be able/ to show how the minerals used in their products are formed.

(a)  Under the ‘Dodd-Frank’ Act, a law aimed

(b)  mainly at tightening the regulation of companies

(c)  firms in the United States had to be able

(d)  to show how the minerals used in their products are formed

(e)  No error

Answer: (d)

39. Launching a satellite into space/ remained a risky and hideously/ expensive proposition, only taken/ up by governments and a few companies.

(a)  Launching a satellite into space

(b)  remains a risky and hideously

(c)  expensive proposition, only taken

(d)  up by governments and a few companies

(e)  No error

Answer: (b)

40. Under the terms of the policy,/ a payment has been triggered only/ if more than 1.39 million people/ are affected by the drought.

(a)  Under the terms of the policy

(b)  a payment has been triggered only

(c)  if more than 1.39 million people

(d)  are affected by the drought

(e)  No error

Answer: (b)

Directions (Q. Nos. 41-50) In the given passage,, there are blanks, each of which  has been numbered. Against each, five words are suggested, one of which fits the blank appropriately. Find the appropriate word in each case.

Brazil’s Tourism Ministry is (41) a 6% increase in the number of tourists visiting the country (42) the next year, following the end of the Rio Olympics. The number is an estimate based on the (43) from the last three Olympic host cities-Athens, Beijing and London. Now that the curtain has been (44) and that the host country has taken its final bow. Brazil (45) released a glowing self-assessment of its performance based (46) the survey results from its international visitors. Rio (47) five lakh visitors (48) the games, with 88% of foreigners saying that they intend to come back to Brazil. However, when it (49) to security, public transport, infrastructure and prices, satisfaction levels fall (50) the ‘good’ category.

41.

(a)  negotiating

(b)  announcing

(c)  predicting

(d)  generating

(e)  approving

Answer: (c)

42.

(a)  on

(b)  over

(c)  from

(d)  off

(e)  at

Answer: (c)

43.

(a)  searching

(b)  highlight

(c)  citing

(d)  findings

(e)  discoveries

Answer: (d)

44.

(a)  drawn

(b)  fallen

(c)  risen

(d)  there

(e)  picked

Answer: (b)

45.

(a)  have

(b)  had

(c)  has

(d)  were

(e)  was

Answer: (c)

46.

(a)  unto

(b)  on

(c)  for

(d)  in

(e)  off

Answer: (b)

47.

(a)  stopped

(b)  felt

(c)  resided

(d)  brought

(e)  received

Answer: (e)

48.

(a)  during

(b)  on

(c)  without

(d)  by

(e)  down

Answer: (a)

49.

(a)  goes

(b)  falls

(c)  comes

(d)  moves

(e)  leaves

Answer: (c)

50.

(a)  above

(b)  down

(c)  besides

(d)  under

(e)  out

Answer: (b)

Part II Reasoning

51. Among five people – A, B, C, D and E, each scoring different marks, only one person scored less marks than B. D scored more than B, but less than A. A did not score the highest. Who scored the second highest?

(a)  E

(b)  Cannot be determined

(c)  A

(d)  C

(e)  D

Answer: (c)

Directions (Q. Nos. 52-54) Study the given information carefully to answer the given questions.

A is 14 m East of B. C is 6 m South of A. P is 4 m West of C. Point C is the midpoint of Points P and H, such that Points P, C and H form a straight line. Q is 6 m South of Point H.

52. Kunal walks 10 m towards North from Point H, takes a left turn, and walks for 4 m. How far will he be from Point C?

(a)  2 m

(b)  10 m

(c)  4 m

(d)  6 m

(e)  7 m

Answer: (b)

53. If A is 2 m to the North of L and R is 4 m West of Q, how far is Point L from Point R?

(a)  4 m

(b)  12 m

(c)  7 m

(d)  10 m

(e)  9 m

Answer: (d)

54.In which direction is B with respect to H?

(a)  North-East

(b)  South-East

(c)  North-West

(d)  North

(e)  West

Answer: (c)

Directions (Q. Nos. 55-59) Study the following information and answer the given questions

Nine friends- P, Q, R, S, T, U, V, W and X live on nine different floors of a building but not necessarily in the same order. The lower most floor of the building is numbered one, the one above that is numbered two and so on till the topmost floor is numbered nine. T lives on an odd numbered floor below the floor numbered five. Only three people live between T and V. Only one person lives between X and V. V lives above X. Q lives on an odd numbered floor immediately below P. Q does not  live on the floor numbered five. the number of persons living between X and P is equal to the number of people living between T and S. W lives on an even numbered floor above R.

55. S is related to the floor numbered seven and R is related to the floor numbered three in a certain way, same as X is related to which of the following floor numbers?

(a)  Seven

(b)  Nine

(c)  Five

(d)  Eight

(e)  Four

Answer: (e)

56. U lives on which of the following floor numbers?

(a)  Five

(b)  Four

(c)  Nine

(d)  Three

(e)  Other than those given as options

Answer: (c)

57.Four of the following five are alike in a certain way as per the given arrangement and thus form a group. Which of the following does not belong to that group?

(a)  UP

(b)  VW

(c)  XR

(d)  PR

(e)  QS

Answer: (d)

58. How many persons live below the floor on which W lives?

(a)  One

(b)  Two

(c)  More than three

(d)  Three

(e)  None

Answer: (d)

59. Which of the following statements is true as per the given arrangement?

(a)  Only three people live between S and V.

(b)  None of the given statements is true

(c)  Q lives immediately above T.

(d)  Less than three persons live above R.

(e)  U lives on the floor numbered nine.

Answer: (e)

60. How m any such pairs of digits are there in the number 71864392 (both in forward and backward directions), each of which has as many digits between them as in the arithmetic series?

(a)  More than three

(b)  None

(c)  Three

(d)  One

(e)  Two

Answer: (a)

61. Four of the following five are alike in a certain way (based on their positions of alphabets in the English alphabetical series) and hence form a group. Which is the one that does not belong to that group?

(a)  RQT

(b)  NPL

(c)  FHD

(d)  KMI

(e)  WYU

Answer: (a)

Directions (Q. Nos. 62-66) Study the following information to answer the given questions

Nine persons B, C, D, E, L, M, N, O and P are seated in a straight line facing North with equal distance between each other, but not necessarily in the same order.

As many people sit to the left of E as to the right of E. Only one person sits between E and O. L sits third to the left of P. P is not an immediate neighbour of O. Neither P nor D sits at any of the extreme ends of the line. Only three persons sits between B and C. B is not an immediate neighbour of E. the number of people sitting between B and E is double as that between E and N.

62. Who sits third to the right of B?

(a)  E

(b)  O

(c)  N

(d)  L

(e)  No one as B sits at one of the extreme ends of the line

Answer: (a)

63. Four of the following five are alike in a certain way based on the given arrangement and thus form a group. Which is the one that does not belong to that group?

(a)  C, E

(b)  M, D

(c)  O, C

(d)  B, L

(e)  D, B

Answer: (e)

64. Which of the following is true with respect to M as per the given arrangement?

(a)  M sits second to the left of O.

(b)  M is an immediate neighbour of L

(c)  More than two people sit between D and M.

(d)  None of the given options is true

(e)  M sits at one of the extreme ends of the line.

Answer: (e)

65. Who amongst the following sit exactly between L and P?

(a)  E, N

(b)  M, O

(c)  B, N

(d)  C, O

(e)  D, E

Answer: (c)

66. In which of the given pairs of people, is odd number of people sitting between them?

(a)  E, M

(b)  B, M

(c)  L, C

(d)  L, P

(e)  P, O

Answer: (a)

Directions (Q. Nos. 67-71) Study the following information to answer the given questions.

Gaurav watches seven movies viz. Gladiator, Bravheart, Titanic, Inception, Chinatown, Avatar and Passion on seven different days of the same week, starting from Monday and ending on Sunday, but not necessarily in the same order. Thus on one day he watches only one movie. Gaurav watches Inception on Friday. He watches only one movie between Inception and Titanic. He watches only three movies between Titanic and Gladiator. he watches only two movies between Titanic and Chinatown. Gaurav watches Avatar immediately before the day he watches Titanic. He watches Passion on one of the days after he watches Aavatar.

67. How many movies does Gaurav watch between Braveheart and Passion?

(a)  None

(b)  Three

(c)  One

(d)  Two

(e)  Four

Answer: (d)

68. Four of the following five are alike in a certain way based on the given arrangement and thus form a group. Which is the one that does not belong to that group?

(a)  Wednesday-Avatar

(b)  Friday-Braveheart

(c)  Thursday-Titanic

(d)  Saturday-Inception

(e)  Sunday-Chinatown

Answer: (b)

69. Which of the following is true as per the given arrangement?

(a)  Gaurav watches Inception on the day immediately after the day on which he watches Braveheart.

(b)  None of the given statements are true.

(c)  Gaurav watches only two movies between Avatar and Chinatown.

(d)  Gaurav watches Passion on Wednesday.

(e)  Gaurav watches Gladiator on Sunday.

Answer: (e)

70. On which day of the week does Gaurav watch Avatar?

(a)  Saturday

(b)  Sunday

(c)  Tuesday

(d)  Wednesday

(e)  Thursday

Answer: (c)

71. Which movie does Gaurav watch on Monday?

(a)  Chinatown

(b)  Gladiator

(c)  Titanic

(d)  Braveheart

(e)  Passion

Answer: (d)

Directions (Q. Nos. 72-76) In these questions. relationship between different elements is shown in the statement(s) are followed by two conclusions. Study the conclusions based on the given statements and

Give answer

(a) if only conclusion I is true

(b) if only conclusion II is true

(c) if either conclusion I or II is true

(d) if neither conclusion I nor II is true

(e) if both conclusions are true

72. Statements T = V ≤ R < B, T ≤ D

Conclusions I. D ≥ R       II. B ≤ T

Answer: (d)

73. Statements R < O ≥ E; Y = O > C

Conclusions I. Y > R       II. E < C

Answer: (a)

74. Statements P > R = S ≥ Q; M ≤ R

Conclusions I. M < P       II. Q ≤ M

Answer: (a)

75. Statements H < M ≥ I > N = C ≥ R

Conclusions I. C < H       II. M > R

Answer: (b)

76. Statements R < O ≥ E; Y ≥ O > C

Conclusions I. Y = E       II. Y > E

Answer: (c)

77. In a certain code language, ‘give me call’ is coded as ‘jo kl mx’ and ‘call for me’ is coded as ‘mx jo st’. How will ‘for’ be coded as in the given code language? (Note: all codes are two letter codes only)

(a)  Either ‘mx’ or ‘jo’

(b)  Either ‘jo’ or ‘kj’

(c)  kj

(d)  mx

(e)  st

Answer: (e)

Directions (Q. Nos. 78-82) In these questions, two/three statements followed by two conclusions number I and II are given. You have to take the given statements to be true even if they seem to be at variance from commonly known facts and then decide which of the given conclusions logically follows from the given statements.

Give answer

(a) if only conclusion I follows

(b) if only conclusion II follows

(c) if either conclusion I or II follows

(d) if neither conclusion I or II follows

(e) if both conclusions follow

78. Statements No horse is a goat. Some goats are deers.

Conclusions I. No horse is a deer.

II. At least some horses are deers.

 

Answer: (a)

79. Statements Some trees are branches. Some braches are roads. No road is a sky.

Conclusions I. No tree is a sky.

II. Some branches are definitely not skies.

Answer: (a)

80. Statements All plates are spoons. All spoons are glasses. Some glasses are mugs.

Conclusions I. All plates being mugs is a possibility.

II. All glasses are plates.

Answer: (d)

81. Statements All plates are spoons. All spoons are glasses. Some glasses are mugs.

Conclusions I. Some mugs are spoons.

II. No mug is a spoon.

Answer: (d)

82. Statements Some trees are branches. Some branches are roads. No road is a sky.

Conclusions I. All roads are trees.

II. All skies are branches.

Answer: (d)

Directions (Q. Nos. 83-85) Study the given information carefully to answer the given question.

K and M are the children of G. G is married to R. S is the sister of G. A is the only son of R. P is the son of K.

83. How is M related to P?

(a)  Father

(b)  Uncle

(c)  Aunt

(d)  Grandmother

(e)  Brother-in-law

Answer: (c)

84. How is S related to R?

(a)  Sister-in-law

(b)  Daughter-in-law

(c)  Niece

(d)  Daughter

(e)  Grandmother

Answer: (a)

85. If S does not have any sister, then how is G related to P?

(a)  Uncle

(b)  Grandfather

(c)  Father-in-law

(d)  Aunt

(e)  Grandmother

Answer: (e)

Directions (Q. Nos. 86-90) Study the given information carefully to answer the given questions.

Eight people M, N, O, P, Q, R, S and T are sitting around a square table (but not necessarily in the same order) in such a way that four of them sit at the corners while four sit in the middle of each of the four sides. The ones sitting at the corners are facing the centre and the ones sitting in the middle of the sides are facing outside. (i.e. opposite to the centre)

O sits in the middle f one of the sides. Only two people sit between O and R. T sits to the immediate right of R. T and S face the same direction. N sits to the immediate left of S. P is an immediate neighbour of N. M sits third to the right of P.

86. What is the positions of P with respect to T?

(a)  Third to the left

(b)  Second to the left

(c)  Immediate left

(d)  immediate right

(e)  Second to the right

Answer: (e)

87. Which of the following pairs represents the people sitting between O and the one sitting to the immediate right of P, when counted from the left of O?

(a)  N, S

(b)  R, N

(c)  S, M

(d)  T, Q

(e)  R, T

Answer: (c)

88. How many people sit between T and M when counted from the right of T?

(a)  More than three

(b)  Two

(c)  One

(d)  Three

(e)  None of these

Answer: (a)

89. Which of the following statements is true with respect to the given arrangement?

(a)  None of the given statements is true.

(b)  Q is an immediate neighbour of T.

(c)  S sits at one of the corners of the table.

(d)  Only two people sit between M and Q.

(e)  R and N face opposite directions

Answer: (b)

90. Who sits third to the left of O?

(a)  R

(b)  M

(c)  P

(d)  N

(e)  S

Answer: (d)

Directions (Q. Nos. 91-95) Study the following information carefully to answer the given questions.

Eight friends A, B, C, D, E, F G and H are seating around a circular table. Some of them are facing inside & others are facing outside. Opposite direction means if one is facing inside the centre, second one face outside the centre and Vice versa.

Three people are seating between F and D and both face opposite direction to each other. E is seating second to right of both D and F and face opposite direction as F faces. C is seating third to right of E  who is not opposite to B. G is neighbour of both E and D, and seating second to right of B, who is not a neighbour of E. G is third to left of A, who is third to right of both H and G. B face towards the centre and is seating second to left of C.

91. If all the friends are seating according to alphabetical order in anti clock wise direction, starting from A, how many friends remain at same position (excluding A)?

(a)  One

(b)  Two

(c)  Three

(d)  Four

(e)  None of these

Answer: (b)

92. If D interchanged his position with H and, F interchanged his position with G, who sits immediately right of H in new arrangement?

(a)  F

(b)  G

(c)  E

(d)  A

(e)  None of these

Answer: (a)

93. Who is sitting third to the left of B?

(a)  H

(b)  C

(c)  A

(d)  F

(e)  None of these

Answer: (d)

94. How many persons are facing away from the centre?

(a)  H, B

(b)  C, G

(c)  F, D

(d)  E, A

(e)  None of these

Answer: (b)

95. Who among the following pairs are facing same direction and seating opposite to each other?

(a)  H, B

(b)  B, G

(c)  F, D

(d)  E, A

(e)  None of these

Answer: (a)

Directions (Q. Nos. 96-100) Read the given information to answer the given questions.

Ten friends are sitting on twelve seats in two parallel rows containing five people each, in such a way that there is an equal distance between adjacent persons. In Row 1: A, B, C, D and E are seated and all of them are facing South and in Row 2: P, Q, R, S and T are sitting and all of them are facing North. One seat is vacant in each row. Therefore, in the given seating arrangement each member seated in a row faces another member of the other row.

All of them like different colors – Red, Green, Black, Yellow, White, Blue, Brown, Purple, Pink and Grey, but not necessarily in the same order.

There are two seats between Q and the vacant seat. Q does not like White, Red and Purple. E is not an immediate neighbor of C. B likes Grey. Vacant seat of row 1 is not opposite to S and is also not at any of the extreme ends of Row-1. The one who likes Black sits opposite to the one, who sits third to the right of the seat, which is opposite to S. C is not an immediate neighbor of D. T, who likes neither White nor Blue, does not face vacant seat. D faces R. The vacant seats are not opposite to each other. Two seats are there between E and B. The one who sits third right of the seat, on which the person who likes brown is sitting. S sits third to the right of seat on which R sits and likes Yellow. the one who likes Pink faces the one who likes Yellow. The persons who like Red and Purple are adjacent to each other. The vacant seat in row 1 is not adjacent to D. Q sits at one of the extreme ends. E neither likes Pink nor faces the seat which is adjacent to the one who likes Blue. The one who likes Yellow. The persons who likes Green doesn’t face the person who likes Purple.

96. How many persons are sitting between T and the one who likes Yellow colour?

(a)  None

(b)  One

(c)  Two

(d)  Three

(e)  None of these

Answer: (c)

97. Which of the following faces the vacant seat of Row-2

(a)  The one who like White colour

(b)  A

(c)  D

(d)  The one who like Grey colour

(e)  Cannot be determined

Answer: (d)

98. Who is sitting at the immediate left of person who likes Purple colour?

(a)  T

(b)  The one who likes black colour

(c)  D

(d)  The one who likes Green colour

(e)  The one who likes Grey colour

Answer: (a)

99. Who amongst the following sits at the extreme end of the row?

(a)  R, Q

(b)  E, S

(c)  T, C

(d)  C, D

(e)  None of these

Answer: (b)

100. If Q is made to sit on vacant seat of his row, then how many persons are there between the persons who sit opposite to Q now and who sat opposite to Q previously?

(a)  Two

(b)  Three

(c)  Four

(d)  None

(e)  One

Answer: (a)

Part III Quantitative Aptitude

Directions (Q. Nos. 101-110) Each question below is followed by two statements I and II. You are to determine whether the data given in the statement is sufficient for answering the question. You should use the data and your  knowledge of Mathematics to choose between the  possible answer.

Give answer

(a) if the statement I alone is sufficient to answer the question, but the statement II alone is not sufficient.

(b) if the statement II alone is sufficient to answer the question but the statement I alone is not sufficient.

(c) if both statements I and II together are needed to answer the question

(d) if either the statement I alone or statement II alone is sufficient to answer the question.

(e) if answer cannot be obtained from the statements I and II together but need even more data.

101. What is the area of the circle?

(I) Perimeter of the circle is 88 cms.

(II) Diameter of the circle is 28 cms.

Answer: (d)

102. What is the rate of interest?

(I) Simple Interest accrued on an amount of Rs25000 in two years is less than the compound interest for the same period by Rs 250.

(II) Simple Interest accrued in 10 years is equal to the principal.

Answer: (d)

103. What is the number of trees planted in the field in rows and columns?

(I) Number of columns is more than the number of rows by 4.

(II) There are five rows of trees in the field.

Answer: (e)

104. What is the speed of the current?

(I) A man can swim a distance of 9 km in  downstream

(II) While coming back upstream it takes him 3 hours to cover the same distance.

Answer: (c)

105. What is the minimum passing percentage in a test?

(I) Raman scored 25% marks in the test and Sunil scored 288 marks which is 128 more than Raman.

(II) Raman scored 64 marks less than the minimum passing marks.

Answer: (c)

106. What is the value of x2 + y + z?

(I) 4x + 3y + 5z = 60 and 2x = y, 2y = z

(II) 3x + 3y + 2z = 34 and 2x + 5y+ 6z = 72

Answer: (c)

107. Whose body weight is second highest among the five boys Arun, Vinay, Suraj, Raju and Pratap?

(I) Average weight of Arun, Suraj and Vinay is 68 kg and average weight of Raju and Pratap is 72 kg.

(II) Average weight of Arun, Suraj, Vinay and Raju is 68 kg and also Suraj is 78 kg. Raju is 68 kg and Vinay is 46 kg. All of them have different weight.

Answer: (a)

108. What is Subodh’s marks in Physics?

(I) The average marks of Subodh in History, Geography and Chemistry are 75.

(II) His average marks in History, Geography and Physics are 78.

Answer: (e)

109. What is the population of the city A?

(I) The ratio of the population of males and females in city A is 27 : 23 and the difference between their population is 100000.

(II) The population of city A is 80% of that of city B. The difference of population of city A and city B is 312500.

Answer: (a)

110. How many students did participate in elocution?

(I) The students who participate in dancing were 150% were than that who participated in elocution.

(II) 150 students participated in dancing.

Answer: (c)

Directions (Q. Nos. 111-115) Study the following graph carefully and answer the given question below it.

111. Expenditure of company B in 2009 and 2010 are Rs 12 lakh and Rs 14.5 lakh respectively. What was the total income of company B in 2009 and 2010 together?

(a)  Rs 39.75

(b)  Rs 37.95

(c)  Rs 38.75

(d)  Rs 38.55

(e)  None of these

Answer: (b)

112. Ratio of expenditure of company A and B in 2012 was 3 : 4 respectively. What was the respective ratio of their incomes in 2012?

(a)  21 : 26

(b)  13 : 14

(c)  14 : 13

(d)  26 : 21

(e)  None of these

Answer: (e)

113. Total expenditure of company A in all the years together was 82.5 lakhs. What was the total income of the company A in all the years together?

(a)  1.23 crores

(b)  98.75 crores

(c)  99.85 crores

(d)  Cannot be determined

(e)  None of these

Answer: (d)

114. If the expenditure of company A and B in 2013 were equal and the total incomes of the two companies was Rs 5.7 lakh. What was the total expenditure of the two companies in 2013?

(a)  4 lakh

(b)  2 lakh

(c)  4.2 lakh

(d)  Cannot be determined

(e)  None of these

Answer: (b)

115. If the income of company B in 2010 and 2011 were in the ratio of 2 : 3 respectively. What was the respective ratio of expenditure of that company in these two years?

(a)  20 : 29

(b)  9 : 10

(c)  29 : 45

(d)  10 : 29

(e)  None of these

Answer: (c)

Directions (Q. Nos. 116-120) Study the pie-charts carefully to answer the given questions.

116. Number of girls enrolled in dancing form are what percent of total number of students enrolled in the school? (Rounded off to two digits after decimal)

(a)  12.83

(b)  14.12

(c)  11.67

(d)  10.08

(e)  None of these

Answer: (a)

117. How many boys are enrolled in Singing and Craft together?

(a)  750

(b)  610

(c)  485

(d)  420

(e)  None of these

Answer: (a)

118. What is the respective ratio of number of girls enrolled in swimming to the number of boys enrolled in Craft?

(a)  47 : 49

(b)  23 : 29

(c)  29 : 23

(d)  49 : 47

(e)  None of these

Answer: (d)

119. What is the total number of girls enrolled in Craft and Drawing together?

(a)  480

(b)  525

(c)  505

(d)  495

(e)  None of these

Answer: (b)

120. What is the appropriate percentage of boys in the school?

(a)  34%

(b)  56%

(c)  42%

(d)  50%

(e)  None of these

Answer: (c)

Directions (Q. Nos. 121-130) In each of the question, two equations numbered I and II are given. You have to solve both the equations and

Give answer

(a) x < y        (b) x > y      (c) x ≥ y     (d) x ≤ y

(e) relationship between x and y cannot be established or x = y

121 (I) 6x2 + 5x + 1 = 0 (II) 15y2 + 8y + 1= 0

Answer: (d)

122. (I) x2 + 5x + 6 = 0 (II) 4y2 + 27y + 35= 0

Answer: (e)

123. (I) 2x2 + 5x + 3 = 0 (II) y2 + 9y + 14 = 0

Answer: (b)

124. (I) 88x2 – 19x + 1 = 0 (II) 132y2 – 23y + 1 = 0

Answer: (c)

125. (I) 6x2 – 7x + 2 = 0 (II) 20y2 – 31y + 12 = 0

Answer: (a)

126. (I) 6x2 + 23x + 20 = 0 (II) 6y2 + 31y + 35 = 0

Answer: (e)

127. (I) x2 = 81 (II) y2 – 18y + 81 = 0

Answer: (d)

128. (I) 4x2 + 20x + 21 = 0 (II) 2y2 + 17y + 35 = 0

Answer: (b)

129. (I) x2 – 14x + 48 = 0 (II) y2 + 6 = 5y

Answer: (b)

130. (I) 38x2 – 3x – 11 = 0 (II) 28y2 + 32y + 9 = 0

Answer: (c)

131. Two men P and Q start a journey from same place speed at a speed of 3 km/h and 3 1/2 km/h respectively. If they move in the same direction then, what is the distance between them after 4 hours?

(a)  3 km

(b)  2 1/2 km

(c)  2 km

(d)  3 1/2 km

(e)  None of these

Answer: (c)

Directions (Q. Nos. 132-135) What will come in place of question marks in the given questions?

132. 

(a)  5√5

(b)  (125)3

(c)  25

(d)  5

(e)  None of these

Answer: (d)

133. 

(a)  10

(b)  8

(c)  512

(d)  324

(e)  None of these

Answer: (a)

134. 

(a)  36 + 44√(7)

(b)  6

(c)  216

(d)  36

(e)  None of these

Answer: (d)

135. 

(a)  6320

(b)  6400

(c)  6350

(d)  6430

(e)  6490

Answer: (c)

136. 28 39 63  102  158  ?

(a)  232

(b)  242

(c)  233

(d)  244

(e)  None of these

Answer: (c)

137. 7 16 141  190  919  ?

(a)  1029

(b)  1019

(c)  1020

(d)  1030

(e)  None of these

Answer: (e)

138. 12 17 32  57  92  ?

(a)  198

(b)  195

(c)  137

(d)  205

(e)  None of these

Answer: (c)

139. 19 25 45  87  159  ?

(a)  254

(b)  279

(c)  284

(d)  269

(e)  None of these

Answer: (d)

140. 83 124 206  370  698  ?

(a)  1344

(b)  1324

(c)  1364

(d)  1334

(e)  None of these

Answer: (e)

Directions (Q. Nos. 141-147) Study the table carefully and answer the given questions.

141. What is the difference between the number of academic books published by publishing house M and P?

(a)  450

(b)  640

(c)  540

(d)  504

(e)  None of these

Answer: (c)

142. How many books were given to each distributor by publisher Q if each publisher gets equal number of books?

(a)  1806

(b)  1068

(c)  1608

(d)  1308

(e)  None of these

Answer: (b)

143. What is the average number of non-academic books published by publishers R and S?

(a)  18750

(b)  18850

(c)  19950

(d)  18950

(e)  19990

Answer: (c)

144. If the total number of books publishers P, Q and R is increased by 30% and the total number of books published by remaining publishers be decreased by 20%, what will be the new average of books published by all the publishers?

(a)  33418

(b)  33318

(c)  32518

(d)  33618

(e)  None of these

Answer: (b)

145. What is the total number of books distributed by publishers O and Q?

(a)  26702

(b)  27324

(c)  55026

(d)  54026

(e)  None of these

Answer: (d)

146. Meena kumara goes to a shop and buys a saree, costing Rs 5225 including sales tax of 12%. The shopkeeper gives her a discount, so that the price is decreased by an amount equivalent to sales tax. The price is decreased by (nearest value)

(a)  Rs 615

(b)  Rs 650

(c)  Rs 560

(d)  Rs 580

(e)  Rs 680

Answer: (c)

147. Phanse invests an amount of Rs 24200 at the rate of 4% per annum for 6 years to obtain a simple interest. later he invests the principal amount as well as the amount obtained as simple interest for another 4 years at the same rate of interest. What amount of simple interest will be obtain at the end of the last 4 years?

(a)  Rs 4800

(b)  Rs 4850.32

(c)  Rs 4801.28

(d)  Rs 4700

(e)  Rs 4870.32

Answer: (c)

Directions (Q. Nos. 148-149) These questions are based on the following information

There are three different cable channels namely ahead, luck and bang. In a survey, it was found that85% of viewers, respond to bang, 20% to luck and 30% of ahead. 20% of viewers respond exactly two channels and 5% to none.

148. What percentage of the viewers responded to all three?

(a)  10%

(b)  12%

(c)  14%

(d)  16%

(e)  11%

Answer: (a)

149. Assuming 20% respond to ahead and bang and 16% respond to bang and luck. What is the percentage of viewers who watch only luck?

(a)  20%

(b)  10%

(c)  16%

(d)  18%

(e)  14%

Answer: (e)

150. A milkman mixes 20 L of water with 80 L of milk. After selling one-fourth of this mixture, he adds water to replenish the quantity that he has sold. What is the current proportion of water to milk?

(a)  2 : 3

(b)  1 : 2

(c)  1 : 3

(d)  2 : 1

(e)  3 : 4

Answer: (a)

IBPS Clerk Pre (Phase I) Online Examination Held on December 4, 2017 Question Paper With Answer Key

IBPS Clerk Pre (Phase I) Online Examination Held on December 4, 2017
IBPS Clerk Pre (Phase I) Online Examination Held on December 4, 2017 Question Paper With Answer Key

IBPS Clerk Pre (Phase I) Online Examination Held on December 4, 2017

Part I English Language

 

Directions (Q. Nos. 1-5) Read the following passage carefully and answer the questions given below it.

When Hurricane Harvey loomed off the coast of my home State of Texas, it seemed to fill the entire Gulf of Mexico.

When it roared on land, it pummeled the towns Rockport and Port Aranas, whose tawny beaches I’ve walked with my kids, pointing out the indigo sails of Portuguese man O’war jellyfish. Harvey’s eye took direct aim at the University of Texas’ Marine Science Institute, flattening not just the facility itself, but priceless samples awaiting analysis. After Harvey left Port Aransas, it spun back into the Gulf of Mexico over record sea temperatures as great as 4° above normal. Thermodynamic laws require that warmer air holds more water vapour. The heat armed the storm with a mighty arsenal of water vapour. Then Harvey returned to land, dumping a catastrophic amount of rain on Houston. My Facebook feed filled with pleas for rescue from the rising waters. Friend’s houses flooded – houses that had always been on dry land before A chemical plant blew up twice Toxic chemicals oozed from Superfund sites. Dozens died in the deluge, mostly by drowning. And all the while, alongside the heartbreak and horror, I kept thinking about a strange harbinger : jellyfish. Play Video Diaphanous in form yet menacing in sting, jellyfish have a powerful capacity to capture our imagination. They undulate in a primal rhythm, blinking open and closed like eyes that can peer into the soul of the sea. And what they are seeing are changes produced by us here on land. Because we burn fossil fuels, which release greenhouse gasses, not just the atmosphere, but ocean waters are warming. At the same time, our ship traffic transports animals to new places, and sometimes these exotics find home-like conditions where in the past those conditions would have been unsuitable. That’s what happened in the eastern Mediterranean, where a jellyfish from the tropical Indian Ocean has found warm, homey waters and now forms huge aggregations called blooms that stretch for tens of miles every summer. The fierce stings of these animals chase beach-goers out of the water. Their gooey bodies clog machinery at power plants, halting operations.

Rampant coastal development provides new habitat for a jellyfish stage called polyp that looks like a sea anemone. When it finds a hard surface like a dock or a jetty to grow on, a single polyp can proliferate into a dozen or even more medusa. And fields of polyps grow on those hard structures. That’s likely what happened off the coast of Italy, where gas platforms are thought to be the home for a new invasion of jellyfish.

In the twentieth century in the Adriatic sea, moon jellies, pinkish with their characteristic four-leafed clover on top, were a rarity. Now they are ubiquitous. And as we wash pollution into our waters, we create low-oxygen environments. Some jellyfish, with their flow metabolic rate due to their a-cellular jelly insides, can survive more easily there than fish, with their oxygen guzzling muscled tissues. That is part of what happened in the Yellow sea, where pollution is unchecked. It is the birthplace of a maroon jellyfish that reaches a weight of 500 pounds. Blooms of the creature were a once-a-generation event before 2000 the kind of thing fishermen mentioned to their sons. But jellyzillas swept from China in the Tsushima Current, have plagued Japan’s coast almost every year of the 21st century. In 2009, a fishing boat caught so many that their weight capsized the vessel. (Fortunately, the crew members were rescued.) And our lack of oversight of the fishing industry, which has removed more than 90% of the large fish from the seas, has depleted the predators of jellyfish as well as their competitors. Jellyfish are eaten by some fish, and jellyfish eat the same small zooplankton that fish do. The ecological vacuum left by unrestrained fishing can allow jellyfish to expand their influence in marine ecosystems. That’s what happened off the coast of Namibia, once one of the world.

1. How is jellyfish able to survive in polluted water more than the other fishes?

(1) Jellyfish has oxygen guzzling muzzled tissues.

(2) Due to low metabolic rate.

(3) Jellyfish eat small zooplankton to survive.

(a)   Only 1

(b)   Only 2

(c)   1 and 2

(d)   2 and 3

(e)   All are correct

Answer: (e)

2. How Hurricane Harvey can be termed as devastating with reference to the passage?

(a)   Many peopled died by drowning.

(b)   Hurricane blew up the Chemical plants.

(c)   Toxic chemicals got exuded from superfund sites.

(d)   Hurricane flooded many houses.

(e)   All of the above

Answer: (e)

3. What is the author’s main idea regarding the passage?

(a)   How to ameliorate ecological imbalance.

(b)   The author gave his opinion regarding jellyfish that what they know humans don’t.

(c)   Water pollution creating a low oxygen environment affecting aquatic life.

(d)   Hurricane Harvey destroyed many lives

(e)   Endangered condition of Jellyfish

Answer: (b)

4. Which of the following statements is/are true in context of the given passage?

(a)   Maroon jellyfish is found in Yellow sea where pollution is unchecked.

(b)   Hurricane Harvey with strong winds battered the entire Gulf of Mexico.

(c)   Jellyzillas sweeping from China have afflicted Japan’s coast.

(d)   Jellyfish survive more than other fishes in low oxygen environment

(e)   All of the above

Answer: (e)

5. Which of the following statements regarding jellyfish is/are related to the country Namibia?

(a)   The attenuation in the number of jellyfish is due to the low oxygen environment.

(b)   The predators of jellyfish are depleting because of uncontrolled fishing.

(c)   Jellyfish can proliferate their population in the situation of ecological imbalance.

(d)   Both (b) and (c)

(e)   All of the given options

Answer: (d)

Directions (Q. Nos. 6-11) In each of the questions given below a sentence is given with one blank. Bellow each sentence four words are given out of which two can fit the sentence. Five options are given with various combinations of these words. You have to choose the combination with the correct set of words which can fit in the given sentence.

6. Would you like to have …….. coffee?

(A) Much      (B) More

(C) some       (d) many

(a)   A-B

(b)   A-D

(c)   B-C

(d)   C-D

(e)   A-C

Answer: (c)

7. The Dysart Unified School District says they are investigating threats made …….. the district and Dysart High School.

(A) towards   (B) against

(C) for           (D) after

(a)   A-B

(b)   A-D

(c)   B-C

(d)   C-D

(e)   A-C

Answer: (a)

8. I was going ……… to say hello when I realized that I couldn’t remember his name.

(A) beyond    (B) over

(C) through   (D) across

(a)   A-B

(b)   A-D

(c)   B-C

(d)   C-D

(e)   B-D

Answer: (c)

9. In 1997 the sales tax was lowered to 4%, then in 2001 it was abolished ………

(A) together             (B) laterally

(C) completely        (D) altogether

(a)   A-B

(b)   A-D

(c)   B-C

(d)   C-D

(e)   B-D

Answer: (d)

10. Children ….. their parents’ authority far more nowadays than they did in the past.

(A) dispute             (B) question

(C) confront            (D) challenge

(a)   A-B

(b)   A-D

(c)   B-C

(d)   C-D

(e)   B-D

Answer: (e)

11. The United Nations was ………. its authority to restore peace in the area.

(A) Used       (B) Blow

(C) Exercised (D) Refused

(a)   A-C

(b)   A-D

(c)   B-C

(d)   C-D

(e)   B-D

Answer: (a)

Directions (Q. Nos. 12-20) In each of the question given below a/an idiom/phrase is given in bold which is then followed by five options which then tries to decipher its meaning as used in the sentence. Choose the option which gives the meaning of the phrase most appropriately in context of the given sentence.

12. She comes across really well on television.

(a)   discover

(b)   express

(c)   behave

(d)   offers

(e)   provide

Answer: (e)

13. The deal was completely open and above board.

(a)   mislead

(b)   profitable

(c)   dupe

(d)   respect

(e)   honest

Answer: (e)

14. The most businessmen, the central questions will turn on

(a)   matter

(b)   consider

(c)   attack

(d)   concern

(e)   unsettle

Answer: (d)

15. They sought a controlling interest rather than a takeover.

(a)   spend

(b)   partnership

(c)   endorse

(d)   approve

(e)   buyout

Answer: (e)

16. I think he was just putting on an act to get sympathy.

(a)   tolerate

(b)   assume

(c)   perform

(d)   deceive

(e)   effect

Answer: (b)

17. I can put up with the house being messy, but I hate it if it’s not clean.

(a)   ignore

(b)   forget

(c)   delay

(d)   remain

(e)   bear

Answer: (e)

18. He gave in to my suggestion after I had shown him the plans.

(a)   leave

(b)   defeated

(c)   defy

(d)   agree

(e)   resign

Answer: (d)

19. The cut in on a station wagon, forcing the driver to brake.

(a)   interrupt

(b)   stop

(c)   collide

(d)   too close

(e)   carve

Answer: (a)

20. Most patients find that the numbness from the injection wears off after about an hour.

(a)   calm

(b)   inflate

(c)   deepen

(d)   fade

(e)   emerge

Answer: (d)

Directions (Q. Nos. 21-30) In each of the question given below a sentence is given which is then divided into five parts out of which last part is correct. There is an error in three part of the sentence and only one part is correct. You have to choose the part as y our answer.

21. After he had read / the first two chapters / to the novel, / he had felt like/ reading the book in one sitting.

(a)   After he had read

(b)   the first two chapters

(c)   to the novel

(d)   he had felt like

(e)   All are incorrect

Answer: (b)

22. Since most of the urban people/ have been lived / at polluted areas/ they suffers from severe / diseases caused by pollution.

(a)   Since most of the urban people

(b)   have been lived

(c)   at polluted areas

(d)   they suffers from severe

(e)   All are incorrect

Answer: (a)

23. Broadly speaking of a / layman language disability / could be / classified in two groups/ namely mental and physical.

(a)   Broadly speaking of a

(b)   layman language disability

(c)   could be

(d)   classified in two groups

(e)   All are incorrect

Answer: (c)

24. Although there are some / similarities at the disqualification / of both the candidate/ the differences among them / are considerably pronounced.

(a)   Although there are some

(b)   similarities at the disqualification

(c)   of both the candidate

(d)   the differences among them

(e)   All are incorrect

Answer: (a)

25. When the group of teenagers/ visits the entertainment centre/ little did they knows/ that its outing/ would lead them to a hospital.

(a)   When the group of teenagers

(b)   visits the entertainment centre

(c)   little did they knows

(d)   that its outing

(e)   All are incorrect

Answer: (a)

26. If the present guidelines,/ the bank are required/ to obtaining a photograph/ from any persons/ who wishes to open an account.

(a)   If the present guidelines

(b)   the bank are required

(c)   to obtaining a photograph

(d)   from any persons

(e)   All are incorrect

Answer: (e)

27. Although the clock struck twelve, / we hear the big bang of! the fire crackers and saw/ all the guests scream,/ shouting and wishing each other with joy.

(a)   Although the clock struck twelve,

(b)   we hear the big bang of

(c)   the fire crackers and saw

(d)   all the guests scream,

(e)   All are incorrect

Answer: (c)

28. The Rupali wanted/ to gets/ the clear picture about/ the incident so she spoke/ to the victism.

(a)   The Rupali wanted

(b)   to gets

(c)   the clear picture about

(d)   the incident so she spoke

(e)   All are incorrect

Answer: (d)

29. The centre has accepted/ the report of the Judicial Commission/ that indicted a former Chief Minister/ and six of his ministerial colleagues for corruption,/ favouritism, nepotism and administrative impropriety.

(a)   The centre has accepted

(b)   the report of the Judicial Commission

(c)   that indicted a former Chief Minister

(d)   and six of his ministerial colleagues for corruption

(e)   All are incorrect

Answer: (a)

30. Before invested,/ you should look at the/ overall business dynamics for the company/ along with its efficient management and/ good corporate governance.

(a)   Before invested

(b)   you should look at the

(c)   overall business dynamics for the company

(d)   along with its efficient management and

(e)   All are incorrect

Answer: (b)

Part II Reasoning

Directions (Q. Nos. 31-35) Study the following information carefully and answer the questions given below.

There are eight persons namely S, T, U, V, W, X, Y and Z lives on eight different floors from one to eight. Ground floor is number one and one above that is number two and so on till the topmost floor is number eight.

X lives on odd number floor but does not live on 3rd floor. Z lives immediate below X. More than two person lives between Z and Y. There are six person lives between S and Y. V lives immediate above W, but live below T. U does not live above X. W does not live immediate above Y.

31. Who lives on floor number five?

(a)   U

(b)   S

(c)   Z

(d)   T

(e)   None of these

Answer: (d)

32. How many persons live between W and X?

(a)   One

(b)   Three

(c)   Five

(d)   Two

(e)   None of these

Answer: (b)

33. Who lives immediately above V?

(a)   Z

(b)   T

(c)   Y

(d)   W

(e)   None of these

Answer: (b)

34. Four of the following five are alike in a certain way based on the given arrangement and thus form a group. Which is the one that does not belong to that group?

(a)   Y

(b)   V

(c)   T

(d)   X

(e)   W

Answer: (b)

35. Who lives on 2nd floor?

(a)   X

(b)   S

(c)   U

(d)   Z

(e)   None of these

Answer: (c)

Directions (Q. Nos. 36-38) In each of the questions below are given three statements followed by two conclusions numbered I and II. You have to take the given statements to be true even if they seem to be at variance from commonly known facts. Read all the conclusions and then decide which of the given conclusions logically follows from the given statement disregarding commonly known facts.

Give answer

(a) if only conclusion I is true

(b) if only conclusion II is true

(c) if both conclusion I and II are true

(d) if either conclusion I or II is true

(e) if neither conclusion I nor II is true

36. Statements Some dares are dream. All dreams are real. No real is fake.

Conclusions I. Some dreams are fake.

II. All dream is not fake.

Answer: (b)

37. Statements No rain is game. Some games are chain. No chain is lane.

Conclusions I. Some chains are not rain.

II. Some games are lane.

Answer: (a)

38. Statements Some movies are tickets. No ticket is popcorn. Some popcorn in burger.

Conclusions I. Some movies are burger.

II. Some burgers are not movies.

Answer: (b)

Directions (Q. Nos. 39-43) Study the following information carefully and answer the questions given below.

Eight friends A, B, C, D, P, Q, R and S are sitting in a straight line (but not necessarily in the same order). Some of them are facing South while some are facing North.

Q sits at one of t he extreme ends of the line. C sits third to the left of Q. B, is not an immediate neighbour of Q. P sits third to the right of C. A faces North. D sits to the immediate right of P. D does not sits at any of the extreme end of the line. Only one person sits between R and B. P sits second to the left of R. S sits second to the right of D. Both the immediate neighbour of P faces the same direction to each other. Both the immediate neighbour fo C faces the opposite direction to each other. S faces the same direction as D and A faces the same direction as R.

None Facing the same direction means if one is facing North then the other also faces North and vice versa. Facing opposite direction means if one if acing North then the other faces South and vice versa.

39. How many persons sits between D and A?

(a)   One

(b)   None

(c)   Three

(d)   Two

(e)   None of these

Answer: (c)

40. Who among the following pair sits at extreme ends?

(a)   S, Q

(b)   D, Q

(c)   Q, R

(d)   S, R

(e)   None of these

Answer: (a)

41. Who sits third to the left of D?

(a)   R

(b)   B

(c)   Q

(d)   C

(e)   None of these

Answer: (b)

42. Who sits to the immediate right of B?

(a)   A

(b)   C

(c)   R

(d)   S

(e)   None of these

Answer: (b)

43. Four of the following five are alike in a certain way based on the given arrangement and thus form a group. Which is the one that does not belong to that group?

(a)   A

(b)   C

(c)   S

(d)   B

(e)   D

Answer: (a)

Directions (Q. Nos. 44-46) In these questions, relationship between different elements is shown in the statements. These statements are followed  by two conclusions. Study the conclusion based on the given statements and select the appropriate answer

Give answer

(a) if only conclusion I follows

(b) if only conclusion II follows

(c) if both conclusion follow

(d) if either conclusion I or II follows

(e) if neither conclusion I nor II follows

44. Statements Z > W > V = K < L < I

Conclusions I. W > K      II. I > K

Answer: (c)

45. Statements Q > B, K < E < B, J ≥ E, R < Q

Conclusions I. Q > K       II. Q = E

Answer: (a)

46. Statements E = F < G < H; G ≥ I

Conclusion I. H > I          II. E > I

Answer: (a)

Directions (Q. Nos. 47-51) Study the following alphabetical sequence and answer the questions based on it.

ABBCDEFEIBCAFECBBACAOBNUVW

47. How many Cs are there in the alphabetical series which are immediately preceded by a vowel and immediate followed by consonant?

(a)   One

(b)   Two

(c)   Three

(d)   More than three

(e)   None of these

Answer: (a)

48. If all the vowels are dropped from the series, then which alphabet will be eighth from the left end?

(a)   C

(b)   B

(c)   N

(d)   F

(e)   None of these

Answer: (d)

49. How many vowels are there in the alphabetical series which are immediately preceded by a consonant?

(a)   One

(b)   Two

(c)   Four

(d)   More than five

(e)   None

Answer: (d)

50. If the position of the 1st and the 14th alphabets, the 2nd and the 15th alphabets and so on up to the 13th and the 26th alphabets, are interchanged, Then which alphabets will be 7th to the right of 10th alphabets from the right end?

(a)   A

(b)   C

(c)   N

(d)   B

(e)   None of these

Answer: (b)

51. How many total vowels in the alphabetical series?

(a)   Five

(b)   Ten

(c)   Three

(d)   Nine

(e)   None of these

Answer: (b)

Directions (Q. Nos. 52-56) Study the following information carefully and answer the questions given below.

There are eight notes of different denominations ie., Rs 1, Rs 5, Rs 20, Rs 50, Rs 100, Rs 200, Rs 500, Rs 2000 which are arranged in a circular arrangement facing towards the centre but not necessarily in the same order. Rs 50 note is arranged second to the right of Rs 200 note. Odd denominations note will not be immediate neighbour of Rs 200 and Rs 50 note. There will be less than Rs 100 denomination note to the immediate left and immediate right of Rs 100 note except Rs 1 note. Rs 100 note is second to the left of Rs 5 note. Rs 2000 note is second to the left of Rs 100 note.

52. Which rupee not will be arranged to the immediate left of Rs 50 note?

(a)   Rs 100

(b)   Rs 2000

(c)   Rs 500

(d)   Rs 5

(e)   None of these

Answer: (b)

53. Which rupee note will be arranged to the third to the right of Rs 5 note?

(a)   Rs 2000

(b)   Rs 100

(c)   Rs 200

(d)   Rs 20

(e)   None of these

Answer: (c)

54. What will be the sum of note which is immediate left of Rs 50 note and immediate right of Rs 500 note?

(a)   Rs 2100

(b)   Rs 550

(c)   Rs 700

(d)   Rs 2200

(e)   None of these

Answer: (d)

55. Which denomination note are between Rs 1 and Rs 100 note, when counted clockwise from Rs 1 note?

(a)   Rs 5 and Rs 20

(b)   Rs 50 and Rs 100

(c)   Rs 200 and Rs 2000

(d)   Rs 1 and Rs 500

(e)   None of these

Answer: (a)

56. Which rupee note will be arranged to the third to the right of Rs 2000 note?

(a)   Rs 2000

(b)   Rs 100

(c)   Rs 200

(d)   Rs 300

(e)   None of these

Answer: (d)

Directions (Q. Nos. 57-61) These questions are based on the following five numbers.

834      427    563    649    975

57. If all the digits of numbers are arranged in ascending order within the number, then which of the following is lowest number?

(a)   975

(b)   649

(c)   834

(d)   563

(e)   427

Answer: (e)

58. If 1st digit of highest number is divided by 2nd digit of 2nd highest number, then what will be the resultant?

(a)   2

(b)   4

(c)   3

(d)   5

(e)   None of these

Answer: (c)

59. If 2 is subtracted from the every even digit and 1 is subtracted from every odd digit number of each number, which number among them will be lowest number?

(a)   834

(b)   427

(c)   563

(d)   649

(e)   975

Answer: (b)

60. What is addition of 3rd digit of highest number and 2nd digit of lowest number?

(a)   7

(b)   6

(c)   4

(d)   3

(e)   None of these

Answer: (a)

61. If 1st digit of 2nd highest number is divided by 1st digit of lowest number, then what will be the resultant?

(a)   5

(b)   3

(c)   2

(d)   4

(e)   None of these

Answer: (c)

62. If 1 is subtracted from each even number and 2 is subtracted to each odd number in the number 5827936, then how many digits will appear twice in the new number thus formed?

(a)   Only 7

(b)   Only 5 and 7

(c)   1, 5 and 7

(d)   4, 5 and 9

(e)   None of these

Answer: (c)

63. How many letter will be remain at the same position in the word ‘SURFACE’ when they are arranged in the alphabetical order from left to right?

(a)   Four

(b)   Three

(c)   One

(d)   Two

(e)   None of these

Answer: (c)

64. There are rive person i.e. P, Q, R, S and T. If Q is taller than R and S but smaller than T. S is smaller than T, who is not the tallest, then who is the tallest person among all?

(a)   T

(b)   Q

(c)   S

(d)   P

(e)   None of these

Answer: (d)

65. How many pairs of letters are there in the word ‘SENATOR’ which have as many letters between them in the word as in alphabetical series?

(a)   None

(b)   One

(c)   Two

(d)   Three

(e)   Four

Answer: (c)

Part III Quantitative Aptitude

66. The ratio of speed of boat in downstream and speed of stream is 9 : 1, if speed of current is 3 km/h, then find distance travelled upstream in 5 hours.

(a)   105 km

(b)   110 km

(c)   120 km

(d)   90 km

(e)   95 km

Answer: (a)

67. The sum of 4 consecutive even numbers are greater than three consecutive odd numbers by 81. If sum of least odd and even number is 59, then find the sum of largest odd and even numbers.

(a)   69

(b)   71

(c)   73

(d)   67

(e)   None of these

Answer: (e)

68. Two different amounts are invested in two schemes. In scheme A, amount X is invested at 8% per annum and in scheme B amount (X + 1400) is invested at 12% per annum. After 2 years difference between both interests is 880, then find value of X?

(a)   7200

(b)   5500

(c)   6800

(d)   7300

(e)   None of these

Answer: (c)

Directions (Q. Nos. 69-73) Read the table and answer the given questions.

69. What is the ratio of total number of books sold from store D on Sunday, Monday and Tuesday together to that sold from store E on Tuesday, Wednesday and Thursday together?

(a)   13 :11

(b)   11 : 15

(c)   11 : 13

(d)   13 : 15

(e)   11 : 17

Answer: (c)

70. Number of books sold on Friday from stores C and D increased by 15% and 20% respectively as compared to that sold on previous day. What number of books the stores C & D sold on Friday?

(a)   162

(b)   158

(c)   150

(d)   160

(e)   168

Answer: (d)

71. Total numbers of books sold from store B on Tuesday and Wednesday together are what percent more or less than total books sold from stores C & D together on Monday?

(a)   27 3/13%

(b)   27 3/11%

(c)   25 2/11%

(d)   24 3/11%

(e)   None of these

Answer: (b)

72. What is the average number of books sold from stores B on Sunday, C on Tuesday and E on Thursday?

(a)   82

(b)   80

(c)   88

(d)   75

(e)   85

Answer: (e)

73. Find the difference in total number of books sold from stores C and E together on Tuesday and From store B on Monday and Thursday together?

(a)   61

(b)   65

(c)   59

(d)   60

(e)   63

Answer: (a)

Directions (Q. Nos. 74-78) What should come in place of the question marks in following number series?

74. 255   230    250    235    245    ?

(a)   245

(b)   240

(c)   225

(d)   260

(e)   265

Answer: (b)

75. 5    3.5     5        ?        21.5   56.75

(a)   7.5

(b)   8.5

(c)   9.5

(d)   10.5

(e)   9

Answer: (c)

76. 8    4        4        8        32      ?

(a)   512

(b)   128

(c)   248

(d)   256

(e)   232

Answer: (d)

77. 129    128    124    115    ?        74

(a)   99

(b)   98

(c)   101

(d)   97

(e)   103

Answer: (a)

78. 0.5   1.5     5        18      76      ?

(a)   380

(b)   385

(c)   390

(d)   375

(e)   395

Answer: (b)

79. The average age of A and B, 2 years ago was 26. If age of A, 5 years hence is 40 years, and B is 5 year younger to C, then find difference between age of A and C?

(a)   8

(b)   10

(c)   9

(d)   12

(e)   None of these

Answer: (c)

80. The average of X, Y, Z is 24, Z is 24, X : Y = 2 : 3, X + Y = 60, then find X-Z = ?

(a)   16

(b)   14

(c)   8

(d)   10

(e)   12

Answer: (e)

81. The cost price of two articles is same, trade man got profit of 40% on first article, selling price of second article is 25% less than first article, then find overall profit percent?

(a)   23%

(b)   12 1/2%

(c)   25%

(d)   22 1/2%

(e)   None of these

Answer: (d)

82. The length of rectangle is 80% of diagonal of square of area 1225, then find area of rectangle, if its perimeter is 94√2.

(a)   1016

(b)   500

(c)   1604

(d)   1064

(e)   625

Answer: (d)

83. The annual salary of Arun is Rs 7.68 lakh. In a month if he spends Rs 12000 on his children, 1/3th of rest on food and Rs 8000 in mutual funds from his monthly salary, then find the monthly saving he is left with.

(a)   Rs 40000

(b)   Rs 45000

(c)   Rs 50000

(d)   Rs 36000

(e)   None of these

Answer: (a)

Directions (Q. Nos. 84-98) What should come in place of question marks in the following equations?

84. 40% of (20/4 × ?) = 48

(a)   20

(b)   24

(c)   28

(d)   32

(e)   22

Answer: (b)

85. (0.25 + 1.5) (3.5 + 1.5) = ?

(a)   15

(b)   30

(c)   20

(d)   24

(e)   20.5

Answer: (*)

86. 40 × 64 ÷ 80 = ?2 + 7

(a)   5

(b)   6

(c)   7

(d)   4

(e)   2

Answer: (a)

87. 25% of 16 × (15 – ?)3 = 256

(a)   15

(b)   13

(c)   9

(d)   11

(e)   7

Answer: (d)

88. 90 = 15% of 1000 × 3% of (100/?)

(a)   4

(b)   5

(c)   7

(d)   9

(e)   3

Answer: (b)

89. ? + 72 = (7 × 4) + (43 × 5)

(a)   192

(b)   194

(c)   196

(d)   198

(e)   200

Answer: (b)

90. 

(a)   2/3

(b)   2/9

(c)   4/9

(d)   9/4

(e)   7/9

Answer: (c)

91. 40% of (102 ÷ 17) × ? = 23 × 6

(a)   20

(b)   22

(c)   25

(d)   30

(e)   15

Answer: (a)

92. ? – 23 = (72 – 12) + (102 – 17)

(a)   132

(b)   126

(c)   124

(d)   128

(e)   130

Answer: (d)

93. 400% of ? = (13 × 15) – (891 ÷ 9)

(a)   20

(b)   24

(c)   22

(d)   28

(e)   18

Answer: (b)

94. 82 × 82 = 2? × 26 × 24

(a)   1

(b)   3

(c)   2

(d)   0

(e)   4

Answer: (c)

95. 20% of (22 × 32) – 2 = ?

(a)   2.5

(b)   5.2

(c)   5.4

(d)   5.6

(e)   6.2

Answer: (b)

96. 20% of 1/5 of 2250 = 50 + ?

(a)   55

(b)   90

(c)   50

(d)   40

(e)   45

Answer: (d)

97. 986 – 432 + 116 = ? + (13)2

(a)   501

(b)   505

(c)   401

(d)   451

(e)   None of these

Answer: (a)

98. 14.2% of 11000 + 15.6% of ? = 3590

(a)   12000

(b)   13000

(c)   14560

(d)   12250

(e)   13500

Answer: (b)

99. A can do a work in 24 days, B is 20% more efficient than A, if C can do the work in 10 more days than B, find days taken by A and C together to complete the work.

(a)   20/3 days

(b)   44/3 days

(c)   40/3 days

(d)   6 days

(e)   None of these

Answer: (c)

100. The ratio of milk to water is 5 : 4. If two litres of water is added, ratio becomes 10 : 9, then find new amount of water in the mixture.

(a)   14 L

(b)   16 L

(c)   18 L

(d)   20 L

(e)   22 L

Answer: (c)

IBPS CRP VII RRBs Office Assistant Preliminary (Phase I) Online Examination Held on September 23, 2017 Question Paper With Answer Key

IBPS CRP VII RRBs Office Assistant Preliminary (Phase I) Online Examination Held on September 23, 2017
IBPS CRP VII RRBs Office Assistant Preliminary (Phase I) Online Examination Held on September 23, 2017 Question Paper With Answer Key

IBPS CRP VII RRBs Office Assistant Preliminary (Phase I) Online Examination Held on September 23, 2017

Part I Reasoning

1. Point K is 7 m towards North of Point L. Point L is 13 m towards West of Point M. Point N is to the North of Point M and also to the East Point K. What is the distance between Point K and Point N?

(a)  7 m

(b)  13 m

(c)  19 m

(d)  20 m

(e)  12 m

Answer: (b)

Directions (Q. Nos. 2-6) In these questions, a group of numbers/symbol followed by five combinations of letters have been given. You have to find out which of the combinations correctly represents the group of number/symbol based on the following coding system and the conditions and mark that combination as your answer.

Conditions

(i) If the sums of all the digits in the group of elements is more than ten, the codes for the second and the fifth elements are to be interchanged.

(ii) If an odd number is immediately preceded by a symbol and immediately followed by an even number, then this odd number is to be coded as the code for the last element,

(iii) If there is no perfect square in the group of elements, the third element is to be coded as ‘©’.

(iv) If a symbol is a immediately preceded as well as immediately followed by an element, having a its letter code falling between A to M to in the English alphabetical order, then that symbol is to be coded as ‘B’.

Note

(a) If more than one condition applies, all are to be applied for coding.

(b) Counting is done from left to right.

2. #%54&β

(a)  UAQHKQ

(b)  UAEKHQ

(c)  UKEAHQ

(d)  UAQKHQ

(e)  UKEHAQ

Answer: (b)

3. %26#3⋆

(a)  AZ©SUF

(b)  AS©USF

(c)  AS©SFU

(d)  AZYUZF

(e)  AZ©USF

Answer: (e)

4. 2%$#7!

(a)  SA©RUM

(b)  DRS©UM

(c)  SRDUAM

(d)  SA©URM

(e)  SR©URM

Answer: (d)

5. 2@4%5#

(a)  SMKBHU

(b)  SMHBKU

(c)  SEKABU

(d)  SMKUBH

(e)  SMBAHU

Answer: (c)

6. 2β9⋆6@

(a)  SQLFQM

(b)  SYLQFM

(c)  SYLFQM

(d)  SYLFYM

(e)  SYLFMY

Answer: (c)

Directions (Q. Nos. 7-11) Study the following information to answer the given questions.

K, L, M, N, O, P and Q are seated in a straight line facing North, with equal distance between each other, but not necessarily in the same order.

Q is an immediate neighbour of the person sitting at the extreme end of the line. More than three people sit between Q and P.

N sits second to the left of O. O is neither an immediate neighbour of Q nor P.

As many people sit between Q and N as between K and L. K is not an immediate neighbour of N.

7. Based on the given arrangement, P is related to l following a certain logic. Following the same logic, L is related to M. To whom amongst the following is M related to following the same logic?

(a)  P

(b)  Q

(c)  K

(d)  N

(e)  O

Answer: (c)

8. Which of the following represents the correct position of N in the given arrangement?

(a)  Extreme left end of the line

(b)  Third to the left of L

(c)  Second to the right of Q

(d)  Exactly between K and M

(e)  Immediate right of P

Answer: (e)

9. Which of the following is true with respect to the given arrangement?

(a)  Five people sit between Q and P.

(b)  Only one person sits between O and P.

(c)  Only one person sits to the left of Q.

(d)  None of the given options is true.

(e)  There are as many persons between M and Q as between L and O.

Answer: (e)

10. Who amongst the following sits exactly between N and Q?

(a)  Both M and P

(b)  Both K and P

(c)  L, M and O

(d)  Only M

(e)  Both L and K

Answer: (c)

11. How many persons sit to the right of M?

(a)  None

(b)  Four

(c)  One

(d)  Two

(e)  Three

Answer: (d)

12. How many such pairs of letters are there in the word AUTOPSY, each of which has as many letters between them in the word as in the alphabetical series?

(a)  Two

(b)  None

(c)  Three

(d)  More than three

(e)  One

Answer: (a)

Directions (Q. Nos. 13-17) In these questions, relationship between different elements is shown in the statements. The statements are followed by conclusions. Study the conclusions based on the given statements and select the appropriate answer.

Give answer

(a) if only conclusion I is true

(b) if only conclusion II is true

(c) if either conclusion I or II is true

(d) if either conclusion I or II is true

(e) if neither conclusion I nor II is true

13. Statement S ≤ A > T = U ≤ R < N

Conclusions I. N > T       II. S > R

Answer: (a)

14. Statement P ≥ L ≥ O = U ≥ G ≥ H

Conclusions I. P < H        II. H < P

Answer: (d)

15. Statement L ≤ M = R > O < P

Conclusions I. L < P        II. L = P

Answer: (e)

16. Statement A ≤ B < E = G ≥ H > K

Conclusions I. A < G       II. E > K

Answer: (c)

17. Statement D < R ≥ A ≤ I = N ≤ S

Conclusions I. R ≤S        II. D > I

Answer: (e)

18. In a certain code language, ‘schools closed down’ is coded as ‘©#%’, ‘down the line’ is coded as ‘$#⋆’ and ‘schools and colleges’ is coded as ‘&f%’. How is ‘closed’ definitely coded in the given code language?

(a)  %

(b)  ©

(c)  #

(d)  $

(e)  ⋆

Answer: (b)

Directions (Q. Nos. 19-23) Study the given information to answer the given questions.

Seven people viz. A, B, C W, X, Y and Z live on seven different floors of a seven storeyed building, but not necessarily in the same order. The ground floor of the building is numbered one, the one above that is numbered two and so on till the topmost floor is numbered seven.

Only four people live between X and B. X lives on one of the floors above B. Only two people live between B and W. A lives immediately above C. More than two people live between Z and Y. Y lives below Z.

19. Who lives immediately below and immediately above Z respectively?

(a)  A, X

(b)  A, W

(c)  C, X

(d)  W, X

(e)  W, C

Answer: (d)

20. On which floor number does A live as per the given arrangement?

(a)  Seven

(b)  Five

(c)  Three

(d)  Six

(e)  Four

Answer: (e)

21. If C interchanges his floor with Z and B interchanges his floor with A, how many people will live between A and Z, as per the new arrangement thus formed?

(a)  Three

(b)  None

(c)  Two

(d)  One

(e)  Four

Answer: (b)

22. As per the given arrangement, X is related to Z following a certain pattern. Similarly, Z is related to W. To which of the following is A related to following the same pattern?

(a)  Y

(b)  B

(c)  X

(d)  Other than those given as options

(e)  C

Answer: (e)

23. Which of the following is true as per the given arrangement?

(a)  No one lives between X and C.

(b)  X lives on an even numbered floor.

(c)  None of the given options is true.

(d)  A lives on floor number three.

(e)  Non one lives below Y.

Answer: (e)

24. Among four people viz. J, K, L and M, each having a different number of books, K has more books than L and M. M has more books than J. Who has the highest number of books?

(a)  L

(b)  M

(c)  K

(d)  Cannot be determined

(e)  J

Answer: (c)

Directions (Q. Nos. 25-29) Study the following arrangement carefully and answer the questions.

7  9  4  1  3  2  8  6  9  5  7  4  8  2  1  3 7  4  2  6  8  3  5  1  6 7  4  9  2  1  5  3 2

(While answering some of the questions please note that 1 is also perfect square)

25. How many 2s are there in the given arrangement each of which immediately followed by a digit which is a multiple of 3?

(a)  Three

(b)  Two

(c)  None

(d)  One

(e)  More than three

Answer: (a)

26. What will be the resultant if the digit that is eighth from the left end is subtracted from t he digit that is sixth from the right end?

(a)  5

(b)  −2

(c)  6

(d)  3

(e)  8

Answer: (d)

27. How many such perfect square are there in the given arrangement which is immediately preceded by an odd number (each digit to be counted as many times as it appears)

(a)  Two

(b)  Six

(c)  Five

(d)  None

(e)  Four

Answer: (b)

28. If all the 5s are deleted, then which of the following will be fourth from the right end?

(a)  2

(b)  4

(c)  6

(d)  9

(e)  5

Answer: (a)

29. What is the sum of the digits that are the third, the sixth and the eleventh positions from the left end?

(a)  19

(b)  13

(c)  16

(d)  24

(e)  21

Answer: (b)

Directions (Q. Nos. 30-35) These questions are based on the five words given  below.

DEW TUB AID SKY POT

(The new words formed after performing the mentioned operations may or may not necessarily be meaningful English words)

30. If the letter ‘R’ is added before all the given words, how many words thus formed will be meaningful English words?

(a)  More than three

(b)  None

(c)  Three

(d)  Two

(e)  One

Answer: (e)

31. If second alphabet in each of the words is changed to the previous alphabet as per the English alphabetical series, how many words having more than one vowels (same or different vowels) will be formed?

(a)  Three

(b)  Two

(c)  One

(d)  Four

(e)  None

Answer: (e)

32. If each letter in each word is arranged in alphabetical order (within the word), how many words will remain unchanged as compared to the original set of words?

(a)  Four

(b)  Three

(c)  Two

(d)  None

(e)  One

Answer: (e)

33. How many letters are there in the English alphabetical series between the first letter of the word which his second from the left and the second letter of t he word, which his second from the right of the given words?

(a)  Eight

(b)  Five

(c)  Six

(d)  Three

(e)  None

Answer: (a)

34. If the given words are arranged in the order as they would appear in a dictionary from left to right, which of the following will be fourth from the left?

(a)  DEW

(b)  TUB

(c)  POT

(d)  SKY

(e)  AID

Answer: (d)

35. Four of the following five are alike in a certain way based on the English alphabetical series and hence form a group. Which one of the following does not belong to the group?

(a)  LJH

(b)  KJG

(c)  ECA

(d)  DBZ

(e)  CAY

Answer: (b)

Directions (Q. Nos. 36-40) In these questions, two/three statements followed by two conclusions numbered I and II have been given. You have to take the given statements to be true even if they seem to be at variance from commonly known facts and then decide which of the given conclusions logically follows from the given statements.

Give answer

(a) if only conclusion I follows  (b) if only conclusion II follows

(c) if both conclusions follow     (d) if either conclusion I or II follows

(e) if neither conclusion I nor II follows

36. Statements Some lawyer are judges.

All judges are magistrates.

Some magistrates are bosses.

Conclusions I. Some lawyers are definitely not bosses.

II. All bosses are judges.

Answer: (a)

37. Statements Some lawyers are judges.

All judges are magistrates.

Some magistrate are bosses.

Conclusions I. At least some lawyers are magistrates.

II. Some magistrates are definitely not bosses.

Answer: (c)

38. Statements All sweets are juices.

All juices are drinks.

Conclusions I. All sweets are drinks.

II. All drinks are juices.

Answer: (e)

39. Statements No bed is a chair.

Some chairs are tables.

Conclusions I. At least some beds are tables.

II. No bed is a table.

Answer: (b)

40. Statements All numbers are figures.

Some figures are data.

Conclusions I. Some numbers are data.

II. At least some figures are numbers.

Answer: (a)

Part II Numerical Ability

41. Carol’s total marks in Physics and Chemistry together was 173 (out of 200). If she scored 65 marks (out of 80) in Mathematics, what was her overall percent marks scored in all 3 subjects?

(a)  85%

(b)  90%

(c)  87%

(d)  84%

(e)  82%

Answer: (a)

42. Population of a village in 2001 was 4800. It increased by 40% from 2001 to 2002 and decreased by x% from 2002 to 2003. If the population of the village in 2003 was 5712, what is the value of x?

(a)  15

(b)  18

(c)  20

(d)  25

(e)  28

Answer: (a)

43. Working together, A and B complete a given task in 7½ days. Working alone, A and C take 12 days and 30 days respectively to complete the task, in how many days can B and C working together, complete the task?

(a)  12

(b)  14

(c)  18

(d)  16

(e)  20

Answer: (a)

44. Article A was sold at 20% profit and Article B was sold at 60% profit. The selling price of Article B was 50% more than that of Article A. Cost price of Article B is, what percent more than that of Article A?

(a)  7.5

(b)  12.5

(c)  10.5

(d)  11

(e)  15

Answer: (b)

45. Jar A contained a mixture of apple, orange and guava juice in the respective ratio of 4 : 5 : 7. 16 L of mixture was taken out. As a result, the difference between quantity of orange juice and guava juice in the mixture becomes 4 L. What was the initial quantity of apple juice in the mixture?

(a)  5 L

(b)  6 L

(c)  8 L

(d)  10 L

(e)  12 L

Answer: (c)

46. A, B and C start a small business by investing a total sum of Rs 15600 together. A invested Rs 600 less than B and C invested an amount, which was the sum of A’s and B’s investment, if at the end of the year total profit of Rs 10400 was received, what was year a total profit of Rs 10400 was received, what was A’s share?

(a)  Rs 2400

(b)  Rs 3600

(c)  Rs 2800

(d)  Rs 3000

(e)  Rs 3200

Answer: (a)

47. A total of Rs 800 was invested in 2 Schemes A and B for 2 years each. Schemes A and B offer compound interest at 10% p.a. and 20 p.a. respectively, both compounded annually. If the total interest earned from both schemes together was Rs 260, what was the sum invested in Scheme B?

(a)  Rs 200

(b)  Rs 400

(c)  Rs 600

(d)  Rs 100

(e)  Rs 50

Answer: (b)

Directions (Q. Nos. 48-52) What will come in place of questions marks in the given number series?

48. 240   96  48  32  32  ?

(a)  48

(b)  56

(c)  72

(d)  64

(e)  96

Answer: (d)

49. 4   7  16  43  124  ?

(a)  359

(b)  367

(c)  363

(d)  355

(e)  372

Answer: (b)

50. 104   103  99  90  74  ?

(a)  49

(b)  57

(c)  59

(d)  63

(e)  61

Answer: (a)

51. 17   25  37  52  69  ?

(a)  97

(b)  79

(c)  87

(d)  93

(e)  95

Answer: (c)

52. 242   116  54  24  10 ?

(a)  2

(b)  3

(c)  4

(d)  6

(e)  5

Answer: (c)

Directions (Q. Nos. 53-67) What will come in place of questions marks in the following questions?

53. ?% of 360 ÷ 20% of 70 = 9

(a)  30

(b)  40

(c)  45

(d)  55

(e)  35

Answer: (e)

54. 1.52 × 50 ÷ ? = 1.12

(a)  7.5

(b)  9.5

(c)  11.5

(d)  8.5

(e)  6.5

Answer: (b)

55. 

(a)  12.5

(b)  9.5

(c)  3.5

(d)  5.5

(e)  6.5

Answer: (e)

56. 11 × 49 ÷ ? = 283 – 206

(a)  21

(b)  7

(c)  2

(d)  14

(e)  5

Answer: (b)

57. 

(a)  3

(b)  5

(c)  4

(d)  9

(e)  7

Answer: (e)

58. 

(a)  5.55

(b)  55.5

(c)  50.5

(d)  5.05

(e)  50.05

Answer: (a)

59. ?% of 450 + 46% of 285 = 257.1

(a)  34

(b)  36

(c)  21

(d)  28

(e)  32

Answer: (d)

60. 

(a) 

(b) 

(c) 

(d) 

(e) 

Answer: (a)

61. 

(a)  450

(b)  410

(c)  458

(d)  350

(e)  358

Answer: (c)

62. 

(a)  24

(b)  35

(c)  29

(d)  31

(e)  32

Answer: (b)

63. 

(a)  17

(b)  12

(c)  8

(d)  9

(e)  7

Answer: (b)

64. (4444 ÷ 40) + (645 ÷ 25) + (3991 ÷ 26) = ?

(a)  280.4

(b)  290.4

(c)  295.4

(d)  285.4

(e)  292.4

Answer: (b)

65. [(144)2 ÷ 48 × ?] ÷ 22 = 216

(a)  23

(b)  16

(c)  11

(d)  32

(e)  24

Answer: (c)

66. 

(a)  252

(b)  312

(c)  238

(d)  280

(e)  240

Answer: (a)

67. (?)2 + (65)2 = (160)2 – (90)2 – 7191

(a)  75

(b)  77

(c)  49

(d)  81

(e)  78

Answer: (e)

68. The number of years by which Arushi is elder to Parul is 3 years less than that by which Arushi is younger than Uma. If 5 years ago, the sum of their ages was 93 years, what is Arushi present age?

(a)  21 yr

(b)  25 yr

(c)  36 yr

(d)  38 yr

(e)  30 yr

Answer: (c)

69. The diagonal of a rectangle measures  If its breadth is 20% less than its length, what is its perimeter?

(a)  27 cm

(b)  62 cm

(c)  18 cm

(d)  44 cm

(e)  54 cm

Answer: (c)

70. From his monthly income, Jatin spend 8% on paying electricity bill, 4% on cooking gas bill, 10% on paying mobile bill and Rs 400 on paying cable bill. He saved the remaining amount. If the amount saved was Rs 38600, what is Jatin’s monthly income?

(a)  Rs 60000

(b)  Rs 50000

(c)  Rs 45000

(d)  Rs 40000

(e)  Rs 55000

Answer: (b)

Directions (Q. Nos. 71-75) Study the table and answer the given questions.

71. Number of balls sold by Store R on Tuesday was, what percent more than those sold by Store U on Tuesday?

(a)  12.5%

(b)  25%

(c) 

(d)  26%

(e)  15%

Answer: (b)

72. What was the difference between the total number of balls sold by Store T on Tuesday and Wednesday together and the number of balls sold by Store U on Saturday?

(a)  6

(b)  12

(c)  10

(d)  8

(e)  13

Answer: (c)

73. Number of balls sold by Store V on Friday was 50% of the total number of balls sold by Store S and T together on Friday. How many balls were sold by Store V on Friday?

(a)  26

(b)  20

(c)  24

(d)  23

(e)  21

Answer: (e)

74. What was the average number of balls sold by store Q on Wednesday, Thursday and Friday?

(a)  17

(b)  21

(c)  19

(d)  14

(e)  16

Answer: (a)

75. What was the respective ratio between the total number of balls sold b y stores R and S together on Thursday and those sold by the same stores together on Saturday?

(a)  7 : 5

(b)  11 : 8

(c)  9 : 7

(d)  6 : 5

(e)  10 : 7

Answer: (e)

76. What time will be taken by a boat to cover a distance of 128 km along the stream, if speed of a boat in still water is 24 km/h and speed of stream is 8 km/h?

(a)  4 h

(b)  6 h

(c)  8 h

(d)  7 h

(e)  5 h

Answer: (a)

77. The square root of a number is divided by 6 and then 13 is added to it, the result obtained is 19. If 4 is added to the same number and then it is decreased by 10%, what would be the result obtained?

(a)  1120

(b)  1150

(c)  1170

(d)  1200

(e)  1240

Answer: (c)

78. The length of a rectangle is 20% more than its breadth. If the perimeter of the rectangle is 66 m, what is the perimeter of a square (in m) whose side is 5% more than the length of the rectangle?

(a)  72.4

(b)  75.6

(c)  72.2

(d)  74.8

(e)  70.6

Answer: (b)

79. Train A, 305 m long can cross an electrical pole in 18 seconds. The respective ratio of speed of trains A and B is 5 : 8. If the length of Train B is 188 m, how much time (in seconds) would it take to cross a 300 m long platform?

(a)  15

(b)  16

(c)  17

(d)  18

(e)  20

Answer: (d)

80. If the difference between the compound interest and the simple interest on a certain sum for 2 years at 8% per annum is Rs 32, then the sum is

(a)  Rs 5000

(b)  Rs 5250

(c)  Rs 5500

(d)  Rs 6000

(e)  Rs 6500

Answer: (a)

IBPS PO/MT Preliminary (Phase I) Online Examination Held on October 6, 2017 Question Paper With Answer Key

IBPS PO/MT Preliminary (Phase I) Online Examination Held on October 6, 2017
IBPS PO/MT Preliminary (Phase I) Online Examination Held on October 6, 2017 Question Paper With Answer Key

IBPS PO/MT Preliminary (Phase I) Online Examination Held on October 6, 2017

Directions (Q. Nos. 1-3) Study the given information carefully to answer the given questions.

Six books A, B, C, D, E and F each of different thickness, are kept on a table. C is thicker than A, but thinner than E. A is thicker than both B and D. E is not the thickest. The third thinnest book is 9 cm thick and the thickest book is 16 cm thick.

(Note the thickness of all the books are in whole numbers.)

1. If E is 5 cm thicker than A, then how thick is E?

(a)  11 cm

(b)  9 cm

(c)  12 cm

(d)  14 cm

(e)  Cannot be determined

Answer: (d)

2. With respect to the thickness of the given books, if C + F = 27, then A + C = ?

(a)  20

(b)  Other than those given as options

(c)  23

(d)  15

(e)  19

Answer: (a)

3. If B is 8 cm thick, then which of the following is true about B?

(a)  B is the third thinnest book of all.

(b)  F is 5 cm thicker than B.

(c)  B is thicker than D.

(d)  All the given statements are true.

(e)  Other than those given as options

Answer: (c)

Directions (Q. Nos. 4-6) Study the following information carefully to answer the questions based on it.

A is the m other of B. B is the sister of C. D is the son of C. E is the brother of D. F is the mother of E. G is the grand-daughter of A. H has only two children B and C.

4. How is F related to H?

(a)  Son-in-law

(b)  Daughter-in-law

(c)  Father-in-law

(d)  Grand-daughter

(e)  Niece

Answer: (b)

5. How is C related to E?

(a)  Father

(b)  Son

(c)  Mother

(d)  Cousin

(e)  Grandfather

Answer: (a)

6. Who is mother of G?

(a)  C

(b)  B

(c)  F

(d)  Either B or F

(e)  Either C or F

Answer: (c)

Directions (Q. Nos. 7-11) In the following questions relationships between different elements is shown in the statements. The statements are followed by conclusions. Study the conclusions based on the given statements and select the appropriate answer.

Give answer

(a) if only conclusion II is true

(b) if only conclusion I is true

(c) if both conclusion are true

(d) if either conclusion I or II is true

(e) if neither conclusions I nor II is true

7. Statements F ≤ U ≥ R; N ≤ U ≤ L

Conclusions I. L ≥ R        II. N ≥ F

Answer: (e)

8. Statements Q ≤ U = E ≥ N; R > A ≥ E; T ≤ O ≤ N

Conclusions I. T ≤ E        II. R > N

Answer: (c)

9. Statements R < A ≥ C ≥ E; A ≥ N ≥ T; C ≤ I ≤ L

Conclusions I. A ≤ L        II. I ≥ E

Answer: (a)

10. Statements Q ≤ U = E ≥ N; R > A ≥ E; T ≤ O ≤ N

Conclusions I. Q = A       II. A > Q

Answer: (e)

11. Statements R < A ≥ C ≥ E; A ≥ N ≥ T; C ≤ I ≤ L

Conclusions I. N > R       II. T ≤ E

Answer: (e)

Directions (Q. Nos. 12-17) Study the given information carefully to answer the given questions.

Seven athletes M, N, O, P, Q, R and S live on seven different floors of a building but not necessarily in the same order. The lowermost floor of the building is numbered 1, the one above that is numbered 2 and so on till the topmost floor is numbered 7. Each one of them runs for a different distance in marathon – 850 m, 1300 m, 2200 m, 2800 m, 3300 m, 4000 m, and 4700 m, but not necessarily in the same order. The one who runs for 2200 m lives on floor numbered 3. Only one person lives between O and the one who runs for 2200 m. The one who runs for 4000 m lives immediately above O. Only one persons lives between the one who runs for 4000 m and the one who runs for 1300 m. The number of people living between O and the one who runs for 1300 m is same as that between the one who runs for 4000 m and R. N lives on an odd numbered floor. N ran for 2000 m more than the one who lives on floor number 4. Only two people live between Q and the one who runs for 3300 m. The one who runs for 2800 m lives on one of the floors below Q but not on the floor number 2, only two people live between M and S. The one who runs for 850 m lives immediately below M.

12. How many people live between S and N?

(a)  Three

(b)  One

(c)  Five

(d)  Four

(e)  Two

Answer: (a)

13. Who amongst the following live(s) between P and the one who runs for 1300 m?

(a)  Both Q and R

(b)  Only S

(c)  Both R and the one who runs for 850 m

(d)  Only the one who runs for 4000 m

(e)  Both R and the one who runs for 2200 m

Answer: (b)

14. As per the given arrangement, four of the following five are alike in a certain way and so form a group. Which one of the following does not belong to the group?

(a)  Q-3300 m

(b)  O-1300 m

(c)  Floor number 4-S

(d)  Floor number 2-R

(e)  Floor number 7-1300 m

Answer: (c)

15. Which of the following statements is true with respect to the given arrangement?

(a)  Only two people live between P and O

(b)  Q runs for 4000 m.

(c)  N lives on floor number 7.

(d)  The one who runs for 850 m lives immediately above P

(e)  None of the above

Answer: (c)

16. If the total distance covered By in M is 5300 m, then how much did B run alone?

(a)  2000 m

(b)  4000 m

(c)  3100 m

(d)  1300 m

(e)  600 m

Answer: (e)

17. Who amongst the following runs for 2200 m?

(a)  P

(b)  N

(c)  Q

(d)  R

(e)  S

Answer: (e)

Directions (Q. Nos. 18-22) Study the following information to answer the given questions.

Eight friends G, H, I, J, K, L, M and N are seated in a straight line with equal distance between each other, but not necessarily in the same order. Some of them are facing North while some are facing South.

K is an immediate neighbour of the person sitting at an extreme end of the line. Only three people sit between K and M. J sits second to the right of M. J does not sit at an extreme end of the line. N sits to the immediate left of G. N is not an immediate neighbour of M. Immediate neighbours of G face opposite directions (i.e. if one neighbour faces North then the other faces South and vice-versa.) Persons sitting at the extreme ends face opposite directions (i.e., if one person faces North then the other faces South and vice-versa.) H sits second to the left of L. L faces North. L is not an immediate neighbour of K. Immediate neighbours of L face the same direction (i.e., if one neighbour faces North then the other also faces North and vice-versa.) Both K and H face a direction opposite to that of J(i.e., if J faces North then K and H faces South and vice-versa.)

18. As per the given arrangement, which of the following statements is true with respect to I?

(a)  I sits at an extreme end of the line.

(b)  K is an immediate neighbour of I.

(c)  Only four person sit between I and M.

(d)  I faces a direction to that of J.

(e)  All the given statements are true.

Answer: (c)

19. Which of the following pairs represents the immediate neighbours of J?

(a)  L, I

(b)  K, L

(c)  L, H

(d)  I, K

(e)  H, K

Answer: (c)

20. What is the position of K with respect to L?

(a)  Immediate right

(b)  Second to the right

(c)  Third to the left

(d)  Third to the right

(e)  Immediate left

Answer: (c)

21. Four of the given five are alike in a certain way based on the given arrangement and hence form a group. Which of them does not belong to that group?

(a)  JM

(b)  HL

(c)  HI

(d)  KL

(e)  MG

Answer: (c)

22. How many persons sit to the left of G?

(a)  One

(b)  Two

(c)  More than three

(d)  None

(e)  Three

Answer: (a)

Directions (Q. Nos. 23-27) Study the following information carefully and answer the given questions.

Eight different people Viz K, L, M, N, V, W, X and Y are sitting around a circular table facing the centre but not necessarily in the same order. Each one of them likes a different brand of clothes Viz, Nautica, Chemistry, Zara, Mango, Puma, Adidas, Zodiac and Park Avenue, but not necessarily in the same order. Only three people sit between X and the one who likes Zodiac. V sits second to the right of X. The one who like Nautica sits third to the left of Y. Y does not like Zodiac. The one who likes Zodiac is not a immediate neighbour of Y. Only three people sit between Y and the one who likes Chemistry. W does not like Chemistry. The one who likes Puma sits to the immediate left of K. K is not an immediate neighbour of V. Only two people sit between the ones who like Puma and Zara. M is one of the immediate neighbours of the one who likes Zara. The one who likes Park Avenue sits to the immediate right of L. Only three people sit between L and the one who likes Adidas.

23. Four of the following five are alike in a certain way based on the given arrangement and thus form a group. Which is the one that does not belong to that group?

(a)  M-Nautica

(b)  X-Adidas

(c)  V-Zodiac

(d)  Y-Puma

(e)  N-Park Avenue

Answer: (a)

24. Who amongst the following likes Mango?

(a)  W

(b)  K

(c)  V

(d)  Y

(e)  N

Answer: (d)

25. Which of the following represents the brand which W likes?

(a)  Adidas

(b)  Puma

(c)  Park Avenue

(d)  Zara

(e)  Nautica

Answer: (b)

26. Who amongst the following sits second to the left of the one likes Zodiac?

(a)  W

(b)  The one who likes Adidas

(c)  Y

(d)  M

(e)  X

Answer: (b)

27. Who amongst the following sit exactly between the ones who like Puma and Zara when counted from the right of the one who likes Zara?

(a)  The one who likes Adidas and Mango

(b)  N and the one who likes Park Avenue

(c)  X and Y

(d)  K and W

(e)  V and the one who like Chemistry

Answer: (a)

Directions (Q. Nos. 28-32) In each questions below, three/two statements followed by two conclusions numbered I and II have been given. You have to take the given statements to be true even if they seem to be at variance from commonly known facts and then decide which of the given conclusions logically follows from the given statements disregarding commonly known facts.

Give answer

(a) if either conclusion I or II follows

(b) if both conclusions follow

(c) if neither conclusion I nor II follows

(d) if only conclusion II follows

(e) if only conclusion I follows

28. Statements All packets are envelopes. No envelop is a gift. Some gifts are boxes.

Conclusions I. All envelops are packets.

II. All boxes can never be envelopes.

Answer: (d)

29. Statements All diaries are novels. All novels are biographies. Some biographies are scripts.

Conclusions I. At least some diaries are scripts.

II. No diary is a script.

Answer: (a)

30. Statements Some days are months. Some months are weeks.

Conclusions I. Some weeks are days.

II. No week is a day.

Answer: (a)

31. Statements All packets are envelopes. No envelop is a gift. Some gifts are boxes.

Conclusions I. All packets being boxes is possibility.

II. No packet is a gift.

Answer: (d)

32. Statements All diaries are novels. All novels are biographies. Some biographies are scripts.

Conclusions I. All diaries are biographies.

II. Some scripts are definitely not novels.

Answer: (d)

Directions (Q. Nos. 33-35) Study the given information carefully to answer the given questions.

Point E is 16 m to the South of Point C. Point F is 4 m to the West of Point E. Point H is 5 cm to the South of Point F. Point J is 12 m to the East of Point H. Point Y is to the East of Point F. A persons walks 15 m from Point Y towards West, reaches Point Z, takes a left turn and reaches Point J.

33. What is the difference of the distance between the Point Z, J and Points Y, F?

(a)  26 m

(b)  22 m

(c)  12 m

(d)  8 m

(e)  10 m

Answer: (b)

34. If a person walks 20 m towards North from Point Z, takes a left turn and walks 8 m, how far will be from Point C?

(a)  2 m

(b)  6 m

(c)  3 m

(d)  4 m

(e)  5 m

Answer: (d)

35. In which directions is Point Z with respect to Point H?

(a)  North-West

(b)  North-East

(c)  South-West

(d)  East

(e)  South-East

Answer: (b)

Part II Quantitative Aptitude

Directions (Q. Nos. 36-40) What approximate value should come in the place questions marks in the given questions.

36. 90.05 + 281 ÷ 4 – 151.06 = ∛?

(a)  27

(b)  343

(c)  216

(d)  729

(e)  285

Answer: (d)

37. 17.822 ÷ 4.05 × 90.11 ÷ 4.98 = ?

(a)  396

(b)  336

(c)  2420

(d)  1458

(e)  1270

Answer: (d)

38. 80.04% of 150.16 + 60.02% of 50.07 = ?

(a)  150

(b)  125

(c)  210

(d)  175

(e)  187

Answer: (a)

39. 

(a)  30

(b)  10

(c)  5

(d)  20

(e)  22

Answer: (a)

40. 

(a)  5

(b)  45

(c)  15

(d)  20

(e)  25

Answer: (a)

Directions (Q. Nos. 41-45) What should come in the place the questions marks in following number series?

41. 14  8  7  5  22  ?

(a)  54

(b)  64

(c)  62

(d)  58

(e)  56

Answer: (e)

42. 8  14  25  46  82   ?

(a)  132

(b)  130

(c)  138

(d)  144

(e)  148

Answer: (c)

43. 13   14  30  93  ?   1885

(a)  358

(b)  336

(c)  364

(d)  376

(e)  396

Answer: (d)

44. 65   70    63   74   61   ?

(a)  78

(b)  58

(c)  72

(d)  68

(e)  74

Answer: (a)

45. 9   11   16   33  98   ?

(a)  350

(b)  355

(c)  364

(d)  372

(e)  380

Answer: (b)

46. Five years ago, Somi’s age at that time was 1/3rd of Amit’s age at that time. The respective ratio between Amit’s age six years hence and Somi’s age twelve years hence, will be 7 : 4. What was Somi’s age three years ago?

(a)  13 yr

(b)  29 yr

(c)  17 yr

(d)  25 yr

(e)  16 yr

Answer: (c)

47. A bag contains 24 eggs out of which 8 are rotten. The remaining eggs are not rotten eggs. The two eggs are selected at random. What is the probability that one of the eggs is rotten?

(a)  11/23

(b)  17/23

(c)  13/23

(d)  19/23

(e)  21/23

Answer: (c)

48. A, B and C started a business with their investment in the ratio 1 : 3 : 5. After 4 months, A invested the same amount as before and B as well as C withdrew half of their investments. The ratio of their profits at the end of the year is

(a)  1 : 2 : 3

(b)  3 : 4 : 15

(c)  3 : 5 : 10

(d)  5 : 6 : 10

(e)  5 : 3 : 7

Answer: (d)

49. The HCF and LCM of two natural numbers are 12 and 72 respectively. What is the difference between the two numbers, if one of the numbers is 24?

(a)  12

(b)  18

(c)  21

(d)  24

(e)  27

Answer: (a)

50. The circumference of the semi-circle is 180 cm. If the side of a square is 60% more than the diameter of the circle, what is the perimeter of the square?

(a)  368 cm

(b)  464 cm

(c)  486 cm

(d)  448 cm

(e)  344 cm

Answer: (d)

51. Cost p rice of two beds are equal. One bed is sold at a profit of 30% and the other one for Rs 5504 less than the first one. If the overall profit earned after selling both the beds is 14%, what is the cost price of each bed?

(a)  Rs 17000

(b)  Rs 16800

(c)  Rs 17600

(d)  Rs 17800

(e)  Rs 17200

Answer: (b)

52. In Jar A, 180 L milk was mix with 36 L water. Some of this mixture was taken out from Jar A and put in Jar B. If after adding 6 L of water in the mixture, the respective ratio between milk and water in Jar B was 5 : 2 respectively, what was the amount of mixture that was taken out from Jar A?

(a)  24 L

(b)  54 L

(c)  30 L

(d)  36 L

(e)  42 L

Answer: (d)

Directions (Q. Nos. 53-57) Refer to the graph and answer the given questions.

53. In 2008, 30% of the bags sold by store M and 25% of the bags sold by store N were leather bags. What was the total number of leather bags sold by store M and N together in 2008?

(a)  163

(b)  155

(c)  145

(d)  149

(e)  158

Answer: (d)

54. What is the respective ratio between total number of bags sold by stores M and N together in 2009 and that in 2010?

(a)  12 : 17

(b)  11 : 14

(c)  11 : 12

(d)  12 : 15

(e)  15 : 17

Answer: (a)

55. If the average number of bags sold by store M in 2011, 2012 and 2013 was 350, what was the number of bags sold by the same store in 2013?

(a)  510

(b)  540

(c)  550

(d)  530

(e)  520

Answer: (d)

56. Number of bags sold by store N decreased by what percent from 2008-12?

(a)  18 3/4%

(b)  20 3/4%

(c)  14 1/4%

(d)  16 1/4%

(e)  15 1/4%

Answer: (a)

57. What is the difference between total number of bags sold by stores M and N together in 2009 and that in 2011?

(a)  110

(b)  130

(c)  100

(d)  90

(e)  120

Answer: (a)

58. The respective ratio between the monthly salary of Om and that of Pihu is 7 : 9. Om and Pihu, both save 20% and 40% out of their respective monthly salary. Om invest 1/2 of his savings in PPF and Pihu invests 7/9th of his savings in PPF. If Om and Pihu together saved Rs 17500 in PPF, what is Pihu’s monthly salary?

(a)  Rs 72000

(b)  Rs 36000

(c)  Rs 45000

(d)  Rs 40000

(e)  Rs 55000

Answer: (c)

Directions (Q. Nos. 59-63) In these questions, two equations numbered I and II have been given. You have to solve both the equations and choose the correct option.

Give answer

(a) if x > y     (b) if x ≥ y

(c) if x < y     (d) if x ≤ y

(e) if x = y or relationship between x and y cannot be established.

59. I. 4x2 – 15x + 14 = 0 II. 6y2 – 10y + 4 = 0

Answer: (a)

60. I. 3x2 + 10x + 3 = 0 II. 2y2 – 15y + 27 = 0

Answer: (b)

61. I. 7x2 + 12x + 5 = 0 II. 3y2 + 7y + 2 = 0

Answer: (e)

62. I. 16x2 – 14x + 3 = 0 II. 6y2 – 19y + 15 = 0

Answer: (c)

63. I. x2 + 11x + 18 = 0 II. y2 – √81 = 0

Answer: (d)

64. Ravi invested Rs P in a scheme A offering simple interest at 10% per annum for two years. He invested the whole amount he received from scheme A, in another scheme B offering simple interest at 12% per annum for five years. If the difference between the interests earned from schemes A and B was Rs 1300, what is the value of P?

(a)  Rs 2500

(b)  Rs 2000

(c)  Rs 3000

(d)  Rs 3500

(e)  Rs 4000

Answer: (a)

Directions (Q. Nos. 65-69) Study the table and answer the given questions.

65. Total number of of students studying in stream C, in St. Christ and PD together are what percent less than those studying in stream B in the same universities together?

(a)  20%

(b)  12.5%

(c)  18%

(d)  15.25%

(e)  16%

Answer: (b)

66. Number of males studying in stream B is what percent of that studying in stream C in St. Christ?

(a)  42 6/7%

(b)  40 1/9%

(c)  48%

(d)  54 1/6%

(e)  171 3/7%

Answer: (e)

67. What is the respective ratio between the total number of females studying in streams A and B together in PD university and the total number of females studying in the same streams together in Kelly?

(a)  5 : 6

(b)  10 : 13

(c)  1 : 2

(d)  2 : 3

(e)  4 : 5

Answer: (a)

68. Total number of males studying in stream D in all the universities together in 2013, were 1190 more than that in the year 2012. In 2013, what was the total number of students (Male + Female) studying in stream D in all the universities together, if the total number of male students in stream D in 2013, constituted 4/7 of the total number of students?

(a)  2000

(b)  2500

(c)  3000

(d)  3500

(e)  3800

Answer: (d)

69. What is the average number of male students studying in stream A in all the given universities?

(a)  1100

(b)  1250

(c)  1150

(d)  1200

(e)  1280

Answer: (a)

70. The distance between two cities (M and N) is 569 km. A train starts from city M at 8 am and travel towards city N @ 53 km per hour. Another train starts from city N at 9 am and travel towards city M @ 76 km per hour. At what time will the trains meet?

(a)  12 : 30 pm

(b)  1 : 00 pm

(c)  2 : 30 pm

(d)  1 : 30 pm

(e)  2 : 00 pm

Answer: (b)

Part III English Language

Directions (Q. Nos. 71-77) Read the following passage and answer the given questions.

Politics is local, but most problems are international. That is the fundamental problem for national governments caught between the twin forces of globalization and voter’s anger. The European refugee crisis, for example, seem to cry out for a continent-wide solution. But the tide of migrants has been vast and national governments have been tempted to put up barriers first, and answer questions later. The latest example saw Sweden introduce checks on those travelling from Denmark, leading the turn country, in turn, to impose temporary controls on its southern border with Germany. Anti-immigration parties have been gaining in the polls, with the exception of the German Chancellor; mainstream politicians want to head off the challenge. In a way, this looks like the same mismatch that has plagued the euro, a single currency without a unitary fiscal and political authority.

Many economists have advocated much greater integration of the euro zone in the wake of the bloc’s crisis. The European banking system would be stronger if there was a comprehensive deposit-insurance scheme, the economy would be more balanced if there were fiscal transfer from rick to poor countries. But such plans are unpopular with voters in rich countries (who perceive them as handouts) and in poor countries (who worry about the implied loss of local control that reforms would require). All that the European Union’s (EU) leaders have managed so far is to cobble together solutions (such as the Greek bailouts) at the last minute. Gone is the pledge of unity of the G20’s summit in London in 2009, when leaders agreed on a coordinated stimulus in response to the financial crisis. Central Banks are now heading in different directions, the Federal Reserve Bank has just tightened monetary policy while the European Central Bank and the Bank of Japan are committed to easing. Trade creates tighter links between countries, but global trade growth has been sluggish in recent years. The OECD thinks that trade grew by only 2% in volume in 2015. No longer is trade rising faster than Global GDP, as it was before the crisis. International agreements require compromise, which leaves politicians vulnerable to criticism from inflexible components. Voters are already dissatisfied with their lot after years of sluggish gains (or declines) in living standards. When populist politicians suggest that voter’s woes are all the fault of foreigners, they find a ready audience. Furthermore, economic woes can lead to much more aggressive foreign policy. In the developed world, demographic constrains (a static or shrinking workforce) may limit the scope for th kind of rapid growth needed to reduce the debt burden and make voters happier. Boosting that sluggish growth rate through domestic reforms (breaking up producer cartels, making labour markets more flexible) is very hard because such reforms arouse strong opposition from those affected. The danger is that a vicious cycle sets in. Global problems are not tackled because governments fail to cooperate, voters get angrier and push their leaders into more nationalistic positions and conflict which poses a threat to all.

71. What can be concluded from the example of the Greek bailout cited in the passage?

(a)  There is tremendous political turmoil in Greece.

(b)  The approach to the Greek financial crisis by Euro zone was not appropriate.

(c)  Greece was recovered from the financial crisis.

(d)  A comprehensive system of deposit insurance need not to be effected.

(e)  Greece is on the verge of another financial bailout.

Answer: (d)

72. Which of the following is the central idea of the passage?

(a)  A unified approach to regional issues is unwanted and impractical.

(b)  Globalization is on the decline which will reduce social unrest.

(c)  Unlike America and Asia, Europe is in severe financial difficulty.

(d)  International cooperation is declining which is dangerous.

(e)  Restoring faith in developed economies will take a long time.

Answer: (d)

73. Which of the following has/have been the outcomes(S) of economic woes?

(A) Uncompromising or antagonistic foreign policy.

(B) An all-powerful single financial regulator for Europe.

(C) Drop in trade volumes.

(a)  Only A

(b)  Only B

(c)  A and C

(d)  B and C

(e)  None of these

Answer: (c)

74. Which of the following is true in the context of the passage?

(a)  It is difficult for developed countries to achieve a high growth rate at present.

(b)  Europe needs greater economic integration.

(c)  Politicians need to take the right steps rather than popular ones.

(d)  Anti-globalization sentiment is quite high.

(e)  All of the above

Answer: (e)

75. Which of the following can be said about the G20 summit in London in 2009?

(a)  It was organized to addresses the fallout of the financial crisis.

(b)  Countries did not follow-up with a harmonized approach to the crisis.

(c)  Sentiments of unity were expressed at the summit.

(d)  It was unsuccessful as assurances did not translate into action.

(e)  All of the given options can be said.

Answer: (a)

76. Which of the following best explains the phrase ‘The danger is that a vicious cycle sets in’ in the context of the passage?

(a)  Failure to sacrifice individual interests for common goods perpetuates global problems.

(b)  With rise in income, consumption is boosted and so is debt.

(c)  Having common reforms take away a country’s autonomy.

(d)  Boosting trade with OECD countries makes economies vulnerable to oil price fluctuations.

(e)  A shrinking workforce in developed and developing countries worsens poverty.

Answer: (a)

77. Which of the following is the author’s view of the refugee crisis?

(a)  It is unmanageable problem controlling Europe and Asia.

(b)  To stem migration, rick countries need to safeguard their orders.

(c)  Politicians have responded appropriately.

(d)  A joint approach is required to resolve the crisis.

(e)  None of the above

Answer: (d)

Directions (Q. Nos. 78-82) Rearrange the given six sentences/group of sentences A, B, C, D, E and F in a proper sequence so as to form a meaningful paragraph and then answer the given questions.

(A) It also launched a policy to ban private cars one work-day a week based on the last digit of the number plate and has put restrictions on the number of vehicles from outside the city and raised parking fees in urban areas.

(B) Beijing’s annual bill for traffic congestion amounts to 70 billion Yuan ($11.3 billion), a recent study has found.

(C) However, such measures have done little in reducing congestion.

(D) The study further states that 80% of the total loss related to time wasted waiting, 10% to gas and 10% to environmental damage.

(E) In 2011, it introduced a lottery system to rein in the number of vehicles people buy.

(F) As a result, owing to these losses, the city started tacking the problem years ago.

78. Which of the following should be SECOND sentence after the rearrangement

(a)  A

(b)  B

(c)  F

(d)  D

(e)  E

Answer: (d)

79. Which of the following should be FOURTH sentence after the rearrangement

(a)  A

(b)  B

(c)  C

(d)  F

(e)  E

Answer: (e)

80. Which of the following should be SIXTH(LAST) sentence after the rearrangement

(a)  C

(b)  D

(c)  A

(d)  B

(e)  F

Answer: (a)

81. Which of the following should be FIRST sentence after the rearrangement

(a)  A

(b)  C

(c)  B

(d)  D

(e)  F

Answer: (c)

82. Which of the following should be FIFTH sentence after the rearrangement

(a)  E

(b)  D

(c)  A

(d)  F

(e)  C

Answer: (c)

Directions (Q. Nos. 83-92) Read these sentences to find out whether there is any grammatical mistake/error in them. The error, if any, will be in are part of the sentence. Mark the part with the error in your answer. If there is no error mark ‘No error’ as your answer. (Ignore the error of punctuations if any.)

83. In a short span of time, this startup/website has entrenched itself as the/go-to abode for cloth that are/well-designed with care and attention.

(a)  In a short span of time, this startup

(b)  website has entrenched itself as the

(c)  go-to abode for cloth that are

(d)  well-designed with care and attention

(e)  No error

Answer: (c)

84. The batter-operated scooter/equipped on Lithium-ion batteries that/provide it with a longer life/as compared to the conventional ones.

(a)  The battery-operated scooter

(b)  equipped on Lithium-ion batteries that

(c)  provide it with a longer life

(d)  as compared to the conventional ones

(e)  No error

Answer: (b)

85. Despite being such a small/country, Japan have been gone on to conquer/South-East Asia almost entirely/during the mid-twentieth century.

(a)  Despite being such a small

(b)  Country, Japan have been gone on to conquer

(c)  South-East Asia almost entirely

(d)  during the mid-twentieth century

(e)  No error

Answer: (b)

86. Scientists are increasingly concerned/about the potential long-term effects/ global warming on our/ natural environment and on the planet.

(a)  Scientists are increasingly concerned

(b)  about the potential long-term effects

(c)  global warming on our

(d)  natural environment and on the planet

(e)  No error

Answer: (e)

87. The most powerful advantage/ of the internet is that it/ decentralizes work centres and /therefore makes widespread empowerment.

(a)  The most powerful advantage

(b)  of the internet is that it

(c)  decentralizes work centres and

(d)  therefore makes widespread empowerment

(e)  No error

Answer: (b)

88. The youngster has prove his ability as/ an aggressive sportsperson and age being/ on his sides, he has a huge/ chance of succeeding in the near future.

(a)  The youngster has prove his ability as

(b)  an aggressive sportsperson and age being

(c)  on his sides, he has a huge

(d)  chance of succeeding in the near future

(e)  No error

Answer: (d)

89. Every year in summer, many/tourists visit to Kumartull in North/Kolkata to watch the artisans/ prepare the idols of Goddess Durga.

(a)  Every year in summer, many

(b)  tourists visit to Kumartull in North

(c)  Kolkata to watch the artisans

(d)  prepare the idols Goddess Durga.

(e)  No error

Answer: (b)

90. A partnership has been signed by / an Indian pharmaceutical company with/ a one from Japan in order to/ develop a vaccine for the chikungunya virus,

(a)  A partnership has been signed by

(b)  an Indian pharmaceutical company with

(c)  a one from Japan in order to

(d)  develop a vaccine for the chikungunya virus

(e)  No error

Answer: (c)

91. The new government has repealed/ the policy of free speech in/ the country, which has invited/ nationwide criticism from all and sundry.

(a)  The new government has repealed

(b)  the policy of free speech in

(c)  the country, which has invited

(d)  nationwide criticism from all and sundry

(e)  No error

Answer: (e)

92. Nothing can be built in this area since/ its soil is saline and contains minerals that/ would eat into any concrete structure/ that coming into contact with this soil.

(a)  Nothing can be built in this area since

(b)  its soil is saline and contains minerals that

(c)  would eat into any concrete structure

(d)  that coming into contact with this soil

(e)  No error

Answer: (d)

Directions (Q. Nos. 93-100) In the given passage, there are blanks, each of which has been numbered. Against each, five words are suggested, one of which fits the blank appropriately. Find the appropriate word in each case.

According to a new study from an international consortium, (93) up technology in the classroom doesn’t always lead to better education for children. The report from the OECD tracked educational outcome among students based on their use of technology at home and in the classroom. (94) student performance improved when they use technology in moderation, the group found, (95) to computers and the internet caused educational outcomes to drop.

The report further stated that (96) considerable investments in computers, internet connections and software for educational use, there is little solid evidence that greater computer use among students leads to  better scores in Mathematics and reading. Report results are based on an assessment in 2015 that tracked students in more than 40 countries and surveyed them on computer habits and conducted both written and digital tests. On average, 7 out of 10 students in countries surveyed, use computers at school and students average at least 25 minutes a day online. In some countries, like Turkey and Mexico, about half of the students do not have (97) to a computer at home. The survey found that students with more exposure to computers do better, on average, than those with little exposure to computers but the OECD (98) against drawing conclusions based on that result. The data could simply (99) that school systems that invest in technology also invest in better teachers and draw on students from a higher socio-economics class, who (100) to perform better in school.

93.

(a)  dispersing

(b)  building

(c)  installing

(d)  giving

(e)  amplifying

Answer: (b)

94.

(a)  Though

(b)  Even

(c)  While

(d)  Admitting

(e)  Through

Answer: (a)

95.

(a)  overdoing

(b)  exaggerating

(c)  working

(d)  exhausting

(e)  overexposure

Answer: (e)

96.

(a)  despite

(b)  withdrawal

(c)  by

(d)  though

(e)  since

Answer: (a)

97.

(a)  access

(b)  approach

(c)  availability

(d)  possibility

(e)  probability

Answer: (a)

98.

(a)  wake

(b)  alerted

(c)  acted

(d)  behaved

(e)  symptomise

Answer: (c)

99.

(a)  reverse

(b)  highlights

(c)  disregard

(d)  reflect

(e)  cast

Answer: (d)

100.

(a)  inclined

(b)  tend

(c)  bound

(d)  contribute

(e)  bear

Answer: (b)

IBPS RRBS CWE-IV Officers Grade-B Recruitment Preliminary Examination-2015 Held on September 13, 2015 Question Paper With Answer Key

IBPS RRBS CWE-IV Officers Grade-B Recruitment Preliminary Examination-2015 Held on September 13, 2015
IBPS RRBS CWE-IV Officers Grade-B Recruitment Preliminary Examination-2015 Held on September 13, 2015 Question Paper With Answer Key

IBPS RRBS CWE-IV Officers Grade-B Recruitment Preliminary Examination-2015 Held on September 13, 2015

Part 1 Reasoning

 

Directions (Q. Nos. 1-5) Study the given information carefully to answer the given questions.

In a certain code language,

‘challenges for rural education’ is written as ‘vx pr bt ze’

‘find measures for problems’ is written as ‘ws dl ze ho’

‘experts find challenges difficult’ is written as ‘bt ka mu dl’

‘education difficult in villages’ is written as ‘xq cg pr ka’

(All codes are two-letter codes only)

1. If ‘experts find solutions’ is written ad ‘dl ny mu’ in the given code language, hen what will ‘rural solutions difficult’ be coded as?

(a)   ny vs ka

(b)   ny pr dl

(c)   ka mu ny

(d)   Other than those given as options

(e)   vx ka xq

Answer: (d)

2. What is the code for ‘villages’ in the given code language?

(a)   ws

(b)   mu

(c)   Either ‘xq’ or ‘cg’

(d)   Other than those given as options

(e)   Either ‘pr’ or ‘ze’

Answer: (c)

3. What will be the code for ‘find education’ in the given code language?

(a)   pr dl

(b)   ka pr

(c)   ze ka

(d)   dl ws

(e)   Other than those given as options

Answer: (a)

4. What is the code for ‘challenges’ in the given code language?

(a)   cr

(b)   Other than those given as options

(c)   bt

(d)   ze

(e)   ka

Answer: (c)

5. In the given code language, what does the code ‘ho’ stand for?

(a)   Either ‘measures’ or ‘problems’

(b)   Either ‘for’ or ‘experts’

(c)   rural

(d)   find

(e)   difficult

Answer: (a)

6. Read the following information and answer the given question.

In spite of conducting several awareness programmes and receiving monetary help from the government, the women self-help groups are not very successful in rural areas of State B.

Which of the following statements cannot be a ‘reason’ for the given problem?

(a)   Mobility is highly limited for women due to social barriers in rural areas.

(b)   Women self-help groups are  unable to complete the orders on time due to manpower shortage.

(c)   Women in rural areas of State B hesitate in starting their own ventures due to lack of practical.

(d)   Only 55% of the women in rural areas of State B have matriculation certificate.

(e)   Self-help groups in rural areas of State B lack an organized structure to pump in enough money for advertising their products.

Answer: (d)

Directions (Q. Nos. 7-12) Study the following information and answer the questions.

S, T, U, V, W, X, Y and Z live on eight different floors of a building but not necessarily in the same order. The lowermost floor of the building is numbered one, the one above that is numbered two and so on till the topmost floor is numbered eight. Each of them also works at a different bank  namely, IDBI, SBI, HDFC, BOI, PNB, TJSB, Axis Bank and SVC, but not necessarily in the same order.

Z lives on an even numbered floor. Only three people live between Z and the one who works at BOI.

W lives immediately below the one who works at BOI.

Only three people live between W and the one who works at Axis Bank.

V lives immediately above T. V lives on an odd numbered floor. T does not work at BOI.

Only two people live between T and the one who works at SBI. The one who works at SBI does not live on the lowermost floor.

The one who works at SVC lives immediately above the one who works at PNB. The one who works at SVC lives on an even numbered floor but not on floor numbered two.

Only one  person lives between the one who works at SVC and the one who works at IDBI.

X lives immediately above S. X lives on an even numbered floor. X does not work at TJSB.

U does not work at PNB and does not live on floor numbered four.

7. Four of the following five are alike in a certain way based on the given arrangement and hence form a group. Which of the following does not belong to that group?

(a)   S – Floor numbered two

(b)   T – Floor numbered eight

(c)   W – Floor numbered six

(d)   Z – Floor numbered one

(e)   U – Floor numbered seven

Answer: (b)

8. V works at which of the following banks?

(a)   Axis  Bank

(b)   SVC

(c)   IDBI

(d)   SBI

(e)   Other than those given as options

Answer: (a)

9. Which of the following pair represent those who live immediately above and immediately below Z?

(a)   S, T

(b)   S, V

(c)   Other than those given as options

(d)   T, V

(e)   X, Y

Answer: (b)

10. W works at which of the following banks?

(a)   Other than those given as options

(b)   PNB

(c)   Axis Bank

(d)   BOI

(e)   SBI

Answer: (a)

11. U lives on which of the following floor numbers?

(a)   One

(b)   Two

(c)   Other than those given as options

(d)   Five

(e)   Seven

Answer: (b)

12. As per the given arrangement, U is related to PNB and S is related to HDFC in a certain way. To which of the following is V related to in the same way?

(a)   SVC

(b)   SBI

(c)   IDBI

(d)   TJSB

(e)   Axis Bank

Answer: (c)

Directions (Q. Nos. 13-15) In these questions, relationship between different elements is shown in the statements. These statements are followed by two conclusions.

Given answer

(a) if only conclusion I follows

(b) if only conclusion II follows

(c) if  both conclusions follow

(d) if none follows

(e) if either I or II follows

13. Statements

S ≤ P ≤  U ≥ N; U > B; L ≤ S

Conclusions

I. B > P II. L ≤ U

Answer: (b)

14. Statements

A > G ≥ O ≥ N < Y; O ≥ S ≥ R

Conclusions

I. R < A II. Y > S

Answer: (a)

15. Statements

M > O ≥ C ≥ K = E ≤ D; J ≥ C; O < Z

Conclusions

I. J ≥ E II. K < Z

Answer: (b)

Directions (Q. Nos. 16-19) In these questions, three statements followed by two conclusions I and II are given. Take the given statements to be true and then decide which of the given conclusion logically follows.

Give answer

(a) if only conclusion I follows

(b) if only conclusion II follows

(c) if either conclusion I or II follows

(d) if neither conclusion I nor II follows

(e) if both conclusions follow

16. Statements

Some letters are digits.

All digits are numbers.

All symbols are numbers.

Conclusions

(I) At least some letters are numbers.

(II) All symbols being digit is a possibility.

Answer: (e)

17. Statements

No point is a spot.

All spots are marks.

No mark is a dot.

Conclusions

(I) All marks being points is a possibility

(II) No spot is a dot.

Answer: (b)

18. Statements

Some letters are digits.

All digits are numbers.

All symbols are numbers.

Conclusions

(I) No letter is a symbol.

(II) All letters being digits is a possibility.

Answer: (d)

19. Statements

No point is a spot.

All spots are marks.

No mark is a dot.

Conclusions

(I) All dots being points is a possibility.

(II) At least some points are marks.

Answer: (a)

20. Read the following information carefully and answer the question which follows.

The farmers of village D have been growing crop Y for many years as there is hardly any rainfall in the area and crop Y has minimum water requirements. However, this year many farmers of village D have sown crop X instead of crop Y despite the fact that crop Y yielded a good produce last year.

Which of the following statements, if considered true, may not be a reason for the action of the farmers of village D?

(a)   In order to reduce the import of crop X, the government of the State has declared certain incentives for farmers cultivating crop X this year.

(b)   Crop X requires only a specific kind of fertilizer, which is not easily available as it is manufactured only in village S, whereas any fertilizer with high nitrogen component is suitable for crop Y.

(c)   The demand for crop X has risen considerably over the past one year as a result farmers growing crop X in the neighbouring villages of village D have made much more profit than expected.

(d)   The government had introduced a subsidy this year for good quality seeds of crop X whereas seeds available in the market for crop Y were of substandard quality.

(e)   A research committee, set-up this year by the government has recommended that the soil and climate conditions of village D are the  best for crop X.

Answer: (b)

21. Study the given information carefully to answer the given question.

For the past three years, the government of State X  has  been organizing awareness programmes for the women. Hoardings and banners have been put across the State promoting women upliftment and gender equality.

Which of the following cannot be consequence of the steps taken by the government of State?

(a)   The rate of female foeticide in the State has came down from 280 cases per year to 30 cases per year.

(b)   Dowry and domestic violence cases in the State dropped by 85% in the last couple of years.

(c)   Cases reported by women of the State regarding chain snatching on the roads have come down by 40% in the last two years.

(d)   Two years ago minimum 15% fee exemption was mad mandatory for girl child in all public and private schools of the State.

(e)   In all the public organizations of the State, a minimum of 10% female employees has been made compulsory since the past two and a half years.

Answer: (c)

22. Read the following information carefully and answer the question which follows.

Farmers should use hydroponics for farming. Hydroponics is growing plants in liquid nutrient solution. Since it does not require soil, farmers can save a large amount of time by avoiding tilling and de-weeding of their fields.

Which of the following statements does not weaken the given statement/

(a)   Initial set-up cost for hydroponics is very high as the necessary equipment and inputs are three times more expensive than that used in traditional methods.

(b)   Disease causing microorganisms that are water-based can enter and subsist much  more easily  in hydroponics based  environment as compared to traditional methods.

(c)   Rather than using only hydroponics or only traditional farming, most of the farmers use both simultaneously.

(d)   Without soil to serve as a buffer, if the hydroponics system fails, plant death occurs very rapidly leading to loss of 70-85% crop.

(e)   The  production using hydroponics is very limited and is more efficient for house gardens rather than large scale farming.

Answer: (c)

23. Which of the following definitely true if the statement given below is considered to be true?

(You have to take the given statements to be true even if they seem to be at variance from commonly known facts and then decide which of the given conclusions logically follows from the given statements disregarding commonly known facts.)

All farms are houses. All houses are jungles. No jungle is a ranch. All ranches are estates.

(a)   No jungle is an estate.

(b)   No jungle is a farm.

(c)   All estates being houses is a possibility.

(d)   No farm is a ranch.

(e)   All ranches being houses is a possibility.

Answer: (d)

Directions (Q. Nos. 24-28) Study the given information carefully to answer the given questions.

Eight people E, F, G, H, W, X, Y and Z are sitting in two parallel rows containing four people each E, F, G and H are sitting in row-1 facing North and X, X, Y and Z are sitting in row-2 facing South. (but not necessarily in the same order.) Thus, each person sitting in row-1 faces another person sitting in row-2. Each of the two rows consists of one Professor, one Leader, one Technician and one Doctor (but not necessarily in the same order.)

The Doctor of row-1 sits second to the right of H. E is an immediate neighbout of H. E faces the Leader of row-2.

X sits to the immediate right of the Leader. X faces one of the immediate neighbours of the Professor of row-1. The Professor of row-1 does not sit at any of the extreme ends of the line.

W sits second to the left of Y. Z does not face G. F faces the Professor of row-2. In both the rows, only one person sits  between the Professor and the Technician. W is not a Doctor.

24. Which of the following represent both the immediate neighbours of W?

(a)   X and Leader of row-2

(b)   Y and Technician of row-2

(c)   Y and Doctor of row-2

(d)   X and Professor of row-2

(e)   Z and Doctor of row-2

Answer: (d)

25. Who amongst the following sits to the immediate left of the Leader of row-1?

(a)   Professor of row-1

(b)   F

(c)   H

(d)   G

(e)   Doctor of row-1

Answer: (d)

26. Which of the given statements is true with respect to the given arrangement?

(a)   None of the given statements is true.

(b)   Y sits to the immediate right of Z.

(c)   H is a Teacher.

(d)   Technician of one row faces the Doctor of another row.

(e)   Y and H face each other.

Answer: (d)

27. If E and X interchange their places and so do G and Z, then who amongst the following will face W?

(a)   Z

(b)   Other than those given as options

(c)   A

(d)   X

(e)   B

Answer: (d)

28. Which of the following represent the people sitting at extreme ends of both the lines?

(a)   F, H and Z, W

(b)   F, H and X, Y

(c)   E, H and Y, Z

(d)   G, F and Z, Y

(e)   G, F and W, Y

Answer: (d)

29. Which of the give expression is definitely true if the expression Q > U < I ≤ E = T ≤ S’ is definitely

(a)   Q ≥ T

(b)   U < T

(c)   E > S

(d)   E ≥S

(e)   I > S

Answer: (b)

30. Which of the following symbols should be placed in the blank spaces respectively (in the same order from left to right) in order to complete the given expression in such a manner that both ‘A > I’ as well as ‘V ≥ T’ definitely hold true?

A_V_I_C_T

(a)   >, <, ≥, ≥

(b)   <, ≥, = <

(c)   ≥, <, >, =

(d)   >, =, ≥, =

(e)   Other than those given as options

Answer: (d)

31. In a class of 42 students, Mahesh’s rank is 16th from the bottom. What is his rank from the top?

(a)   25th

(b)   26th

(c)   24th

(d)   27th

(e)   29th

Answer: (d)

32. A man travels 4 km due North, then travels 6 km due East and further travels 4 km due North. How far he is from the starting point?

(a)   6 km

(b)   14 km

(c)   8 km

(d)   10 km

(e)   Other than those given as options

Answer: (d)

Directions (Q. Nos. 33-37) Study the given information carefully and answer the given questions.

Eight friends P, Q, R, S, W, X, Y and Z are sitting around a circular table but not necessarily in the same order. Some of them are facing the centre and some of them are facing outside (i.e. in a direction opposite to the centre)

R sits seconds to the right of Y. Only two people sit between R and W.

P sits to the immediate right of W. W faces outside.

Only one person sits between P and Z. Immediate neighbours of P face opposite directions.

Q sits third to the left of Z. Q is not immediate neighbour of P.

X faces directions opposite to that Y. X is neither an immediate neighbour of Y nor P.

Immediate neighbours of S face same direction. P does not face outside.

R and Q face a direction opposite to that of S.

33. Four of the following five are alike in a certain way based on the direction they are facing and so form a group. Which is the one that does not belong to that group?

(a)   YX

(b)   QZ

(c)   PQ

(d)   RW

(e)   XP

Answer: (d)

34. Who among the following sit exactly between X and P when counted from the left of X?

(a)   Z, R

(b)   W, S

(c)   R, Q

(d)   S, Z

(e)   Q, Y

Answer: (d)

35. Who among the following are immediate neighbours of Y?

(a)   Z, P

(b)   S, X

(c)   Q, S

(d)   W, Q

(e)   X, Z

Answer: (d)

36. What is Q’s position with respect to W?

(a)   Immediate left

(b)   Third to the left

(c)   Second to the left

(d)   Immediate right

(e)   Second to the right

Answer: (c)

37. Which of the following is true regarding S as per the given seating arrangement?

(a)   R is an immediate neighbour of S.

(b)   S faces outside.

(c)   Only two people sit between S and Q.

(d)   X sits second to the right of S.

(e)   None of the given options is true.

Answer: (e)

Directions (Q. Nos. 38-40) Study the following information and answer the given questions.

K is the brother of J. J is the mother of Y.

Y is the sister of T. T is married to Q. S is the father of J.

S has only one daughter. S is married to R.

K is the brother of D. U is the father-in-law of D.

38. How is D related to Y ?

(a)   Cannot  be determined

(b)   Mother

(c)   Uncle

(d)   Father

(e)   Aunt

Answer: (c)

39. If Y is married to S, then how is S related to T?

(a)   Cannot be determined

(b)   Sister

(c)   Brother-in-law

(d)   Sister-in-law

(e)   Brother

Answer: (c)

40. If U is the father of C, then how is D related to C?

(a)   Cousin

(b)   Sister

(c)   Wife

(d)   Cannot be determined

(e)   Brother

Answer: (d)

Part 2 Quantitative Aptitude

41. A trader sells an item to a retailer at 20% discount, but charges 10% on the discounted price, for delivery and packaging. The retailer sells it for Rs 2046 more, thereby earning a profit of 25%. At what price had the trader marked the item?

(a)   Rs 9400

(b)   Rs 9000

(c)   Rs 8000

(d)   Rs 12000

(e)   Rs 9300

Answer: (e)

42. X’s age 3 years ago was three times the present age of Y. At present, Z’s age is twice the age of Y. Also Z is 12 years younger than X. What is the present age of Z?

(a)   15 yr

(b)   24 yr

(c)   12 yr

(d)   6 yr

(e)   18 yr

Answer: (e)

43. Average score of a class of 60 students, in an exam, was 43. Average score of the students who had passed is 52 and the average score students who had failed is 16. How many failed the exam?

(a)   25

(b)   20

(c)   15

(d)   18

(e)   30

Answer: (c)

44. The interest received on a sum of money when invested in scheme A is equal to the interest received on the same sum of money when invested for 2 years in scheme B. Scheme A offers simple interest (percent per annum) and scheme B offers compound interest (compounded annually). Both the scheme offers the same rate of interest. If the numerical value of the number of years for which the sum is invested in scheme A is same as the numerical value of the rate of interest offered by the same scheme, what is the rate of interest (percent per annum) offered by scheme A?

(a)   3%

(b) 

(c) 

(d) 

(e)   2%

Answer: (d)

45. A vessel contains 100 L mixture of milk and water in the respective ratio of 22 : 3. 40 L of the mixture taken out from the vessel and 4.8 L of each pure milk and pure water is added to the mixture. By what percent is the quantity of water in the final mixture less than the quantity of milk?

(a) 

(b) 

(c) 

(d)   76%

(e) 

Answer: (b)

Directions (Q. Nos. 46-50) What will come in place of question mark (?) in the given number series?

46. 500   484    451    384    266    ?

(a)   36

(b)   80

(c)   56

(d)   64

(e)   42

Answer: (b)

47. 9    4        5        6        14      ?

(a)   48.5

(b)   42

(c)   32.5

(d)   20

(e)   36

Answer: (c)

48. 100    52      28      16      10      ?

(a)   7

(b)   3

(c)   6

(d)   4

(e)   5

Answer: (a)

49. 1     6        32      163    819    ?

(a)   1705

(b)   1066

(c)   4100

(d)   3800

(e)   4284

Answer: (c)

50. 3    5        13      43      177    ?

(a)   723

(b)   645

(c)   930

(d)   891

(e)   550

Answer: (d)

Directions (Q. Nos. 51-55) Refer to the pie-charts and answer the given questions.

51. What is the average number of Dell laptops sold by stores P, R and S together?

(a)   424

(b)   432

(c)   428

(d)   454

(e)   436

Answer: (b)

52. What is the central angle corresponding to number of Dell laptops sold by store S?

(a)   29.4°

(b)   38.6°

(c)   36.2°

(d)   32.4°

(e)   30.8°

Answer: (d)

53. Number of Dell laptops sold by store Q is approximately what percent of the number of the number of laptops (both Dell and Lenovo) sold by store R?

(a)   28%

(b)   45%

(c)   50%

(d)   38%

(e)   32%

Answer: (d)

54. What is the difference between number of laptops (both Dell and Lenovo) sold by store Q and total number of Lenovo laptops sold by store R and S together?

(a)   185

(b)   99

(c)   91

(d)   119

(e)   107

Answer: (a)

55. Number of Dell laptops sold b y store T is what percent more than the number of laptops sold by store P?

(a)   30%

(b)   45%

(c)   40%

(d)   42.5%

(e)   35%

Answer: (c)

56. A and B started a business with initial investments in the respective ratio of 18 : 7. After four months from the start of the business, A invest Rs 2000 more and B invested Rs 7000 more. At the end of one year, if the profit was distributed among them in the ratio of 2 : 1 respectively, what was the total initial investment with which A and B started the business?

(a)   Rs 50000

(b)   Rs 25000

(c)   Rs 150000

(d)   Rs 75000

(e)   Rs 125000

Answer: (a)

Directions (Q. Nos. 57-61) In these questions two equations numbered I and II are given. You have to solve both the equations and mark the appropriate option.

Given answer

(a) if x> y      (b) if x < y

(c) if x ≥ y     (d) if x ≤ y

(e) if relationship between x and y cannot be determined

57. (I) x2 – 4x – 12 = 0

(II) y2 – 5y – 14 = 0

Answer: (d)

58. (I) 3x2 – 22x + 40 = 0

(II) 5y2 – 21y + 16 = 0

Answer: (a)

59. (I) 25x2 + 35x + 12 = 0

(II) 10y2 + 9y + 2 = 0

Answer: (b)

60. (I) 12x2 + 7x + 1 = 0

(II) 6y2 + 5y + 1 = 0

Answer: (d)

61. (I) 3x2 – 13x – 10 = 0

(II) 3y2 + 10y – 8 = 0

Answer: (e)

62. The respective ratio between the monthly salaries of Rene and Som is 5 : 3. Out of her monthly salary Rene gives 1/6th as rent, 1/5th to her mother, 30% as her education loan and keeps 25% aside for miscellaneous expenditure. Remaining Rs 5000 she keeps as savings. What is Som’s monthly salary?

(a)   Rs 21000

(b)   Rs 24000

(c)   Rs 27000

(d)   Rs 36000

(e)   Rs 18000

Answer: (d)

Directions (Q. Nos. 63-67) Refer to the graph and answer the given questions.

3

63. What is the respective ratio of total number of calories burned by A and B together on Wednesday and that by the same individuals together on Tuesday?

(a)   45 : 59

(b)   43 : 57

(c)   41 : 57

(d)   43 : 61

(e)   47 : 61

Answer: (b)

64. If the number of calories burned by A and B increased by 10% and 20% respectively from Friday to Saturday, what was the total number of calories burned by them together on Saturday?

(a)   378

(b)   372

(c)   368

(d)   384

(e)   364

Answer: (b)

65. What is the total number of calories burned by A in Tuesday, Wednesday and Thursday together?

(a)   425

(b)   440

(c)   430

(d)   445

(e)   435

Answer: (e)

66. If the average number of calories burned by B on Thursday, Friday and Saturday together is 125, what was the number of calories burned by B on Saturday?

(a)   110

(b)   95

(c)   115

(d)   90

(e)   105

Answer: (b)

67. Number of calories burned by B increased by what percent from Monday to Thursday?

(a)   80%

(b)   60%

(c)   70%

(d)   75%

(e)   65%

Answer: (a)

68. A project requires 12 women to complete it in 16 days. 12 women started working and after a few days from the start of the project, 4 women left. If the remaining project was completed in 18 days, in how many days the whole project was completed?

(a) 

(b)   26 days

(c)   22 days

(d) 

(e)   20 days

Answer: (c)

Directions (Q. Nos. 69-73) Study the following information carefully to answer the given questions.

This data is regarding number of Junior College students, Graduate students and Post-Graduate (PG) students only, studying in colleges A, B, C and D.

The respective ratio between the total number of students studying in the colleges A, B, C and D is 3 : 5 : 2 : 5. In college A, 40% of the total number of students is Junior College students. Out of the remaining, the respective ratio between the number of Graduate students and number of PG graduates is 5 : 4.

In college B, 2/5th of the total number of students are Junior College students. Out of the remaining, the respective ratio between the number of Graduate students and number of PG students is 3 : 2. In college C, 50% of the total number of students is Junior College students. Out of the remaining, 5/8th are Graduate students and the remaining are PG students.

In College D, 35% of the total number of students is Graduate students, 8/13th of the remaining students are Junior College students and the rest 1500 are PG students.

69. What is the respective ratio between the total number of Graduate students in College A and the total number of Graduate students in College B?

(a)   2 : 5

(b)   5 : 9

(c)   7 : 15

(d)   9 : 20

(e)   9 : 5

Answer: (b)

70. What is the difference between the total number of PG students in College A and B together and that in College C and D together?

(a)   320

(b)   340

(c)   450

(d)   260

(e)   280

Answer: (c)

71. Number of Graduate students in College D are what percent more than the number of Graduate students in College C?

(a)   140%

(b)   200%

(c)   150%

(d)   180%

(e)   175%

Answer: (d)

72. What is the average of total number of Junior College students in College A, B and C together?

(a)   1585

(b)   1680

(c)   1640

(d)   1565

(e)   1660

Answer: (b)

73. What percent of the total number of students in College C are Junior College students?

(a)   45%

(b) 

(c) 

(d)   35%

(e)   32.25%

Answer: (e)

74. A man takes 2.2 times as long to row a distance upstream as to row the same distance downstream. If he can row 55 km downstream in 2 hours 30 minutes, what is the speed of the boat in still water?

(a)   40 km/h

(b)   8 km/h

(c)   16 km/h

(d)   24 km/h

(e)   32 km/h

Answer: (c)

Directions (Q. Nos. 75-79) Study the table and answer the given questions.

Note

(I) Ne revenue = Gross revenue – Amount allocated for commission – Amount allocated for discount and others

(II) Few values are missing in the table (indicated by −). A candidate is expected to calculate the missing value. If it is required to answer the given questions, on the basis of the given data and the information.

75. In July, if 40% of the Gross revenue of the magazine was collected from advertisement, what was the amount of Gross revenue collected from advertisement in that particular month?

(a)   Rs 148000

(b)   Rs 164000

(c)   Rs 144000

(d)   Rs 172000

(e)   Rs 156000

Answer: (d)

76. In March, if Net revenue of the magazine was 85% of its Gross revenue, what was the amount allocated for discount and others?

(a)   Rs 23200

(b)   Rs 24200

(c)   Rs 22400

(d)   Rs 22800

(e)   Rs 21600

Answer: (d)

77. Amount allocated for commission in March is what percent less than amount allocated for commission in July?

(a)   24%

(b)   18%

(c)   28%

(d)   32%

(e)   22%

Answer: (e)

78. What is the difference between Net revenue of the magazine in April and its Gross revenue in June?

(a)   Rs 132000

(b)   Rs 126000

(c)   Rs 118000

(d)   Rs 124000

(e)   Rs 136000

Answer: (b)

79. In May, the respective ratio of amount allocated for commission and amount allocated for discount and others was 4 : 3. What was the Gross revenue of the magazine in May?

(a)   Rs 424000

(b)   Rs 440000

(c)   Rs 380000

(d)   Rs 420000

(e)   Rs 430000

Answer: (d)

80. Two equal circles are drawn in square in such a way that a side of the square forms diameter of each circle. If the remaining area of the square is 42 cm2, how much will the diameter of the circle measures?

(a)   3.5 cm

(b)   4 cm

(c)   14 cm

(d)   7.5 cm

(e)   21 cm

Answer: ()

Part III English Language

Directions (Q. Nos. 81-85) Read each of the following sentences to find out whether there is any grammatical error in it. The error, if any, will be in one part of the sentence. Mark the part with the error as your answer. If there is no error, mark ‘No error’ as your answer. (Ignore the errors of punctuations, if any)

81. The economy is even weaker than it looks at first sight since official government figures show that investment and consumption lower than expected?

(a)   The economy is even weaker than

(b)   it looks at first sight

(c)   since official government figures show that

(d)   investment and consumption lower than expected

(e)   No error

Answer: (d)

82. The popular misconception is that corruption in sports is harmless as it is a victimless crime which can overlook.

(a)   The popular misconception is that

(b)   corruption in sports is harmless as

(c)   it is a victimless crime

(d)   which can overlook

(e)   No error

Answer: (d)

83. According to a consultant, big dams and hydropower offer great potential and can provide about 16% of Africa’s power by 2040 compared to solar power which will provide only about 10%.

(a)   According to a consultant, big dams and

(b)   hydropower offer great potential and can provide

(c)   about 16% of Africa’s power by 2040

(d)   compared to solar power which will provide only about 10%

(e)   No error

Answer: (d)

84. Many of the services which were once delivered on branches such as international transfers or personal loans are now being offered by new generation financial technology firms.

(a)   Many of the services which were once

(b)   delivered on  branches such as

(c)   international transfers or personal loans are now

(d)   being offered by new generation financial technology firms

(e)   No error

Answer: (e)

85. A system of secure employment and performance appraisal that are linked to age not performance have caused many Japanese firms to tail.

(a)   A system of secure employment

(b)   and performance appraisal that are linked

(c)   to age not performance

(d)   have caused many Japanese firms to tail

(e)   No error

Answer: (b)

Directions (Q. Nos. 86-90) Rearrange the following sentences A, B, C, D, E and F in a proper sequence to make a meaningful paragraph, then answer the given questions.

(A) In general, African nations have limited exposure, because they still price their goods in American dollars.

(B) So, while there is no immediate impact, in the medium, to long-term, the sales of African commodities and African currencies could take a knock because of this lowered demand.

(C) The immediate impact of the devaluation of the Yuan cannot be seen or measured, but countries that  have taken steps to transact in Chinese money could see pressure on their own local currencies.

(D) Therefore, the sale of commodities such as platinum, copper or coal may become more expensive, which could reduce demand.

(E) If the Yuan is devalued buying anything priced in dollars becomes pricier for the Chinese.

(F) Nonetheless as China is now the biggest customer they could feel the pinch indirectly.

86. Which of the following should THIRD sentence after rearrangement?

(a)   A

(b)   B

(c)   F

(d)   D

(e)   E

Answer: (c)

87. Which of the following should LAST (SIXTH) sentence after rearrangement?

(a)   A

(b)   F

(c)   D

(d)   C

(e)   E

Answer: (c)

88. Which of the following should FOURTH sentence after rearrangement?

(a)   A

(b)   B

(c)   C

(d)   E

(e)   D

Answer: (a)

89. Which of the following should FIRST sentence after rearrangement?

(a)   A

(b)   B

(c)   C

(d)   D

(e)   E

Answer: (c)

90. Which of the following should SECOND sentence after rearrangement?

(a)   A

(b)   B

(c)   C

(d)   D

(e)   F

Answer: (b)

Directions (Q. Nos. 91-95) In the following sentences there are two blanks for which options have been given. Choose the most appropriate option.

91. The natural oils …………. in roses help ……. the moisture in the skin.

(a)   built; arrest

(b)   found; retain

(c)   witnessed;  place

(d)   full; absorb

(e)   abundant; salvages

Answer: (b)

92. Parents must never …………… their children to play with catapults or augurs, as these so called toys can also ……….. harm to humans accidentally.

(a)   let; affect

(b)   sanction; prefer

(c)   authorize; result

(d)   permit; peace

(e)   allow; cause

Answer: (e)

93. The city police has ……….. a meeting of various mandals organizing festivals, to ………….. them of directives on the sound limit.

(a)   called; update

(b)   organized; convey

(c)   summoned; acquaint

(d)   found; full

(e)   need; place

Answer: (c)

94. When the traffic unit ………… multiple complaints about illegal parking, the police ………… penalties on the defaulters.

(a)   received; exposed

(b)   got; gave

(c)   endorsed; levied

(d)   collected; posed

(e)   issued; created

Answer: (d)

95. …………. the cobra was safely released into the wild, the injured eagle had to be ………… .

(a)   While; hospitalized

(b)   Although; released

(c)   Since; sent

(d)   Despite; caged

(e)   When; constricted

Answer: (a)

Directions (Q. Nos. 96-105) Read the following passage carefully and answer the following questions. Certain words/phrases have been given in bold to help you locate them while answering some of the questions.

Virtual currencies are growing in popularity. While the collective value of virtual currencies is still a fraction of t he total US Dollars in circulation, the use of virtual currencies as a payment mechanism of transfer of value is gaining momentum. Additionally, the number of entities (issuers, exchangers and intermediaries, to name just a few) that engage in virtual currency transactions is increasing and these entities often need access to traditional banking services.

Virtual currencies are digital representations of value that function as a medium of exchange, a u nit of account and a store of value (buy now redeem later policy). In many cases, virtual currencies are ‘convertible’ currencies; they are not legal lenders, but they have an equivalent value in real currency. Despite what seems to be a tremendous interest in virtual currencies their overall value is still extremely small relative to other payment mechanisms, such as cash, checks and credit and debit cards. The virtual currency landscape includes many participants from the merchant that accepts the virtual currency, to the intermediary that exchanges the virtual currency on behalf of the merchant, to the exchange that actually converts the virtual currency to the real currency to the electronic wallet provider that holds the virtual currency on behalf of its owner. Accordingly, opportunities abound for community banks to provide services to entities engaged in virtual currency activities. Eventually, it is also possible that community banks may find themselves holding virtual currency on their own balance sheets. Launched in 2009, Silicon is currently the largest and most popular virtual currency. However, many other virtual currencies have emerged over the past few years, such as Litecoin, Dogecoin, Peercoin and thse provide even more anonymity to its users than that provided by Bitcoin. As the virtual currency landscape is fraught with dangers, what important risks should community bankers consider?

The most significant is compliance risk-a subset of legal risk. Specifically, virtual currency administrators or legal exchangers may present risks similar to other money transmitters, as well in presenting their own unique risks. Quite simply, many users of virtual currencies do so because of the perceptions that transaction conducted using virtual currencies are anonymous. The less-than transparent nature of the transactions, may make it more difficult for a financial institution to truly know and understand the activities of its customer and whether the customer’s activities are legal. Therefore, these transactions may present a higher risk for banks and require additional due diligence and monitoring. Another important risk for community banks to consider is credit risk. How should a community bank respond if a borrower wants to specifically post Bitcoin or another virtual currency as collateral for a loan? For many, virtual currencies are simply another form of cash, so it is not hard to analyze that bankers will face such a scenario at some point. IN this case, caution is appropriate. Bankers should carefully weigh the pros and cons of extending any loan secured by Bitcoin or other virtual currencies (in whole or in part), or where the source of loan repayment is in some way dependent on the virtual currency.

For one, the value of Bitcoin in particular has been volatile. Then, the collateral value could fluctuate widely from day-to-day. Bankers also need to think about over the account. How does the banker control access to a virtual wallet, and how can it control the borrower’s access to the virtual wallet? In the event of a loan default, the bank would need to take control of the virtual currency. This would require access to the borrower’s virtual wallet and private key. All of this suggests that the loan agreement needs to be carefully crafted and that additional steps need to be taken to ensure the bank has a perfected left on the virtual currency. Virtual currencies bring with them, both opportunities and challenges, and they are likely here to stay. Although, it is too early to determine just how prevalent they will be in the coming years, we to expect that the virtual participants in the virtual currency ecosystem will increasingly intersect with the banking industry.

96. Which of the following is the meaning of the phrase ‘fraught with dangers’ is mentioned in the passage?

(a)   Healthy

(b)   Tensed

(c)   Evil

(d)   Risky

(e)   Stable

Answer: (d)

97. Which of the following is most nearly the most opposite in meaning to the word ‘volatile’ as used in the passage?

(a)   Suitable

(b)   Expensive

(c)   Temporary

(d)   Delicate

(e)   Suitable

Answer: (e)

98. Which of the following can be a suitable title for the passage?

(a)   Why virtual currencies work well in developing countries

(b)   Virtual currency and risk involved in its employment.

(c)   Reasons to convert real currency into virtual currency

(d)   Role of participants involved in handling real currency

(e)   Virtual Currency – An exercise in futility

Answer: (b)

99. As mentioned in the passage, banks may face which of t he following while dealing with virtual currencies?

(A) Since these transactions are largely untraceable, virtual currencies may be misused for criminal activities.

(B) The price of virtual currencies including Bitcoin is subject to significant fluctuations.

(C) There are few designated ATMs from which one can withdraw such currencies.

(a)   Only A

(b)   A and B

(c)   Only B

(d)   Only C

(e)   B and C

Answer: (c)

100. The author in the given passage

(a)   criticizes community banks which cater to consumers handling virtual currencies

(b)   wants to do completely do away with real currency

(c)   is optimize but cautious about the three employment of virtual currencies.

(d)   urges the government of his country to take legal action against those involved in virtual currencies.

(e)   is upset over the present economic situation in his country.

Answer: (c)

101. As mentioned in the passage, which of the following is/are the characteristics of virtual currencies?

(A) They are equivalent in value to other payment mechanisms.

(B) They cannot be destroyed.

(C) They can be used as later date as a medium of exchange.

(a)   Only A

(b)   B and C

(c)   Only B

(d)   Only C

(e)   A and B

Answer: (e)

102. Which of the following is most nearly the same in meaning to the word ‘gaining’ as used in the passage?

(a)   Attaining

(b)   Withdrawing

(c)   Highlighting

(d)   Talking

(e)   Increasing

Answer: (a)

103. Which of the following is most nearly the same in meaning to the word ‘crafted’ as used in the passage?

(a)   Designed

(b)   Stitched

(c)   Stoned

(d)   Stack

(e)   Divulged

Answer: (a)

104. Which of the following statements is true in the context of the passage?

(a)   Most entities such as money exchangers do not access to traditional banking services.

(b)   The value of virtual currencies is more than that of the real currency.

(c)   Bitcoin provides most privacy in comparison to recently introduced virtual currencies.

(d)   Community banks can play a crucial while catering to entities engaged in dealing with virtual currencies.

(e)   None of the given statements is true.

Answer: (a)

105. Which of the following is most nearly the opposite in meaning to the word’ anonymity’ as used in the passage?

(a)   Ambiguity

(b)   Surface

(c)   Beauty

(d)   Darkness

(e)   Visibility

Answer: (e)

Directions (Q. Nos. 106-110) In these question below a part of the sentence is printed in bold type. The part printed in bold type contain an error. Below each sentence four phrases (a), (b), (c) and (d) are given. One the these can substitute the part of the sentence in bold type to correct it. The letter of that part is the answer. If the part of the sentence, which is printed in bold type, is correct as it is, mark (e), i.e., ‘No correction required’ as your answer.

106. The cricketers admitted that they were practicing hard for the last six months.

(a)   had practiced hard for

(b)   had been practicing hard by

(c)   had been practicing hard for

(d)   had been practiced hardly

(e)   No correction required

Answer: (b)

107. Had he taken the medicine regularly, he would have felt better much quicker.

(a)   felt better more quickly

(b)   feeling better much quicker

(c)   been felt better more quickly

(d)   felt much better and quicker

(e)   No correction required

Answer: (c)

108. The number of cheating and forgery cases in disguise of running private commercial establishments has increased significantly.

(a)   disguising as

(b)   under the guise of

(c)   in guise of

(d)   disguised in

(e)   No correction required

Answer: (b)

109. It is not very difficult to dishing out a simple recipe for your child’s lunchbox.

(a)   for dishing about

(b)   to dish out

(c)   in order to dish

(d)   to out dish

(e)   No correction required

Answer: (b)

110. The city’s IT hub isn’t just decoding data or counting its export figures these days, but is rather aggressively beefed up ex-security both online and offline.

(a)   beefing

(b)   beef up

(c)   is beefing up

(d)   was beefing zup

(e)   No correction required

Answer: (a)

Directions (Q. Nos. 111-120) In the given passage, there are blanks, each of which has  been numbered. Against each five words are suggested, one of which fits the blank appropriately. Find out the appropriate words in each case.

After spending time and efforts to stabilize rural incomes in the face of plummeting agricultural prices, Thailand’s government has now banned its attention to dealing with unfair lending practices.  This is part of a wider (111) set in motion to reform State dominated rural credit markets. Thailand has made great strides in (112) access to financial services. 73% of the population now has a bank account and only 3% has no access (113) to formal finance. But lending by loan sharks has proved hard to stamp out. One reason is that Thais like to keep things informal the country’s shadow economy (114) for more than 50% of GDP-the highest in Asia. Household borrowing as a share of national income in Thailand now (115) at 68% of GDP, much higher than other middle income countries such as China (20%), India (16%) and Indonesia (17%). To make the matter (116) for loan sharks, the governments wants to strap an interest ceiling that has been in place for sixty years. The cap (117) registered non-bank lenders from changing more than 28% per year (including a 13% service charge). The idea is to encourage non-banks to provide formal credit to poor households. (118) institutions are typically community based groups founded by the Ministry of Inferior, NGOs, local governments and monks. The cap means that it is not (119) for these groups to get into rural lending. Thailand is a vast country and delivering financial services in remote areas is costly. A Thai loan shark typically (120) two lending schemes – a borrower can just pay interest of 2% per day every day for 24 days or repay the principle plus 2% daily interest in equal installments. Both re terrible deals and switching from a loan shark to a sound financial institution can save a household as much as $ 100000 per month.

111.

(a)   gap

(b)   spaces

(c)   effort

(d)   distance

(e)   confusion

Answer: (a)

112.

(a)   give

(b)   insisting

(c)   inhibiting

(d)   improving

(e)   ignoring

Answer: (d)

113.

(a)   service

(b)   whether

(c)   instead

(d)   whatsoever

(e)   further

Answer: (b)

114.

(a)   totals

(b)   value

(c)   equivalent

(d)   made

(e)   accounts

Answer: (d)

115.

(a)   reach

(b)   stands

(c)   fallen

(d)   arrive

(e)   received

Answer: (b)

116.

(a)   better

(b)   worse

(c)   difficult

(d)   easy

(e)   cozmplicated

Answer: (b)

117.

(a)   allow

(b)   prohibits

(c)   tries

(d)   interferes

(e)   confusion

Answer: (b)

118.

(a)   These

(b)   Which

(c)   Its

(d)   Through

(e)   When

Answer: (a)

119.

(a)   economical

(b)   necessity

(c)   essentially

(d)   rich

(e)   successfully

Answer: (a)

120.

(a)   other

(b)   have

(c)   choose

(d)   opts

(e)   profit

Answer: (d)

Part 4 General Awareness

121. The majority stakeholder in the capital of Regional Rural Banks (RRBs) is

(a)   Insurance/Mutual Fund companies

(b)   Central government

(c)   Sponsor Bank

(d)   Corporates/Individuals

(e)   State government

Answer: (b)

122. Recently launched ‘Nai Manzil’ scheme is aimed at

(a)   provision of Mid-day Meals in schools

(b)   development of weaker sections housing

(c)   research and development in science

(d)   providing employment opportunities to visually impaired

(e)   skill development among minority communities

Answer: (e)

123. The ‘Madison Square Garden’ is situated in the city of

(a)   London (United kingdom)

(b)   Canberra (Australia)

(c)   New York City (USA)

(d)   Paris (France)

(e)   Ottawa (Canada)

Answer: (c)

124. Magnetic Ink Character Recognition (MICR) code is a character recognition technology used mainly be the

(a)   insurance industry

(b)   banking industry

(c)   stoke brokerages

(d)   mutual fund industry

(e)   investment brokerage

Answer: (b)

125. The Reserve Bank of India (RBI) has recently granted a license to Reliance Industries Limited (RIL) to set-up a payments bank. For this purpose, the RIL has entered into a partnership with

(a)   State Bank of India

(b)   Union Bank of India

(c)   Bank of Baroda

(d)   Central Bank of India

(e)   ICICI Bank

Answer: (a)

126. The Reserve Bank of India (RBI) has recently allowed 11 business houses to start payments banks. These banks are not permitted to

(a)   offer debit cards/internet banking facilities

(b)   hold a balance of up to Rs 1 lakh per individual

(c)   accept NRI deposits

(d)   set-up branches, ATMs

(e)   distributed mutual fund insurance products

Answer: (e)

127. Indian Railways and LIC of India have recently signed a Memorandum of Understandings wherein over the next five years LIC will make insurance to the Ministry of Railways/its entities a financial assistance of up to

(a)   Rs 115000 crore

(b)   Rs 125000 crore

(c)   Rs 110000 crore

(d)   Rs 100000 crore

(e)   Rs 150000 crore

Answer: (e)

128. The 18th Asian Games also known as ‘Asiad’ shall be held in 2018 in

(a)   Bangkok (Thailand)

(b)   Jakarta (Indonesia)

(c)   Beijing (China)

(d)   Seoul (South Africa)

(e)   Tokyo (Japan)

Answer: (b)

129. The government of India has recently (August, 2015) released a Rs 5 postal stamp in honour of the Indian emperor

(a)   Chandragupta

(b)   Vikramaditya

(c)   Ashoka

(d)   Maharana Pratap

(e)   Bindusara

Answer: (c)

130. UCO Bank is a government owned bank with its headquarters at

(a)   Mumbai (Maharashtra)

(b)   Ahmedabad (Gujarat)

(c)   Chennai (Tamil Nadu)

(d)   Kolkata (West Bengal)

(e)   New Delhi (Delhi)

Answer: (d)

131. The ‘World Penumonia Day’ is observed every year on

(a)   November 12

(b)   August 16

(c)   September 16

(d)   October 8

(e)   August 28

Answer: (a)

132. The amount allocated in the Union Budget 2015-16 to the corpus of Rural Infrastructure Development Fund (RIDF) set-up with NABARD is

(a)   Rs 15000 crore

(b)   Rs 10000 crore

(c)   Rs 20000 crore

(d)   Rs 8000 crore

(e)   Rs 25000 crore

Answer: (e)

133. The ‘Aravali Range’ literally meaning ‘Line of Peaks’ is a range of mountains in Western India running through the States of Gujarat, Rajasthan, Haryana and

(a)   Maharashtra

(b)   Himachal Pradesh

(c)   Delhi

(d)   West Bengal

(e)   Madhya Pradesh

Answer: (c)

134. Bank customers can transfer money to anyone without knowing the Bank account details of the beneficiary through ‘mCASH’. A money transfer tool recently launched by

(a)   Union Bank of India

(b)   State Bank of India

(c)   Punjab National Bank

(d)   Corporation Bank

(e)   Bank of India

Answer: (b)

135. Finance Minister in 2015-16 Budget has stated that quoting PAN would be mandatory for all sale and purchase transactions of over

(a)   Rs 25000

(b)   Rs 100000

(c)   Rs 300000

(d)   Rs 50000

(e)   Rs 500000

Answer: (b)

136. A New Bank Note Paper Line with an annual capacity of 6000 tonnes has recently been inaugurated at the Security Paper Mill (SPM) in

(a)   Ujjain (Madhya Pradesh)

(b)   Nasik (Maharashtra)

(c)   Hoshangabad (Madhya Pradesh)

(d)   Navi Mumbai (Maharashtra)

(e)   Mysore (Karnataka)

Answer: (b)

137. In the first such tie-up of its kind, Kishore Biyani’s ‘Future Group’ will manage the overall Public distribution system and management of ration shops in Indian State of

(a)   Uttar Pradesh

(b)   Gujarat

(c)   Rajasthan

(d)   Maharashtra

(e)   Haryana

Answer: (c)

138. The ‘Veer Savarkar International Airport’ is also known as

(a)   Port Blair Airport

(b)   Chennai Airport

(c)   Nagpur Airport

(d)   Kochi Airport

(e)   Goa Airport

Answer: (a)

139. Who among the following headed the Jury to select India’s official entry for the 88th Academy Awards?

(a)   Amol Palekar

(b)   Rakhi Gulzar

(c)   Subhash Ghai

(d)   Deepa Mehta

(e)   Mahesh Bhatt

Answer: (a)

140. As per Census 2011, the most populous State in the country with a population of around 200 million, is

(a)   West Bengal

(b)   Uttar Pradesh

(c)   Bihar

(d)   Andhra Pradesh

(e)   Madhya Pradesh

Answer: (b)

141. The first-ever hydropower plant in Delhi which will be run through hydraulic turbines propested by effluent from a produce 20000 kwh of electricity per year has recently been commissioned by

(a)   Delhi Transport Corporation

(b)   Delhi Vidyut Board

(c)   Delhi Jal Board

(d)   Delhi University

(e)   Delhi Metro Corporation

Answer: (c)

142. While addressing the National banking Summit recently organized by IBA and FCCI, the RBI Governor Dr. Raghuram Rajan has stated India has forex reserves of around

(a)   $ 880 billion

(b)   $ 580 billion

(c)   $ 3200 billion

(d)   $ 520 billion

(e)   $ 886 billion

Answer: (c)

143. The Minister of State for Water Resources in the Union Cabinet is

(a)   Thawal Chand Gehlot

(b)   Chaudhary Bireder Singh

(c)   Mukhtar Abbas Naqvi

(d)   Giriraj Singh

(e)   Sanwar Lal Jat

Answer: (e)

144. The five BRICS countries represent over 3 billion people or

(a)   42% of world population

(b)   32% of world population

(c)   52% of world population

(d)   22% of world population

(e)   47% of world population

Answer: (a)

145. The Chairman of the National Disaster Management Authority (NDMA), is

(a)   National Security Advisor

(b)   Prime Minister of India

(c)   Chief of DRDO

(d)   Home Minister of India

(e)   Director General of CISF

Answer: (b)

146. The host of the 24th Commonwealth Heads of Government Meeting (CHoGM) 2015 is

(a)   Maldives

(b)   Myanmar

(c)   Malaysia

(d)   Mauritius

(e)   Malta

Answer: (e)

147. The ‘Article-14’ of the Indian Constitution specifies the

(a)   equality in matters of public employment

(b)   right to equal access to public places

(c)   right to freedom

(d)   right to equality before law

(e)   right to social equality

Answer: (d)

148. The author of the non-fiction book ‘Making India Awesome’ is

(a)   Kiran Desai

(b)   Vikram Seth

(c)   Salman Rushdie

(d)   Chetan Bhagat

(e)   Robinson Mistry

Answer: (d)

149. Which of the following institutions is a wholly-owned subsidiary of the Reserve Bank of India?

(a)   ARCIL

(b)   DICGC

(c)   ARSEC

(d)   CIBIL

(e)   ONICRA

Answer: (b)

150. Under the NITI Aayog, ‘Regional Councils’ will be formed to address specific issues and contingencies impacting more than one State of a region. These councils will be chaired by

(a)   Home Minister of India or his nominee

(b)   Chief Minister for the respective State

(c)   Vice-Chairman of NITI Aayog or his nominee

(d)   Chairperson of the NITI Aayog or his nominee

(e)   Ministry of Finance or his nominee

Answer: (d)

151. Under the ‘Digital India’ initiative, the government of India has recently launched a health service called Social Endeavour for Health and Telemedicine (SEHAT) in association with

(a)   Narayana Health Services

(b)   Apollo Hospitals

(c)   Escorts Hospitals

(d)   Fortis Hospitals

(e)   Max Healthcare

Answer: (b)

152. ‘Bandhan Bank’ was recently been launched as a

(a)   scheduled cooperative bank

(b)   scheduled local area bank

(c)   scheduled local area bank

(d)   scheduled regional rural bank

(e)   scheduled commercial bank

Answer: (e)

153. The government of Andhra Pradesh has recently announced a comprehensive development plan in association with of 264 smart villages in

(a)   Nellore Parliamentary constituency

(b)   Tripura Parliamentary constituency

(c)   Vijaywada Parliamentary constituency

(d)   Vishakhapatnam Parliamentary constituency

(e)   Anantapur Parliamentary constituency

Answer: (c)

154. Which of the following is the official currency and legal tender of Italy?

(a)   Peso

(b)   Euro

(c)   Franc

(d)   Casa

(e)   Dinar

Answer: (b)

155. The capital of Sweden is

(a)   Paris

(b)   London

(c)   Stockholm

(d)   Geneva

(e)   Oslo

Answer: (c)

156. Shiv Prakash Mishra who was been named to receive the prestigious Dhyan Chand Award for lifetime achievement is associated with which game?

(a)   Tennis

(b)   Golf

(c)   Hockey

(d)   Athletics

(e)   Football

Answer: (a)

157. India has recently signed a Memorandum of Understandings for the construction of the petroleum pipeline to export petroleum products to

(a)   Nepal

(b)   Bhutan

(c)   Bangladesh

(d)   Sri Lanka

(e)   Myanmar

Answer: (a)

158. ’31 October’ commemorated as the ‘National Unity Day’ to mark the birth anniversary of the

(a)   Dr. Rajendra Prasad

(b)   Lal Bahadur Shastri

(c)   Sardar Vallabhbhai Patel

(d)   Dr. S. Radhakrishnan

(e)   Madan Mohan Malviya

Answer: (c)

159. The ‘Shaheed Veer Narayan Singh International Cricket Stadium’ is located in the Indian State of

(a)   Uttar Pradesh

(b)   Chhattisgarh

(c)   Maharashtra

(d)   Assam

(e)   Madhya Pradesh

Answer: (b)

160. The International Maritime Organization (IMO) is a specialized agency of United Nations with 171 Member States three Associate Members. It is headquartered in

(a)   London (United Kingdom)

(b)   New York (USA)

(c)   Washington DC (USA)

(d)   Geneva (Switzerland)

(e)   Paris (France)

Answer: (a)

Part V Computer Knowledge

161. Usually, installation files have the extension

(a)   .jpeg

(b)   .gif

(c)   .exe

(d)   .doc

(e)   .bng

Answer: (c)

162. ……….. tags on goods enable the tracking of the good electronically

(a)   ASTM

(b)   GPS

(c)   RFID

(d)   ISP

(e)   IMB

Answer: (c)

163. Devices, such as monitors and printers, that are connected to the computer are called

(a)   processing devices

(b)   system devices

(c)   input devices

(d)   peripheral devices

(e)   attachment devices

Answer: (d)

164. RAM is considered volatile storage, which means it is

(a)   random

(b)   optional

(c)   permanent

(d)   read only

(e)   temporary

Answer: (e)

165. Which of the following do digital signatures provide?

(a)   Confidentiality and integrity of data

(b)   Confidentiality

(c)   Authentication and confidentiality of data

(d)   Authentication and integrity of data

(e)   Authentication and availability of data

Answer: (d)

166. If you copy and paste a file

(a)   the original file remains unchanged and a new file is not created

(b)   the original file remains unchanged and a new file is created

(c)   the original file is deleted and the new file is created

(d)   the original file is removed and new file is not created

(e)   the file gets scanned

Answer: (b)

167. An internet technology environment that sends information to a u ser without an explicit request from t he u ser, is

(a)   Infoware

(b)   Push

(c)   Pull

(d)   F202C

(e)   Wiki

Answer: (b)

168. Any ……… appearing on a web page opens another document when clicked.

(a)   heading

(b)   hyperlink

(c)   URL

(d)   reference

(e)   anchor

Answer: (b)

169. Which of the following is the correct sequence, the unit of digital information, smallest to largest?

(a)   Kilobyte, Gigabyte, Megabyte, Terabyte

(b)   Megabyte, Terabyte, Kilobyte, Gigabyte

(c)   Kilobyte, Megabyte, Terabyte, Gigabyte

(d)   Megabyte, Kilobyte, Gigabyte, Terabyte

(e)   Kilobyte, Megabyte, Gigabyte, Terabyte

Answer: (e)

170. Which of the following will you consider as most secure?

(a)   Log-in phrase

(b)   Log-in numeral

(c)   Password

(d)   Log-in ID

(e)   One time Password

Answer: (e)

171. …………. are specially designed computers that perform complex calculation extremely rapidly.

(a)   Supercomputers

(b)   Servers

(c)   Mainframes

(d)   Desktops

(e)   Laptops

Answer: (a)

172. Which of the following refers to programs stored in ROM?

(a)   Hardware

(b)   Programme efforts

(c)   Firmware

(d)   Peripheral

(e)   Software

Answer: (c)

173. Which of the following refers to restarting the system when it is already powered on?

(a)   A strong boot

(b)   A warm boot

(c)   A cold boot

(d)   Hibernation

(e)   Standby mode

Answer: (b)

174. ………….. bits equal one byte.

(a)   Eight

(b)   One thousand

(c)   Four

(d)   Two

(e)   One million

Answer: (a)

175. A (n) …………. analyzes and executes each one of the source code in succession, without looking at the entire program.

(a)   operating system

(b)   computer

(c)   interpreter

(d)   utility

(e)   device driver

Answer: (c)

176. In a microcomputer system, the CPU is contained on a single chip called the

(a)   ROM

(b)   Control Unit

(c)   ALU

(d)   Semiconductors

(e)   Microprocessor

Answer: (e)

177. An EULA in computing stands for

(a)   End-use License Arrangement

(b)   Environmental Use Licensing Agreement

(c)   End-Use License Agreement

(d)   Everyday-User Learning Assistance

(e)   Employee Use Legal Arrangement

Answer: (c)

178. Storage that retains its data after the power is turned off, is referred to as

(a)   sequential storage

(b)   non-volatile storage

(c)   volatile storage

(d)   direct storage

(e)   non-destructive storage

Answer: (b)

179. The individual dots that form the image on a monitor are called

(a)   bits

(b)   bytes

(c)   pixels

(d)   picas

(e)   pitens

Answer: (c)

180. Which of the following is an electronic or paper log used to track computer activity?

(a)   Weblog

(b)   Traceroute

(c)   Monitor

(d)   Cookie

(e)   Internet trait

Answer: (e)

181. FTP in computing stands for

(a)   File Transfer Project

(b)   File Tagging Program

(c)   File Transfer Program

(d)   File Transfer Protocol

(e)   File Tagging Protocol

Answer: (d)

182. In the e-mail address [email protected], ‘abc’ is the

(a)   user name

(b)   password

(c)   client computer

(d)   server name

(e)   Other than those given as options

Answer: (a)

183. If you change Windows 98 operating system to Windows XP, you are actually performing a(n)

(a)   push up

(b)   pull down

(c)   patch

(d)   upgrade

(e)   update

Answer: (d)

184. Security procedures can

(a)   reduce but not eliminate risks

(b)   be inaccessible for the average home user

(c)   eliminate all computer security risks

(d)   dissuade computer usage

(e)   Other than those given as options

Answer: (a)

185. ……….. refers to the unauthorized copying and distribution and software.

(a)   Software piracy

(b)   Plagiarism

(c)   Hacking

(d)   Software literacy

(e)   Cracking

Answer: (a)

186. Which of the following refers to an upside down mouse?

(a)   Trackwell

(b)   Trackboard

(c)   Trackball

(d)   Trackpoint

(e)   Trackpad

Answer: (c)

187. Which of the following users tunneling protocol to securely send private network data over the internet?

(a)   VPN

(b)   WAN

(c)   LAN

(d)   GPS

(e)   Other than those given as options

Answer: (a)

188. Which of the following devices sends and receives data over telephone lines to and from computers?

(a)   Sound Card

(b)   Speaker

(c)   Expansion Slot

(d)   Printer

(e)   Modem

Answer: (e)

189. Which of the following refers to a technique for intercepting computer communications?

(a)   Phishing

(b)   Spoofing

(c)   Pretexting

(d)   Hacking

(e)   Sniffing

Answer: (e)

190. The potential impact of a threat on a system is called as its

(a)   Danger potential

(b)   Degree of harm

(c)   Susceptibility

(d)   Vulnerabilities

(e)   Countermeasures

Answer: (b)

191. Which technology allows users to shift their desktop computing activities to computers on the internet?

(a)   Cloud computing

(b)   Solid-state storage

(c)   Distributed database

(d)   Centralized database

(e)   Other than those given as options

Answer: (a)

192. Converting the computer language of 1s to 0s to characters that a person can understand is called

(a)   Generating

(b)   Decoding

(c)   Selecting

(d)   Creating Clip Art

(e)   Other than those given as options

Answer: (b)

193. The system conversion involves running the old and the new system and comparing their results

(a)   pilot conversion

(b)   direct conversion

(c)   parallel conversion

(d)   convergent conversion

(e)   phased conversion

Answer: (c)

194. Every host computer on the Internet has a(n)

(a)   unique IP address

(b)   unique 10 digit number

(c)   similar IP address

(d)   unique 15 digit number

(e)   Other than those given as options

Answer: (a)

195. Which of the following is/are effective controls for detecting duplicate transactions such as payments made or received?

(a)   Concurrency controls and referential Integrity Controls

(b)   Concurrency Controls

(c)   Reasonableness Checks

(d)   Time Stamps

(e)   Referential Integrity Controls

Answer: (d)

196. Authorization is best characterized as

(a)   certifying a user’s authority

(b)   providing access to a resource according to the principle of least privilege

(c)   ensuring maximum utilization of resources

(d)   authenticating a user’s identify with a password

(e)   a user providing an identity and a password

Answer: (d)

197. What is a detailed process for recovering information or an IT system in the event of a catastrophic disaster such as a fire or food?

(a)   Hot site

(b)   Warm site

(c)   Disaster recovery cost curve

(d)   Buffer plan

(e)   Disaster recovery plan

Answer: (e)

198. TCP/IP stands for

(a)   Translation Computing Procedures/International Protocol

(b)   Transmitter Control Protocol/Internet Protocol

(c)   Transmission Control Procedures/Internet Procedures

(d)   Transmission Control Protocol/Internet Protocol

(e)   Transaction Computing Printing/Internet Processing

Answer: (d)

199. Superscript, Subscript, Outline, Emboss, Engrave are known as

(a)   text effects

(b)   font effects

(c)   word art

(d)   clip art

(e)   Other than those given as options

Answer: (b)

200. …………… is a group of servers that share work and may be able to back each other up if one server fails.

(a)   Tiger team

(b)   Channel bank

(c)   Cluster

(d)   Logical unit

(e)   Serverless back up

Answer: (c)

IBPS CWE-V PO/MT Preliminary Online Examination Held on October 4, 2015 Question Paper With Answer Key

IBPS CWE-V PO/MT Preliminary Online Examination Held on October 4, 2015
IBPS CWE-V PO/MT Preliminary Online Examination Held on October 4, 2015 Question Paper With Answer Key

IBPS CWE-V PO/MT Preliminary Online Examination Held on October 4, 2015

Directions (Q. Nos. 1-5) Read each sentence to find out whether there is any grammatical error or idiomatic error in it. The error, if any, will be in one part of the sentence. The letter of that part is the answer. If there is no error, the answer is (e). (Ignore errors of punctuation, if any)

1. How serious is the country’s/ economic problems, and how/ big an impact will these/ have on the world economies?

(a)  How serious is the country’s

(b)  economic problems, and how

(c)  big an impact will these

(d)  have on the world economies

(e)  No error

Answer: (a)

2. Shuber, the taxi service provider, is/ growing like a weed,/ spending millions of rupees/ to establish its roots in the country.

(a)  Shuber, the taxi service provider, is

(b)  growing like a weed

(c)  spending millions of rupees

(d)  to establish its roots in the country

(e)  No error

Answer: (e)

3. The survey asked respondents/ from more than 50 countries to/ identify kinds of people/ they would want of neighbours.

(a)  The survey asked respondents

(b)  from more than 50 countries to

(c)  identify kinds of people

(d)  they would want of neighbours

(e)  No error

Answer: (d)

4. The pace and scale of/ the country’s economic transformation / have no/ historical precedent.

(a)  The pace and scale of

(b)  the country’s economic transformation

(c)  have no

(d)  historical precedent

(e)  No error

Answer: (c)

5. The countries most/ affected by the country’s slowdown/ are likely to be / those whose export raw materials.

(a)  The countries most

(b)  affected by the country’s slowdown

(c)  are likely to be

(d)  those whose export raw materials

(e)  No error

Answer: (d)

Directions (Q. Nos. 6-10) Rearrange the given six sentences/group of sentences A, B, C, D, E and F in a proper sequence so as to form a meaningful paragraph and then answer the given questions.

(A) “It’s undefined No one knows if you reach it. It gives the flexibility to revise it later.” he added.

(B) Off late, doubts are being raised concerning the health of the world’s second-largest economy, China.

(C) This change is relatively small but suggests that the country’s effort to meet its official growth target was tougher than it seemed.

(D) If comes as worries grow that China will struggle to reach this year’s goal of ‘about’ 7%.

(E) “That’s the beauty of using ‘about’ in your targets,” said HIS Global Insight economist Brian Jackson.

(F) The reasons for these doubts stem from the fact that the country revised its 2014 growth rate to 7.3% from 7.5% due to weaker than reported contribution from the services sector.

6. Which of the following should be the FIRST sentence after rearrangement?

(a)  A

(b)  C

(c)  B

(d)  F

(e)  E

Answer: (c)

7. Which of the following should be the SIXTH (LAST) sentence after rearrangement?

(a)  C

(b)  F

(c)  D

(d)  B

(e)  A

Answer: (a)

8. Which of the following should be the FOURTH sentence after rearrangement?

(a)  A

(b)  D

(c)  C

(d)  F

(e)  E

Answer: (e)

9. Which of the following should be the SECOND sentence after rearrangement?

(a)  A

(b)  B

(c)  F

(d)  D

(e)  C

Answer: (c)

10. Which of the following should be the FIFTH sentence after rearrangement?

(a)  A

(b)  E

(c)  D

(d)  F

(e)  C

Answer: (a)

Directions (Q. Nos. 11-15) Each sentence below has two blanks, each blank indicates that something has been omitted. Choose the words that best fits the meaning of the sentence as a whole.

11. ……….. he had mixed success in the past with his new technique; this time around the player had enough …….. up his sleeve to win this match.

(a)  Since; talent

(b)  Despite; drama

(c)  Although; tricks

(d)  Because; energy

(e)  Hence; magic

Answer: (c)

12. Name of the roles offered to me ………. the strong, funny and dynamic Indian woman that I had grown up …………. .

(a)  stated; today

(b)  said; surrounded

(c)  reflected; around

(d)  depicted; to

(e)  assessed; within

Answer: (d)

13. The ………… to the free sim-card scheme offered by the telecom company has been ………….and most people expressed complete ignorance about the scheme.

(a)  reaction; stupendous

(b)  access; cordial

(c)  contract; simple

(d)  takers; high

(e)  response; abysmal

Answer: (e)

14. In our close relations it is easy to come ………. clever men and women, but ……….. to find virtuous ones.

(a)  find; simpler

(b)  up; arduous

(c)  see; terrible

(d)  across; difficult

(e)  close; impossible

Answer: (d)

15. Due to the …………. number of swine-flu cases in the district, the health department has decided to spread ……….. about the disease.

(a)  raising; alertness

(b)  populated; knowledge

(c)  prolonged; understanding

(d)  increasing; awareness

(e)  high; ability

Answer: (d)

Directions (Q. Nos. 16-25) Read the following passage carefully and answer the questions given below it. Certain words/phrases are given in bold in the passage to help you locate them while answering some of the questions.

There is good news in the form of Europe’s unemployment falling from 11.1% in June to 10.0% in July. But unemployment generally lags behind the economic cycle. Business surveys, which provide more up-to-date readings to activity, point to a continuing subdued recovery in Europe. The European commission’s long-running economic-sentiment indicator, which combines business as well as consumer confidence and tends to track GDP has been broadly stable since picking up in early 2015. This suggests that the Euro area is not about to break out of its unspectacular growth. This is worrying because the euro-zone economy is benefitting from a powerful triple stimulus. Lower energy costs caused by the slump in global oil prices have been providing the same effect as a tax cut. A big  programme of Quantitative Easing (QE); has been under way since, early in the year under, which the European Central Bank (ECB) is creating money to buy 60 billion ($67 billion) of ponds each month. As well as pushing down long-term interest rates QE as helped to keep the euro down on the currency markets to the benefit of exporters. Given the extent of help that the euro area  has been getting, growth should be faster. The sluggish performance leaves it vulnerable to China’s slowdown. A particular worry is the impact of weakening Chinese growth on Germany, the hub economy of the region, whose resilience has been crucial in sustaining Europe since the euro crisis started five years ago. One reason has been strong Chinese demand for traditional German manufacturing strengths. Even though German exports appear to be holding up for the time being, that boost from China is waning. Lacklustre growth in the euro area will in turn make it harder for the ECB to meet its goal of pushing inflation back towards its goal of almost 2%. Although core inflation (excluding in particular energy and food) has moved up from its low of 0.6% earlier this year, to 1.0%, headline inflation (excluding in particular energy and food) has moved up from its low of 0.6% earlier this year, to 1.0%, headline inflation has been stuck at 0.2% over the summer. ‘There is increasing concern that the ECB’s effort to break the grip of ‘lowflation’ will be swamped by global deflationary effects. The ECB’s council is not expected to make a change in policy and is likely to indicate that the ECB recognizes the downside risk to growth and stands ready to respond if they materialize. That may in turn produce a policy erasing later this year. One option would be to raise the amount of assets that it is buying each month from the current amount of 60 billion. A more likely decision would be for the ECB to extend the planned length of its purchase some for another year. Whether that is enough is question for another day.

16. What is the author’s view regarding the reforms implemented by ECB?

(a)  He is outraged that these measures are being continued.

(b)  He is doubtful about the reforms as inflation is rising.

(c)  These reforms are tough and unpopular in Euro countries.

(d)  These are not focused and have not been effective at all.

(e)  Other than those given as options

Answer: (b)

17. What do the statistics cited in the passage about ECB convey?

(a)  Its stimulus package for Europe’s economy is too low in value.

(b)  ECB is likely to fast run out of ‘bailout’ funds.

(c)  Despite the ECB’s best efforts, its reforms have been unsuccessful.

(d)  It is time ECB withdraws it stimulus package to rich countries.

(e)  ECB has not been proactive in the economic recovery.

Answer: (e)

18. Choose the word which is most opposite in meaning to the word ‘Subdued’ given in bold as used in the passage.

(a)  Passive

(b)  Necessary

(c)  Sleep

(d)  Diluted

(e)  Expansive

Answer: (e)

19. Choose the word which is most opposite in meaning to the word ‘Broadly’ given in bold as used in the passage.

(a)  Widely

(b)  Barely

(c)  Softly

(d)  Responsibly

(e)  Dimly

Answer: (b)

20. Which of the following best describes the conclusion which can be drawn from economic indicators from Europe?

(a)  Investors do not have confidence in European markets.

(b)  Germany in the only economy unaffected by the crisis of 2008 and China’s slowdown.

(c)  Tax rates in Europe are being raised which has hampered growth.

(d)  By and large Europe’s economic performance is steady but not accelerating.

(e)  Europe has not taken measures to protect itself from the crisis in emerging markets.

Answer: (d)

21. Which of the following is true in the context of the passage?

(a)  Germany’s economic performance is linked to the Chinese market.

(b)  Europe’s large economies have recovered from the global crisis.

(c)  Exorbitant oil prices are negating the effects of ECB’s stimulus.

(d)  Europe’s unemployment rate is rising but it may not continue to do so.

(e)  None of the given options is true in the context of the passage.

Answer: (a)

22. According to the passage, which of the following has affected inflation in Europe?

(A) Global deflation is prevailing.

(B) Frequent rising of ECB’s interest rates.

(C) The ECB has withdrawn its Quantitative Erasing programme.

(a)  Only B

(b)  Only A

(c)  All of these

(d)  A and C

(e)  B and C

Answer: (b)

23. Choose the word which is most opposite in meaning to the word ‘Materialize’ given in bold as used in the passage.

(a)  Disappear

(b)  Perform

(c)  Terrorise

(d)  Occur

(e)  Substance

Answer: (a)

24. Which of the following is the central idea of the passage?

(a)  The European economy is buoyant as its energy and manufacturing sectors are doing well.

(b)  Emerging markets have been responsible for Europe’s slow economic performance.

(c)  The European Central bank is denial about the imminent crisis Europe is facing.

(d)  Europe has reduced its exposure to America and Asia.

(e)  The European economy is stagnating leaving it vulnerable to future crises.

Answer: (e)

25. Choose the word which is most nearly the same in meaning to the word ‘Grip’ given in bold as used in the passage.

(a)  Understanding

(b)  Authority

(c)  Awareness

(d)  Clutches

(e)  Fascination

Answer: (d)

Directions (Q. Nos. 26-30) In the following passage, there are blanks, each of which has been numbered. Against each, five words are suggested, one of which fits the blank appropriately. Find out the appropriate word in each case.

Creating a few more schools or allowing hundreds of colleges and private universities to (26) is not going to solve the crisis of education in India. And a crisis it is we are in a country where people are spending their parents’ savings and borrowed money on education- and even then no getting standard education, and struggling to find employment of their (27). In this country, millions of students are (28) of an unrealistic,  pointless, mindless rat race. The mind numbing competition and rote learning not only (29) the creativity and originality of millions of Indian students every year, but also (30) brilliant students to adopt drastic measures.

26.

(a)  base

(b)  mushroom

(c)  point

(d)  set

(e)  crop

Answer: (b)

27.

(a)  parents

(b)  money

(c)  fashion

(d)  equal

(e)  choice

Answer: (e)

28.

(a)  victims

(b)  member

(c)  party

(d)  associates

(e)  together

Answer: (a)

29.

(a)  crush

(b)  flourish

(c)  stir

(d)  halting

(e)  push

Answer: (a)

30.

(a)  aim

(b)  drive

(c)  stop

(d)  responsible

(e)  make

Answer: (e)

Part II Quantitative Aptitude

Directions (Q. Nos. 31-35) Study the table and answer the given questions.

Note I. Employees of the given companies can be categorized only in three types : Science graduates, Commerce graduates and Arts graduates.

II. Few values are missing in the table (indicated by -). A candidate is expected to calculate the missing value, if it is required to answer the given questions, on the basis of the given data and information.

31. What is the difference between the number of Arts graduate employees and Science graduate employees in Company N?

(a)  87

(b)  89

(c)  77

(d)  81

(e)  73

Answer: (c)

32. The average number of Arts graduate employees and Commerce graduate employees in Company Q was 312. What was the total number of employees in Company Q?

(a)  920

(b)  960

(c)  1120

(d)  1040

(e)  1080

Answer: (b)

33. If the respective ratio between number of Commerce graduate employees and Arts graduate employees in Company M was 10 : 7., employees in M?

(a)  294

(b)  266

(c)  280

(d)  308

(e)  322

Answer: (a)

34. Total number of employees in Company N increased by 20% from December, 2012 to 2013. If 30% of the total number of employees in Company N in December, 2013 was Science graduates, what was the number of Science graduate employees in Company N in December, 2013?

(a)  224

(b)  266

(c)  294

(d)  252

(e)  238

Answer: (d)

35. Total number of employees in Company P was 3 times the total number of employees in Company O. If the difference between number of Arts graduate employees in Company P and that in Company O was 180, what was the total number of employees in Company O?

(a)  1200

(b)  1440

(c)  720

(d)  900

(e)  1080

Answer: (d)

36. A bag contains 6 red balls, 11 yellow balls and 5 pink balls. If two balls are drawn at random from the bag, one after another, what is the probability that first ball is red and the second ball is yellow?

(a)  1/14

(b)  2/7

(c)  5/7

(d)  3/14

(e)  None of these

Answer: (e)

37. The sum of the radius and height of a cylinder is 19 m. The total surface area of the cylinder is 1672 m2, what is the volume of the cylinder?

(a)  3080 m3

(b)  2940 m3

(c)  3220 m3

(d)  2660 m3

(e)  2800 m3

Answer: (a)

Directions (Q. Nos. 38-42) What will come in place of question mark (?) in the given numbers series?

38. 264   262    271    243    308    ?

(a)  216

(b)  163

(c)  194

(d)  205

(e)  182

Answer: (e)

39. 1.5     2.5     7        24      100    ?

(a)  345

(b)  460

(c)  525

(d)  380

(e)  505

Answer: (e)

40. 71    78      92      120    ?        288

(a)  160

(b)  164

(c)  199

(d)  208

(e)  176

Answer: (e)

41. 17    9        10      5   35      ?

(a)  192

(b)  80

(c)  114

(d)  76

(e)  90

Answer: (e)

42. 79   39      19      94      ?

(a)  0.2

(b)  1.5

(c)  0.5

(d)  2

(e)  1

Answer: (b)

43. The respective ratio between the speed of the boat upstream and speed of the boat downstream is 2 : 3. What is the speed of the boat in still water if it covers 42 km downstream in 2 h 20 min?

(a)  13.5 km/h

(b)  15 km/h

(c)  14.5 km/h

(d)  12 km/h

(e)  Other than those given as options

Answer: (d)

44. Present age of Raman is equal to the Meera’s 8 years back age. The ratio between Meera’s and Raman’s age after four years will be 5 : 4. What is the present age of Raman?

(a)  28 yr

(b)  24 yr

(c)  32 yr

(d)  40 yr

(e)  20 yr

Answer: (a)

Directions (Q. Nos. 45-49) What should come at the place of question mark (?) in the following questions?

45. 421 ÷ 35 ×97 ÷ 25.05 = ?2

(a)  22

(b)  24

(c)  28

(d)  12

(e)  18

Answer: (d)

46. 

(a)  885

(b)  725

(c)  825

(d)  650

(e)  675

Answer: (b)

47. 99 × 15.98 + 224.98 + 125.02 = ?

(a)  620

(b)  580

(c)  670

(d)  560

(e)  520

Answer: (c)

48. 99 + 285.02 + 600.02 – 4.01 = ?

(a)  3650

(b)  4120

(c)  4200

(d)  3225

(e)  3400

Answer: (b)

49. ?% of 1239.96 + 59.87% of 449.95 = 579.05

(a)  35

(b)  15

(c)  25

(d)  20

(e)  30

Answer: (c)

50. A man sold an article for Rs 6800 and incurred a loss. Had he sold the article for Rs 7850, his gain would have been equal to half of the amount of loss that he incurred. At what price should he sell the article to have 20% profit?

(a)  Rs 7500

(b)  Rs 9000

(c)  Rs 8000

(d)  Rs 8500

(e)  Rs 9200

Answer: (b)

51. A certain sum is divided among P, Q and R is such a way that P gets Rs 40 more than the 1/2 of the sum. Q gets Rs 120 less than 3/8th of the sum and R gets Rs 200. What is the total sum?

(a)  Rs 960

(b)  Rs 850

(c)  Rs 906

(d)  Rs 805

(e)  None of these

Answer: (a)

Directions (Q. Nos. 52-56)In these questions, two equations numbered I and II are given. You to solve both the equations and mark the appropriate option.

Give answer

(a) if x > y     (b) if x < y

(c) if x ≥ y     (d) if x ≤ y

(e) if relationship between x and y cannot be established

52. I. 3x2 + 11x + 6 = 0 II. 3y2 + 10y + 8 = 0

Answer: (e)

53. I. 3x2 – 7x + 2 = 0 II. 2y2 – 9y + 10 = 0

Answer: (d)

54. I. x2 = 9 II. 2y2 – 19y + 44 = 0

Answer: (b)

55. I. 2x2 – 15x + 28 = 0 II. 4y2 – 23y + 30 = 0

Answer: (e)

56. I. 2x2 – 15x + 27 = II. 5y2 – 26y + 33 = 0

Answer: (c)

Directions (Q. Nos. 57-61) Refer to the graph and answer the given questions.

57. What is the average number of watches sold in Town X in January, February, March and June?

(a)  180

(b)  190

(c)  175

(d)  170

(e)  185

Answer: (d)

58. The number of watches sold in Town Y in April is what percent more than the number of watches sold in Town X in the same month?

(a)  42%

(b)  40%

(c)  30%

(d)  50%

(e)  38%

Answer: (b)

59. The number of watches sold in Town Y increased by what percent from February to May?

(a)  96%

(b) 

(c)  97%

(d) 

(e) 

Answer: (d)

60. The number of watches sold in Town X in July z was 10% more than the number of watches sold in the same Town in May. What is the respective ratio between the number of watches sold in July and those sold in January in the same town?

(a)  34 : 23

(b)  32 : 25

(c)  31 : 20

(d)  33 : 23

(e)  33 : 20

Answer: (e)

61. What is the difference between the total number of watches sold in both the towns together in June and the total number of watches sold in both the towns together in March?

(a)  50

(b)  90

(c)  60

(d)  70

(e)  80

Answer: (b)

62. A tank has two inlets : P and Q. P alone takes 6 hours and Q alone takes 8 hours to fill the empty tank completely when there is no leakage. A leakage was caused which would empty the full tank completely in X hours when no inlet is open. Now, when only inlet P was opened, it took 15 hours to fill the empty tank completely. How much time will Q alone take to fill the empty tank completely?

(a)  15 h

(b)  10 h

(c)  17 h

(d)  20 h

(e)  28 h

Answer: (e)

63. At present, the respective ratio between the ages of A and B is 3 : 8 and that between A and C is 1 : 4. Three years ago, the sum of ages of A, B and C will be 83 years. What is the present age of C?

(a)  32 yr

(b)  12 yr

(c)  48 yr

(d)  54 yr

(e)  15 yr

Answer: (c)

64. The sum invested in Scheme B is thrice the sum invested in Scheme A. Investment in Scheme A is made for 4 years at 8% per annum simple interest and in Scheme B for 2 years at 13% per annum simple interest. The total interest earned from both the schemes is 1320. How much was invested in Scheme A?

(a)  Rs 1200

(b)  Rs 1140

(c)  Rs 960

(d)  Rs 1500

(e)  Rs 840

Answer: (a)

65. Kim and Om are travelling from point A to B, which are 400 km apart. Travelling at a certain seed Kim takes one hour more than Om to reach point B. If Kim doubles her speed she will take 1 hours 3 min less than Om to reach point B. At what speed was Kim driving from point A to B?

(a)  90 km/h

(b)  70 km/h

(c)  160 km/h

(d)  80 km/h

(e)  100 km/h

Answer: (d)

Part III Reasoning Ability

Directions (Q. Nos. 66-70) In these questions two/three statements followed by two conclusions numbered I and II have been given. You have to take the given statements to be true even if they seem to at variance from commonly known facts and then decide which of the given conclusions logically follows from the given statements disregarding commonly known facts.

Give answer

(a) if only conclusion I follows

(b) if only conclusion II follows

(c) if either conclusion I or II follows

(d) if neither conclusion I nor II follows

(e) if both conclusions I and II follow

66. Statements Some tasks are hurdles. All hurdles are jobs. Some jobs are works.

Conclusions I. All works being hurdles is a possibility.

II. At least some works are tasks.

Answer: (a)

67. Statements Some tasks are hurdles.

All hurdles are jobs.

Some jobs are works.

Conclusions I. Some jobs are tasks.

II. All jobs are tasks.

Answer: (c)

68. Statements Some problems are solutions.

No solution is a trick.

All rules are tricks.

Conclusions I. No rule is a solution.

II. Some problems are definitely not tricks.

Answer: (e)

69. Statements All minister are deans.

Some deans are heads.

Some heads are principals.

Conclusions I. No principal is a minister.

II. All heads being ministers is a possibility.

Answer: (b)

70. Statements No queue is a line.

Some queues are rows.

Conclusions I. No row is a line.

II. All rows are lines.

Answer: (d)

Directions (Q. Nos. 71-72) Study the following information and answer the given questions.

D is the father of A. D is married to P. P is the mother of J. P has only one daughter. J is married to U. U the son of L.

71. How is J related to L?

(a)  Daughter

(b)  Granddaughter

(c)  Cannot be determined

(d)  Niece

(e)  Daughter-in-law

Answer: (e)

72. How is A related to U?

(a)  Cannot be determined

(b)  Brother-in-law

(c)  Brother

(d)  Sister

(e)  Sister-in-law

Answer: (b)

Directions (Q. Nos. 73-77) Study the given information carefully and answer the given questions.

Ten people are sitting in two parallel rows containing five people each, in such a way that there is equal distance between adjacent persons. In row-1 J, K, L, M and N are seated (not necessarily in the same order) and all of them are facing North. In row-2 R, S, T, U and V are seated’ (not necessarily in the same order) and all of them are facing South. Therefore in the given seating arrangement each members seated in a row faces another member of the other row.

V sits at one of the extreme ends of the line. Only two people sit between V and R. The one who faces R sits to the immediate left of L. Only one person sits between L and K. The one who faces K sits to the immediate left of S. N sits second to the right of J. Neither K nor L face U.

73. Who amongst the following is facing M?

(a)  U

(b)  R

(c)  T

(d)  V

(e)  S

Answer: (a)

74. Four of the following five are alike in a certain way based on the given arrangement and hence form a group. Which of them does not belong to that group?

(a)  NK

(b)  JL

(c)  ML

(d)  RU

(e)  ST

Answer: (c)

75. What is the position of U with respect to S?

(a)  Third to the right

(b)  Immediate left

(c)  Second to the left

(d)  Second to the right

(e)  Immediate right

Answer: (a)

76. Which of the following statements is true regarding N?

(a)  Only two persons sit between N and M.

(b)  N sits at an extreme end of the row.

(c)  N faces one of the immediate neighbours of R.

(d)  None of the given statements is true.

(e)  L sits to the immediate right of N.

Answer: (a)

77. Who amongst the following is facing T?

(a)  N

(b)  M

(c)  K

(d)  J

(e)  L

Answer: (e)

Directions (Q. Nos. 78-82) In these questions, relationship between different elements is shown in the statement (s). The statements are followed by conclusions. Study the conclusions based on the given statements and select the appropriate answer.

Give answer

(a) if only conclusion I follows

(b) if only conclusion II follows

(c) if either conclusion I or II follows

(d) if neither conclusion I nor II follows

(e) if both conclusion I and II follow

78. Statements P < L ≤ A = N ≥ E ≥ D; Q ≥ N < O

Conclusions I. L ≤ E        II. P < Q

Answer: (b)

79. Statements P ≤ U = N ≤ C ≥ H > S; K ≥ C

Conclusions I. P ≤ C        II. U < H

Answer: (a)

80. Statements P < L ≤ A = N ≥ E ≥ D; Q ≥ N < O

Conclusions I. Q ≥ D       II. A < D

Answer: (a)

81. Statements D ≥ I > S ≥ M ≤ A < L

Conclusions I. D ≥ A       II. L > I

Answer: (d)

82. Statements P ≤ U = N ≤ C ≥ H > S; K ≥ C

Conclusions I. K > U       II. U = K

Answer: (c)

Directions (Q. Nos. 83-87) Study the given information carefully to answer the given questions.

In certain code language,

‘committee to review papers’ is written as ‘es fr re pt’

‘review meeting in morning’ is written as ‘ch ba mo fr’

‘meeting to appoint members’ is written as ‘re dv ch gi’

‘appoint chairman in review’ is written as ‘mo gi fr yu’

(All the codes are two-letter codes only)

83. What is the code for ‘morning’ in the given code language?

(a)  mo

(b)  yu

(c)  ch

(d)  Other than those given as options

(e)  ba

Answer: (e)

84. In the given code language, what does the code ‘pt’ stands for?

(a)  appoint

(b)  Either ‘papers’ or ‘committee’

(c)  morning

(d)  review

(e)  Either ‘for’ or ‘members’

Answer: (b)

85. What is the code for ‘review call’ in the given code language?

(a)  dv lq

(b)  lq gi

(c)  lq fr

(d)  gi es

(e)  fr dv

Answer: (c)

86. What is the code for ‘to’ in the given code language?

(a)  mo

(b)  fr

(c)  gi

(d)  dv

(e)  re

Answer: (e)

87. If ‘appoint new members’ is coded as ‘dv wz gi’ in the given code language, then what is the code for ‘new chairman meeting’?

(a)  wz ch es

(b)  ch wz yu

(c)  yu mo wz

(d)  fre es wz

(e)  ch yu fr

Answer: (b)

Directions (Q. Nos. 88-92) Study the given information carefully to answer the given questions.

C, D, E, F, P, Q, R and S are sitting around a circular table facing the centre with equal distance between each other (but not necessarily in the same order). Each one of them is also related to D in some way or the other. P sits third to the right of E. D sits to the immediate left of E. Only one person sits between P and D’s son. R sits to the immediate right of D’s son. Only three people sit between D’s husband and R. Only one person sits between D’s husband and C. F sits to the immediate right of Q. D’s father sits second to the right of F. Only three people sit between D’s father and D’s brother. D’s daughter sits second to the right of S. D’s sister sits third to the right of D’s mother.

88. Which of the following statements is TRUE with respect to the given information?

(a)  S is the brother of R.

(b)  Only three people sit  between Q and S.

(c)  E sits third to the right of D’s daughter.

(d)  All the given options are true.

(e)  R is an immediate neighbour of D.

Answer: (b)

89. Who amongst the following is the brother of D?

(a)  Q

(b)  E

(c)  F

(d)  C

(e)  R

Answer: (d)

90. As per the given seating arrangement, Q : P in the same way as R : E. Then following the same pattern D : ?

(a)  R

(b)  S

(c)  Q

(d)  C

(e)  F

Answer: (c)

91. Who sits to the immediate left of D’s son?

(a)  D’s father

(b)  R

(c)  Q

(d)  S

(e)  D’s mother

Answer: (a)

92. How is Q related to P?

(a)  Sister

(b)  Sister-in-law

(c)  Niece

(d)  Brother-in-law

(e)  Aunt

Answer: (e)

Directions (Q. Nos. 93-95) Read the given information carefully and answer the given questions.

Each of the six buses R, S, T, U, V and W has different number of occupants. T has more number of occupants than R and S but less than V. U has less number of occupants than only W. S does not have the lowest number of occupants. The bus having second lowest number of occupants has 20 occupants and the bus having second highest number of occupants has 64 occupants. T has 21 less number of occupants than U.

93. Which of the following buses has third lowest number of occupants?

(a)  S

(b)  T

(c)  R

(d)  U

(e)  V

Answer: (b)

94. If the number of occupants of bus R is more than 7 and is an odd number which is divisible by 3 but not 5, how many occupants are there in bus R?

(a)  9

(b)  21

(c)  15

(d)  17

(e)  19

Answer: (a)

95. How many occupants does bus V possibly have?

(a)  43

(b)  72

(c)  20

(d)  46

(e)  68

Answer: (d)

Directions (Q. Nos. 96-100) Study the following information and answer the questions.

Seven people namely M, N, ,O, P, Q, R and S have an anniversary but not necessarily in the same order, in seven different months (of the same year) namely February, March, April, June, September, October and November, Each of them also likes a different flower namely Rose, Jasmine, Lily, Marigold, Daffodil, Sunflower and Orchid, but not necessarily in the same order.

R has an anniversary in the month which has more than 30 days. Only one person has an anniversary between R and the one who likes rose. Both S and O have an anniversary in one of the months after the one who likes Rose. S  has an anniversary immediately before O. The one who likes Lily has an anniversary in the month which has less than 30 days. Only three people have an anniversary between the one who likes Lily and the one who likes Orchid. Only two people have an anniversary between S and the one who likes Marigold. P has an anniversary immediately after the one who likes Marigold. Only two people have an anniversary between P and Q. M has an anniversary immediately before the one who likes Jasmine. O does not like Sunflower.

96. Which of the following represents the month in which S has an anniversary?

(a)  Cannot be determined

(b)  October

(c)  March

(d)  April

(e)  September

Answer: (b)

97. Which of the following does O like?

(a)  Rose

(b)  Jasmine

(c)  Marigold

(d)  Daffodil

(e)  Orchid

Answer: (d)

98. As per the given arrangement, Lily is a related to April and Marigold is related to September following a certain pattern, which of the following is Orchid related to following the same pattern?

(a)  February

(b)  June

(c)  October

(d)  November

(e)  March

Answer: (d)

99. Which of the following represents the people who have an anniversary in April and November respectively?

(a)  N, M

(b)  Q, M

(c)  Q, O

(d)  N, O

(e)  N, S

Answer: (d)

100. How many people have an anniversary between the months in which Q and M have an anniversary?

(a)  None

(b)  One

(c)  Three

(d)  Two

(e)  More than three

Answer: (c)

IBPS CWE-V Clerical Cadre Pre. Online Examination Held on December 5, 2015 Question Paper With Answer Key

IBPS CWE-V Clerical Cadre Pre. Online Examination Held on December 5, 2015
IBPS CWE-V Clerical Cadre Pre. Online Examination Held on December 5, 2015 Question Paper With Answer Key

IBPS CWE-V Clerical Cadre Pre. Online Examination Held on December 5, 2015

Part I English Language

 

Directions (Q. Nos. 1-5) Rearrange the following five sentences A, B, C, D and E in a proper sequence to form a meaningful paragraph, then answer the given questions.

(A) The big orange truck was playing music and had bells that chimed repeatedly.

(B) When Lisa approached the truck, the man inside it said, “Here you go, young lady, enjoy our ice-cream sandwich.”

(C) Lisa was amazed and said “What a brilliant idea this is; don’t wait for your customers to come to you-go out and find them!”

(D) Several people attracted by the bells were walking up to it and speaking to a man who stood inside it.

(E) One day, Lisa walked to the front of her house and was surprised to see a big orange truck on the street.

1. Which of the following should be the SECOND sentence after the rearrangement?

(a)  A

(b)  B

(c)  C

(d)  D

(e)  E

Answer: (a)

2. Which of the following should be the FIRST sentence after the rearrangement?

(a)  A

(b)  B

(c)  E

(d)  D

(e)  C

Answer: (c)

3. Which of the following should be the FOURTH sentence after the rearrangement?

(a)  A

(b)  B

(c)  C

(d)  E

(e)  D

Answer: (b)

4. Which of the following should be the THIRD sentence after the rearrangement?

(a)  A

(b)  B

(c)  C

(d)  D

(e)  E

Answer: (d)

5. Which of the following should be the LAST(FIFTH) sentence after the rearrangement?

(a)  A

(b)  E

(c)  C

(d)  D

(e)  B

Answer: (c)

Directions (Q. Nos. 6-15) Read the following passage carefully and answer the questions given below it. Certain words/phrases have been given in bold to help you locate them while answering some of the questions.

A long time ago, there lived a gardener. He was hardworking and honest. He had planted different types of flowers and vegetables in his garden. He had a younger brother, who was foolish and lazy. He would never help the gardener in his work. Instead, he would way, “There is no need to work so hard. Brother! Nature will take care of itself.”

Once it so happened that the gardener had to go to a nearby town for a week. ‘Please water the plants and trees in my absence!’ said the gardener to his younger brother. ‘Some plants need to be watered daily while some others only on alternate days.’ the gardener explained. But  his words fell on deaf ears. The gardener left for the town. Now the lazy brother thought, ‘Why should I brother about these plants and trees? My brother is away and he wouldn’t know whether I am watering his garden or not.’ Then, he went to sleep as usual.

Two days had passed. Now the small plants had started dying up. ‘The lazy fellow glanced at them and laughed. ‘Hahaha! Poor dumb plants! They cannot even complain to my brother that I am not watering them!’ After a week, when the gardener returned, he was shocked to see the miserable condition of his garden. Most of the plants had died, the climbers had dried and the trees locked diseased. ‘So it seems you didn’t water the plants’, he yelled at his lazy brother. ‘I did’. But the plants were of bad quality, so they died!’ said the lazy brother.

The gardener knew that it was of no use talking to his brother. So he cleaned his garden and nursed the diseased trees. He then went to the market and brought some seeds. When his brother saw him preparing to sow the seeds, he came near him. ‘And that’s it. Brother! Don’t I tell you that nature takes care of itself? See those seeds! You will sow them and once again your garden will be ready as green as ever.’ ‘You are right,’ said the gardener. ‘But have you ever thought about where these seeds come from?’ ‘From the market, of course!’ replied his brother.’ ‘And how did they reach the market?’ the gardener asked again. This time his brother had no reply, ‘E;; uuup uhm!’ he kept searching for words. ‘I’ E tell you!’ said the gardener, ‘The seeds also come from the  plants and trees. If we don’t take  proper care of the plants, the seeds too will die. And a day will come when there will be no new seeds to sow. How will you grow plants then? And how will you get vegetables and fruits? Just think! Won’t you die of hunger?’

‘Oh, I never thought that way!’ the gardener’s brother shrieked. ‘It’s true that nature takes care of itself. And it cares for us too. That’s why it has given us seeds. A small contains the biggest secret of nature. All that we need to do is to unfold that secret. It has no much power in it. It can make a beautiful tree, with flowers and fruits and more seeds.’ explained the gardener. Now his brother realized his mistake. From that day on, he was not lazy anymore. He started helping his brother in nurturing the plants and trees. He was now well aware of the secret of nature.

6. Which of the following statements is true in the context of the story?

(a)  The gardener’s brother was two years elder to him.

(b)  Before leaving for the town, the gardener knew his brother would fail to complete the task assigned to him.

(c)  The gardener’s brother was jealous of him since childhood.

(d)  The gardener did not explain how to take care of his plants because of which his plants died.

(e)  None of the given statements is true.

Answer: (e)

7. Choose the word which is most nearly the opposite in meaning to the word ‘Aware’ was used in the passage?

(a)  Absent

(b)  Short

(c)  Ignorant

(d)  Knowing

(e)  Agree

Answer: (c)

8. Which of the following correctly explains the meaning of the phrase ‘fell on deaf ears’ as used in the story?

(a)  were heard and accepted

(b)  were ignored

(c)  fell head first

(d)  were deafeningly loud

(e)  were too soft

Answer: (b)

9. As mentioned in the story, the gardener had to go to a town nearby because

(a)  he wanted to get a different variety of seeds

(b)  he wished to visit one of his old friends

(c)  his parents were unwell and he went to visit them

(d)  he wanted to build a house for  his brother

(e)  Not clearly mentioned in the story

Answer: (e)

10. Which of the following is most nearly the opposite in meaning to the word ‘Nurturing’ as used in the passage?

(a)  Starving

(b)  Believing

(c)  Supplying

(d)  Dieting

(e)  Watering

Answer: (a)

11. Which of the following is most nearly the same in meaning to the word ‘Miserable’ as used in the passage?

(a)  Bankrupt

(b)  Cherished

(c)  Poor

(d)  Denial

(e)  Scanty

Answer: (c)

12. As mentioned in the story, the gardener’s brother discouraged his brother from taking care of his garden because

(A) he did not want his brother to struggle so much

(B) he was planning to hire a servant for his brother

(C) for him, it was a futile effort

(a)  Only A

(b)  A and B

(c)  Only B

(d)  Only C

(e)  B and C

Answer: (c)

13. Which of the following characteristics of the gardener’s brother comes across distinctly at the end of story?

(a)  He started respective his parents.

(b)  He became violent.

(c)  His hatred towards his brother amplified.

(d)  He became depressed.

(e)  He became sensitive towards nature.

Answer: (e)

14. Which of the following is mot nearly the same in meaning to the word ‘Brother’ as used in the passage?

(a)  Worry

(b)  Content

(c)  Apprehend

(d)  Doubt

(e)  Kin

Answer: (a)

15. Which of the following can be an appropriate title for the story?

(a)  Too Late to Repair the Damage

(b)  The Jealous Brother

(c)  The Idle Gardener

(d)  The Secret of Nature

(e)  The Fate of Disappearing Plants

Answer: (d)

Directions (Q. Nos. 16-20) Read each sentence to find out whether there is any grammatical error in it. The error, if any, will be in one part. The letter of that part is your answer. If there is no error then answer is (e). (Ignore the errors of punctuation, if any)

16. There was/ once a gardener/ who took care/ of the king’s garden.

(a)  There was

(b)  once a gardener

(c)  who took care

(d)  of the king’s garden.

(e)  No error

Answer: (b)

17. By and by, the fox/ comes to a stream/ that ran through the forest/ and quenched his thirst.

(a)  By and by, the fox

(b)  comes to a stream

(c)  that ran through the forest

(d)  and quenched his thirst

(e)  No error

Answer: (b)

18. The giant entered/ the forest at night/ when all the/ animals were asleep.

(a)  The giant entered

(b)  the forest at night

(c)  when all the

(d)  animals were asleep

(e)  No error

Answer: (b)

19. The ant realize that/ a pigeon sitting on the tree/ had dropped a leaf/ into the water to save him.

(a)  The ant realize that

(b)  a pigeon sitting on the tree

(c)  / had dropped a leaf

(d)  into the water to save him

(e)  No error

Answer: (a)

20. While trying to pick up/ the few fallen ones, the monkey dropped almost/ all the fruits encased his hands.

(a)  While trying to pick up

(b)  the few fallen ones

(c)  the monkey dropped almost

(d)  all the fruits encased his hands

(e)  No error

Answer: (d)

Directions (Q. Nos. 21-25) In each of the following questions, a sentence with four words in bold type is given. One of these four words given in bold may be either wrongly spelt or inappropriate in the context of the sentence.  Find out the word which is wrongly spelt or inappropriate. If all the words given in bold are correctly spelt and also appropriate in the context of the sentence, mark ‘All correct’ as your answer.

21. Leaders and managers must communicate well with employees, customers, investors and society as a whole.

(a)  managers

(b)  communicate

(c)  employees

(d)  whole

(e)  All correct

Answer: (a)

22. The government will push through economic reforms once the knew financial year begins.

(a)  push

(b)  knew

(c)  financial year

(d)  begins

(e)  All correct

Answer: (b)

23. Committees have been asked to dispoze of all grievance petitions within the next fortnight.

(a)  dispoze

(b)  petitions

(c)  within

(d)  fortnight

(e)  All correct

Answer: (a)

24. Generally gift cards do not allow a cash refund and have a validitie period.

(a)  Generally

(b)  allow

(c)  validite

(d)  period

(e)  All correct

Answer: (c)

25. Printing money to boost the economy is a threat to financial stablety.

(a)  Printing

(b)  threat

(c)  financial

(d)  stablety

(e)  All correct

Answer: (d)

Directions (Q. Nos. 26-30) In the given passage, there are blanks each of which has been numbered. Against each, five words are suggested, one of which fits the blank appropriately. Find out the appropriate word in each case.

Once upon a time there lived a spider named Anansi who gifted a pot of wisdom to share among all the creatures of the world. But Anansi was unwilling to share it with anybody and decided to (26) the pot at the top of a tall tree. With the pot at his waist in front, it was (27) for Anansi to climb. Seeing his father’s awkward ascent, Anansi’s son was (28) and advised him ‘Father, why don’t you carry the pot on your back (29)?’ Anansi did so and found it easy to scale the tree. He (30) that though he had a pot of wisdom he lacked common sense. Angry at himself he threw the pot down where it broke into pieces and wisdom scattered among all the creatures.

26

(a)  disappear

(b)  vanish

(c)  hide

(d)  observe

(e)  unearth

Answer: (c)

27.

(a)  tired

(b)  difficult

(c)  challenge

(d)  risk

(e)  safely

Answer: (b)

28.

(a)  laughing

(b)  worrying

(c)  anxiously

(d)  busy

(e)  amused

Answer: (e)

29.

(a)  tied

(b)  alternative

(c)  subsequent

(d)  instead

(e)  perhaps

Answer: (d)

30.

(a)  realized

(b)  achieved

(c)  recognize

(d)  aware

(e)  understand

Answer: (a)

Part II Quantitative Aptitude

31. Six years from now, the average of Monu’s age that time and Ninu’s age that time will be 29 years. Five years ago, if the respective ratio between Monu’s age and Ninu’s age that time was 11 : 7, what is Ninu’s present age?

(a)  17 years

(b)  33 years

(c)  27 years

(d)  19 years

(e)  22 years

Answer: (d)

32. Rahim spends 60% of his monthly salary on rent, EMI and miscellaneous expenses in the respective ratio of 2 : 3 : 3. If he spends a total of Rs 16050 on the rent and EMI together, how much is his monthly salary?

(a)  Rs 50300

(b)  Rs 49600

(c)  Rs 46750

(d)  Rs 48200

(e)  Rs 42800

Answer: (e)

33. There are seven positive observations. Average of the first four observations is 11 and of the last four observations is 8. If the average of these seven observations is 9, what is the fourth observation?

(a)  13

(b)  18

(c)  17

(d)  12

(e)  23

Answer: (a)

34. Prem invested certain sum in Scheme A, which offers simple interest at 8% per annum for 4 years. He also invested Rs 2000 in Scheme B, which offers compound interest (compounded annually) at 20% per annum for 2 years. If the interest earned from Scheme A is 17/11th of the interest from scheme B, what is the sum invested in Scheme A?

(a)  Rs 4000

(b)  Rs 3000

(c)  Rs 4500

(d)  Rs 4250

(e)  Rs 5000

Answer: (d)

35. Two train started running from the same point at the same time in opposite directions (one towards North and other towards South). The speed of the two trains is 22 m/s and 8 m/s respectively. How much time will they take to be 378 km apart?

(a)  3 h 30 min

(b)  3 h 20 min

(c)  4 h 30 min

(d)  2 h 15 min

(e)  3 h 45 min

Answer: (a)

36. Two-seventh of a number is two less than half of another number. If the sum of the two numbers is 15, what is their product?

(a)  72

(b)  64

(c)  54

(d)  63

(e)  56

Answer: (e)

Directions (Q. Nos. 37-46) What will come in place of questions marks in the given questions?

37. 

(a)  4

(b)  2

(c)  5

(d)  6

(e)  3

Answer: (e)

38. 

(a)  290

(b)  210

(c)  270

(d)  230

(e)  250

Answer: (d)

39. (0.6 × 450) ÷ 5 = 2 × 3?

(a)  3

(b)  −3

(c)  −2

(d)  −1

(e)  2

Answer: (a)

40. 

(a)  10

(b)  9

(c)  8

(d)  7

(e)  6

Answer: (a)

41. 

(a)  10

(b)  9

(c)  8

(d)  7

(e)  6

Answer: (a)

42. 

(a)  4

(b)  6

(c)  3

(d)  5

(e)  9

Answer: (e)

43. 45% of 360 + 288 = ? % of 750

(a)  52

(b)  60

(c)  70

(d)  75

(e)  65

Answer: (a)

44. 

(a)  56

(b)  66

(c)  64

(d)  91

(e)  74

Answer: (e)

45. 35% of 580 + 70% of ? = 441

(a)  380

(b)  340

(c)  360

(d)  320

(e)  280

Answer: (b)

46. 66 × 40 ÷8 + ? = 1480

(a)  670

(b)  930

(c)  650

(d)  760

(e)  830

Answer: (b)

47. The selling price of 16 chairs is equal to the selling price of 6 tables. If the total selling price of 5 chairs and 3 tables together is Rs 730, then what is the total selling price of 2 chairs and 1 table together?

(a)  Rs 262

(b)  Rs 460

(c)  Rs 362

(d)  Rs 450

(e)  Rs 300

Answer: (a)

48. The sum of two positive numbers is 630. If 75% of the first number is equal to 60% of the second number,, what is the larger number among the two?

(a)  230

(b)  350

(c)  280

(d)  375

(e)  270

Answer: (b)

49. A boat an travel 10.2 km upstream in 51 minutes. If the speed of the water current is 1/5th of the speed of the boat in still water, then how much distance the boat can travel downstream in 48 minutes?

(a)  14.8 km

(b)  15.6 km

(c)  15.2 km

(d)  17.4 km

(e)  14.4 km

Answer: (e)

50. A started a business by investing Rs 4200, and after 2 months B joined by investing Rs 3000. At the end of 4 months from the start of the business, C joined with an investment of Rs 6000. At the end of 10 months from the start of the business, A added an additional amount of Rs 1800. If A’s share in the annual profit was Rs 1620, what was the total annual profit?

(a)  Rs 3960

(b)  Rs 3080

(c)  Rs 4070

(d)  Rs 3320

(e)  Rs 4180

Answer: (a)

51. A and B can independently finish a piece of work in 36 and 45 days respectively. They started working together and after few days A left. After that B could finish the remaining work in 36 days. After how many days of working together A left?

(a)  4

(b)  6

(c)  12

(d)  5

(e)  8

Answer: (a)

52. The perimeter of an equilateral triangle is 21 m more than perimeter of a square. If the respective ratio between the side of the triangle and the side of the square is 9 : 5, what is the area of the square?

(a)  25 m2

(b)  225 m2

(c)  625 m2

(d)  144 m2

(e)  81 m2

Answer: (b)

53. A circular copper wire of radius 35 cm is bent to form a rectangle. If the length of the rectangle is more than the breadth of the rectangle by 26 cm, then what is the length of the rectangle?

(a)  72 cm

(b)  64 cm

(c)  76 cm

(d)  62 cm

(e)  68 cm

Answer: (e)

54. In what respective ratio, rice of variety A worth Rs 36 must be mixed with rice of variety B worth Rs 48, so that the new mixture (of both the varieties A and B) is worth Rs 45?

(a)  1 : 3

(b)  3 : 4

(c)  4 : 5

(d)  1 : 2

(e)  3 : 2

Answer: (a)

Directions (Q. Nos. 55-59) What will come in place of question marks in the given number series?

55. 41    42      45      54      81      ?

(a)  194

(b)  162

(c)  134

(d)  146

(e)  112

Answer: (b)

56. 350    334    366    318    ?        302

(a)  382

(b)  395

(c)  376

(d)  354

(e)  422

Answer: (a)

57. 164   84      44      24      14      ?

(a)  7

(b)  4

(c)  5

(d)  9

(e)  6

Answer: (d)

58. 12    ?        6        9        18      45

(a)  5

(b)  6

(c)  2

(d)  4

(e)  3

Answer: (b)

59. 5    4        7        20      79      ?

(a)  288

(b)  394

(c)  265

(d)  256

(e)  192

Answer: (b)

60. The profit earned by selling an article for Rs 590 is double was incurred when the same article is sold for Rs 245. What would be the selling price of the article, if it is sold at 20% profit?

(a)  Rs 432

(b)  Rs 436

(c)  Rs 424

(d)  Rs 415

(e)  Rs 445

Answer: (a)

Directions (Q. Nos. 61-65) Study the table and answer the given questions

61. What is the difference between total number of members in health clubs A and B together in 2007 and that in the health clubs D and E together in 2011?

(a)  131

(b)  123

(c)  121

(d)  127

(e)  133

Answer: (c)

62. If 44% of the total number of members in health clubs A, B and C together in 2008 are males, what is the total number of male members in health clubs A, B and C together in 2008?

(a)  155

(b)  170

(c)  160

(d)  165

(e)  175

Answer: (d)

63. Number of members in health club A increased by what percent from 2009 to 2011?

(a)  55%

(b)  80%

(c)  65%

(d)  50%

(e)  75%

Answer: (e)

64. What is the average number of members in health clubs B, C and E in 2010?

(a)  132

(b)  126

(c)  122

(d)  124

(e)  128

Answer: (a)

65. What is the respective ratio between total number of members in health club C in 2007 and 2009 together and that in health club E in the same years together?

(a)  17 : 23

(b)  19 : 23

(c)  19 : 21

(d)  21 : 23

(e)  17 : 21

Answer: (a)

Part III Reasoning Ability

Directions (Q. Nos. 66-70) In the following questions, a group of number/symbol followed by five combinations of inter codes is given. You have to find out which of the combinations correctly represents the group of number/symbol based on the given coding system and the conditions and mark that combination as your answer.

Conditions

(i) If the first and the last elements are symbols then their codes are to the interchanged.

(ii) If a symbol is immediately followed as well as immediately preceded by a number then that symbol is to be coded as ‘1’.

(iii) If the last element is an odd number then the second element is to be coded as the code of the odd number.

(iv) If the third element is an even number then the code of that even number is to be interchanged with the code of first element.

(Note All the elements have to be counted from left to right to fulfill the conditions)

66. #@3+42

(a)  ZDKTHE

(b)  ZDXUHE

(c)  ZKE1DH

(d)  EDK1HZ

(e)  EDUXHZ

Answer: (b)

67. 8^@537

(a)  LMDAXM

(b)  LTMAKM

(c)  DTLAKM

(d)  LMKDAM

(e)  MTDAXL

Answer: (a)

68. @32&#8

(a)  EXDPZL

(b)  EZDPXL

(c)  XEDZ1L

(d)  XLEZPL

(e)  ZLEXPL

Answer: (a)

69. @98&^δ

(a)  LYDPTK

(b)  DY1PTK

(c)  KYLPTD

(d)  DYL1TK

(e)  LPDYTK

Answer: (a)

70. $452#%

(a)  GH1EZB

(b)  GHA1EZ

(c)  BHZEAG

(d)  BHAEZG

(e)  AHGEZB

Answer: (d)

71. In a certain code language, ‘avoid going out’ is coded as ‘319’. Similarly ‘going for party’ is coded as ‘612’ and ‘out of party’ is coded as ‘362’. What will be the code for p arty in the given code language?

(a)  3

(b)  1

(c)  9

(d)  Either 3 or 1

(e)  Either 2 or 6

Answer: (e)

Directions (Q. Nos. 72-76) Study the following information carefully and answer the given questions.

S, T, U, V, W, X, Y and Z are sitting around a circular table facing the centre but not necessarily in the same order. S sits third to the right of T. Only three people sit between U and I. U is neither an immediate neighbour of S nor T. Only three people sit between T and W. Z sits to the immediate right of W. V sits third to the left of W.

72. How many people are seated between S and Z, when counted from the right of S?

(a)  None

(b)  Two

(c)  Four

(d)  Three

(e)  One

Answer: (e)

73. Who amongst the following represent the immediate neighbour of S?

(a)  X, Y

(b)  X, Z

(c)  W, Y

(d)  W, X

(e)  V, Y

Answer: (c)

74. Who among the following sits second to the left of Y?

(a)  T

(b)  X

(c)  W

(d)  Z

(e)  S

Answer: (a)

75. Four of the following five are alike in a certain way based on their positions in the given arrangement and so form a group. Which is the one that does not belong to the group?

(a)  UYV

(b)  TVY

(c)  SWU

(d)  XSZ

(e)  VXW

Answer: (b)

76. Which amongst the following is true regarding V, as per the given arrangement?

(a)  None of the given statements is true.

(b)  Only three people sit between V and S.

(c)  U sits second to the right of V.

(d)  V sits second to the right of Y.

(e)  Only two people sit between V and Z.

Answer: (a)

Directions (Q. Nos. 77-78) Study the given information carefully to answer the given questions.

Five cars P, Q, R, S and T each have a different mileage. P has more mileage than only one car. Both Q and T have more mileage than P but less than S. Q has more mileage than R but less than T. The car which has the highest mileage runs for 22 km/L. The car which has the third lowest mileage runs for 19 km/L.

77. Which amongst the following cars has the highest mileage?

(a)  S

(b)  Q

(c)  Cannot  be determined

(d)  T

(e)  R

Answer: (a)

78. If the difference between the mileage of R and Q is of 4 km/L, then what is the mileage of R?

(a)  23 km/L

(b)  21 km/L

(c)  18 km/L

(d)  15 km/L

(e)  12 km/L

Answer: (d)

79. Which of the given statements will be definitely true if the expression ‘V < E > B = H ≥ N ≥ P’ is definitely true?

(a)  P < B

(b)  H < V

(c)  N ≥ V

(d)  E ≥ N

(e)  E ≥ P

Answer: (a)

Directions (Q. Nos. 80-84) In each of the following questions assuming the given statements to be true, find out which of the two conclusions I and II given below them is/are definitely true.

Given answer

(a) if only conclusion I is true

(b) if only conclusion II is true

(c) if either conclusion I or II is true

(d) if neither conclusion I nor II is true

(e) if both conclusions are true

80. Statement Q ≤ U < I ≤  C = K

Conclusions I. U > K       II. Q < C

Answer: (b)

81. Statement T = A < X ≤ S; E < X

Conclusions I. T < S        II. T > E

Answer: (a)

82. Statement H ≥ Y ≥ P = E > R

Conclusions I. E ≤ H        II. Y > R

Answer: (e)

83. Statement J = U ≤ N  ≤ E

Conclusions I. J < E                  II. E = J

Answer: (c)

84. Statement D < O ≥ N > K ≤ Y

Conclusions I. D > K       II. Y < O

Answer: (d)

85. If ‘2’ is added to the first digit of every odd number and ‘3’ is subtracted from the third digit of every even number, then in how many numbers will a digit appear twice?

(a)  Three

(b)  One

(c)  Two

(d)  None

(e)  Four

Answer: (c)

86. If all the numbers are arranged in ascending order from left to right, which of the following will be sum of all the three digits of the number which is third from the right?

(a)  13

(b)  16

(c)  15

(d)  14

(e)  19

Answer: (c)

87. What will be the resultant if the third digit of the lowest number and second digit of the highest number are multiplied?

(a)  42

(b)  24

(c)  10

(d)  35

(e)  20

Answer: (d)

88. The position of the first and third digits of each of the numbers are interchanged. What will be resultant if the first digit of lowest number thus formed is divided by the second digit of the  highest number thus formed?

(a)  1

(b)  2

(c)  1.5

(d)  2.5

(e)  3

Answer: (b)

89. If in each number all the digits are arranged in descending order from left to right within the number, then how many numbers thus formed will be odd numbers?

(a)  None

(b)  Three

(c)  Four

(d)  One

(e)  Two

Answer: (d)

Directions (Q. Nos. 90-91) Study the following information carefully and answer the given question.

P is the father of D. D is the only son of T. T is the daughter of J. T is the mother of G. G is the sister of V.

90. If J is married to B, then how is B related to G?

(a)  Daughter-in-law

(b)  Son-in-law

(c)  Father

(d)  Mother

(e)  Cannot be determined

Answer: (e)

91. How is V related to P?

(a)  Daughter

(b)  Father

(c)  Mother

(d)  Cannot be determined

(e)  Son

Answer: (a)

92. How many such pairs of letters are there in the word TIMELY each of which has as many letters between them in the word (in both forward and backward directions) as they have between them in the English alphabetical series?

(a)  Two

(b)  None

(c)  Three

(d)  One

(e)  More than three

Answer: (a)

93. If all the numbers in 86312749 are arranged in ascending order from left to right, the positions of how many numbers will remain unchanged?

(a)  Three

(b)  One

(c)  Two

(d)  More than three

(e)  None

Answer: (a)

Directions (Q. Nos. 94-98) Study the following information to answer the given questions.

G, H, I, J, S T and U are seated in a straight line facing North, with equal distance between each other but not necessarily in the same order.

I sits second to the right of T. U sits third to the right of I. H sits third to the left of S. S does not sit at any of the extreme ends of the line. Only one person sits between S and G.

94. Who amongst the following sits exactly in the middle of the line?

(a)  G

(b)  J

(c)  U

(d)  I

(e)  S

Answer: (b)

95. How many persons are seated between I and S?

(a)  One

(b)  Two

(c)  Four

(d)  None

(e)  Three

Answer: (a)

96. Which of the following represents persons seated at the two extreme ends of the line?

(a)  G, U

(b)  T, G

(c)  G, J

(d)  I, J

(e)  H, I

Answer: (b)

97. What is the position of U with respect to G?

(a)  Second to right

(b)  Immediate left

(c)  Fourth to the left

(d)  Fifth to the right

(e)  Third to the left

Answer: (b)

98. Who amongst the following sits to the immediate left of H?

(a)  J

(b)  G

(c)  I

(d)  U

(e)  T

Answer: (e)

99. In a certain code language, BLUNT is coded as ‘AKSOU’ and COINS is coded as ‘BNGOT’. In the same code language, how will TRUST be coded as?

(a)  SSTUV

(b)  SQTRS

(c)  RQSRS

(d)  SQSTU

(e)  RSSQN

Answer: (d)

100. All the letters of the word HALFTIME arranged in alphabetical order from left to right. Then all the vowels are replaced with the next alphabet (as per the English alphabetical order), then which of the following will be second letter from the left end?

(a)  H

(b)  F

(c)  M

(d)  L

(e)  E

Answer: (b)

IBPS CRP-V Specialist Officer (Marketing) Examination-2015 Online Examination Held on February 2, 2016 Question Paper With Answer Key

IBPS CRP-V Specialist Officer (Marketing) Examination-2015 Online Examination Held on February 2, 2016
IBPS CRP-V Specialist Officer (Marketing) Examination-2015 Online Examination Held on February 2, 2016 Question Paper With Answer Key

IBPS CRP-V Specialist Officer (Marketing) Examination-2015 Online Examination Held on February 2, 2016

Part I Reasoning

Directions (Q. Nos. 1-2) The following questions consist of a equation and two statements numbered I and II given below it. You have to decide whether the data given in the statements are sufficient to answer the questions. Read both the statements and

Give answer

(a) if the data in statement I alone are sufficient to answer the question, while the data in statement II alone are not sufficient to answer the question

(b) if the data in statement II alone are sufficient to answer the question, while the data in statement I alone are not sufficient to answer the question

(c) if the data either in statement I alone or in statement II alone are sufficient to answer the question

(d) if the data even in both statements together are not sufficient to answer the question.

(e) if the data in both statements together are necessary to answer the question

1. How many people are standing in straight line (Note : All are facing North)?

(I)  Q stands third from the right end of the line. only one persons stands between Q and S. S stands at the extreme left end of the line.

(II) Q stands exactly in the centre of the line. P and M are immediate neighbours of Q. Only one person stands to the left of P.

Answer: (e)

2. How far is point M from point S?

(I)  Point S is 7 m to the South of point M. Point T is 4 m to the East of point S. Point R is 4 m to the North of point T. Point Q is to the West of point P. Point R is 4 m to the North of point Q.

(II) Point R is 4 m to the West of point Q. Point P is 4 m to the North of point Q. Point X is 4 m to the North of point R. Point M is 6 m to the East of point X.

Answer: (a)

Directions (Q. Nos. 3-7) In each of the questions below two/three statements are given followed by two conclusions I and II. You have to take the given statements to be true even if they seem to be at variance from commonly known facts and then decide which of the given conclusion logically follows from the statements disregarding commonly known facts.

Give answer

(a) if only conclusion I follows

(b) if only conclusion II follows

(c) if either conclusion I or II follows

(d) if neither conclusion I nor II follows

(e) if both conclusions follow

3. Statements All crafts are projects.

All projects are missions.

No mission is a guide.

Conclusion I. Some projects are guides.

II. No project is a guide.

Answer: (b)

4. Statements Some outputs are results.

All outputs are products.

All products are yields.

Conclusions I. No product is result.

II. All yields are products.

Answer: (d)

5. Statements Some outputs are results.

All outputs are products.

All products are yields.

Conclusions I. All outputs are yields.

II. All results being yields is a possibility.

Answer: (a)

6. Statements No price is a rate.

All rates are expenses.

Conclusions I. No expense is a price.

II. All prices being expenses is a possibility.

Answer: (a)

7. Statements All crafts are projects.

All projects are  missions

No mission is a guide.

Conclusions I. No guide is a craft.

II. At least some missions are crafts.

Answer: (e)

Directions (Q. Nos. 8-12) Study the following information carefully and answer the questions given.

Eight friends P, Q, R, S, T, U, V and W are sitting around a square table in such a way that four to them sit at four corners while four sit in the m idle of each of the four sides. The ones who sit at the four corners face the centre while those who sit in the middle of the sides face outside. (i.e., opposite to the centre).

• V sits second to the right of R. R sits in the middle of one of the sides of the table.

• Only two people sit between V and Q. S is one of the immediate neighbours of Q.

• T sits second to the left of S.

• P sits second to the left of U.

• V is not an immediate neighbour of U.

8. How many people sit between R and T when counted from the right of R?

(a)  None

(b)  Four

(c)  One

(d)  Three

(e)  Two

Answer: (d)

9. Which of the following is true regarding P?

(a)  Both T and R are immediate neighbours of P.

(b)  Only three people sit between P and S.

(c)  P sits at middle of one of the sides.

(d)  W sits second to the left of P.

(e)  None of the given options is true.

Answer: (e)

10. What is the positions of V with respect to Q?

(a)  Second to the left

(b)  Third to the left

(c)  Second to the right

(d)  Fifth to the right

(e)  Fifth to the left

Answer: (b)

11. Four of the following five are alike in a certain way and so form a group. Which is the one that does not belong to that group?

(a)  Q

(b)  T

(c)  S

(d)  R

(e)  V

Answer: (a)

12. Who sits second to the left of W?

(a)  P

(b)  U

(c)  V

(d)  S

(e)  Q

Answer: (a)

Directions (Q. Nos. 13-17) In these questions, relationship between different elements is shown in the statements. The statements are followed by conclusions. Study the conclusions based on the given statement and select the appropriate answer.

Give answer

(a) if only conclusion I is true

(b) if neither conclusion I nor II is true

(c) if both conclusions are true

(d) if only conclusion II is true

(e) if either conclusion I or II is true

13. Statement K ≥ S = U ≤ R; L < U ≤ J

Conclusions I. R > J        II. L = K

Answer: (a)

14. Statements D > W ≥ C ≤ X; C  ≥ L; W < K

Conclusions I. X > K       II. L ≤ W

Answer: (c)

15. Statements R ≤ A < M ≥ T  ≥ V; M ≤ S

Conclusions I. S > R        II. Y ≤ S

Answer: (a)

16. Statements D > W ≥ C ≤ X; C ≥ L; W < K

Conclusions I. D > L        II. L > D

Answer: (e)

17. Statements S > B = K ≥ L; B = P ≤ U

Conclusions I. L < U        II. U = L

Answer: (e)

18. Study the following information and answer the question.

In a recent performance approval done by company X, more than 70% of the employees were found to be under-performing.

“I think that the restricting done by the company in the previous year is responsible for the under-performance of the employees” – HR Manager f company X.

Which of the following statements weakens the statement of HR Manager of company X?

(a)  The incentives linked to performance were abolished upon restricting creating dissatisfaction among employees.

(b)  After restricting, the decision making  power has taken away from employees thus delaying the work by long hours.

(c)  The number of projects in company X increased by 60% this year thereby increasing the burden on the existing employees this year.

(d)  After restricting, the employees were required to report to multiple bosses leading to ambiguity at the work place.

(e)  Adequate training was not provided to the employees transferred to different departments and locations after restructuring.

Answer: (e)

Directions (Q. Nos. 19-23) Study the following information to answer the given questions.

Twelve people are sitting in two parallel rows containing six people each in such a way that there is an equal distance between adjacent persons. In row-1 J, K, L, M, N and O are seated (but not necessarily in the same order) and all of them are facing South. In row-2 U, V, W, X, Y and Z are seated (but not necessarily in the same order) and all of them are facing North. Therefore, in the given seating arrangement, each member seated in a row faces another member of the other row.

• M sits fourth to the left of J. The one facing J sits third to the left of Y.

• Only one person sits between Y and U. U does not sit at any of the extreme ends of the line.

• The one facing Z sits second to the right of K. Z does not sit at any of the extreme ends of the line.

• Only two people sit between K and O.

• The one facing K sits second to the left of X.

• V is not an immediate neighbour of Z. L is not immediate neighbour of M.

19. Which of the following groups of people represents the people sitting extreme ends of both the rows?

(a)  M, O, X, W

(b)  M, K, V, W

(c)  N, K, V, Y

(d)  J, N, U, V

(e)  J, O, Z, X

Answer: (a)

20. Who amongst the following faces V?

(a)  M

(b)  L

(c)  J

(d)  N

(e)  K

Answer: (e)

21. Which of the following is true with respect to the given information?

(a)  K faces one of the immediate neighbours of X.

(b)  V sits exactly between W and U.

(c)  None of the given options is true.

(d)  J is an immediate neighbour of K.

(e)  J faces Z.

Answer: (e)

22. Which of the following is true regarding N?

(a)  K sits second to right of N.

(b)  V is an immediate neighbour of the person who faces N.

(c)  Both L and O are immediate neighbours of N.

(d)  Only one person sits between N and J.

(e)  None of the given options is true.

Answer: (b)

23. Who amongst the following sits second to the right of the person who faces L?

(a)  V

(b)  Z

(c)  W

(d)  U

(e)  Y

Answer: (e)

24. If ‘4’ is subtracted from each digit and ‘1’ is added to each digit in the number 9476582, which of the following numbers will appear twice in the new number thus formed?

(a)  Only 3

(b)  1 and 7

(c)  3 and 5

(d)  None

(e)  Only 1

Answer: (a)

25. Read the given information and answer the question.

The government of State D was recently criticised for accepting a proposal as per which schools in the State will not conduct exams up to standard nine. Rather, schools will promote all the students up to standard nine and from standard nine onwards the students would either be passed or failed depending upon their marks in the exams.

Which of the following may not be a reason for the criticism of the government of State D for accepting the proposal?

(a)  Several parents have raised concern that students would not be willing to pay attention in class and retain the information because they do not have to write exams.

(b)  Students learn to write exam papers in a stipulated period of time with practice of solving exam papers over the years and many students may not be able to grasp this directly in the standard nine.

(c)  Many teachers are of the opinion that students have become very competitive and are mainly focused on acquiring marks rather than gaining knowledge.

(d)  Studies suggest that students become less prone to exam anxiety and exam fear when they get used to passing exams over the years as compared to when they do not appear for an exam at all.

(e)  Experts suggests that the special needs of children can be adequately assessed through education of their exam papers and early assessment helps initiate early intervention and major projects through exams.

Answer: (e)

Directions (Q. Nos. 26-31) Study the following information and answer the questions.

Seven friends, namely P, Q, R, S, T, U and V visit seven different countries namely Japan, Germany, China, India, Nepal, Australia and Malaysia, not necessarily in the same order, starting from Monday to Sunday (of the same week).

R visits on Thursday. Only two people visit between R and the one who visits Germany. Only four people visit between the one who visits Germany and V. The one who visits Malaysia visits immediately before V. Only two people visit between the one who visits Malaysia and P. S visits on one of the days after the one who visits Malaysia.

U visits immediately after the one who visit Japan. U does not visit Malaysia. Only three people visit between the one who visits Japan and the one who visits Nepal. The one who visits Australia visits immediately before the one who visits China. Q does not visit on Monday.

26. Which of the following countries does Q visit?

(a)  China

(b)  Malaysia

(c)  Japan

(d)  Nepal

(e)  Australia

Answer: (b)

27. On which of the following days does U visit a country?

(a)  Friday

(b)  Saturday

(c)  Sunday

(d)  Wednesday

(e)  Tuesday

Answer: (d)

28. Which of the following is true about T?

(a)  All the given options are true.

(b)  T visits on Friday.

(c)  T visits China.

(d)  Only three people visit between T and R.

(e)  T visits immediately before P.

Answer: (e)

29. Who amongst the following visits India?

(a)  S

(b)  T

(c)  P

(d)  G

(e)  R

Answer: (a)

30. As per the given arrangement, P is related to the one who visits Japan in a certain way and V is related to the one who visits Nepal in the same way. To which of the following is R related to in the same way?

(a)  The one who visits Australia

(b)  The one who visits China

(c)  The one who visits India

(d)  The one who visits Malaysia

(e)  The one who visits Germany

Answer: (b)

31. Four of the following five are alike in a certain way and thus from a group as per the given arrangement. Which of the following does not belong to that group?

(a)  U-Friday

(b)  Q-Thursday

(c)  S-Saturday

(d)  V-Sunday

(e)  T-Tuesday

Answer: (a)

Directions (Q. Nos. 32-35) Study the following information carefully and answer the given questions.

When a word and number arrangement machine is given an input line of words and numbers, it arranges them following a particular rule. The following is an illustration of input and rearrangement. (All the numbers are two-digit numbers.)

Input 23 kinetic amount 64 nature 71 58 opium verdict 96 elderly 15

Step I opium 23 kinetic amount 64 nature 71 verdict 96 elderly 15 58

Step II elderly opium 23 kinetic amount nature 71 verdict 96 15 58 64

Step III amount elderly opium 23 kinetic nature 71 verdict 15 58 84 96

Step IV 15 amount elderly opium 23 kinetic nature 71 58 64 96 verdict

Step V 23 15 amount elderly opium kinetic 71 58 64 96 verdict nature

Step VI 58 23 15 amount elderly opium 71 84 96 verdict nature kinetic

Step VI is the last step of the above arrangement and as the intended arrangement is obtained.

As per the rules followed in the given steps, find out the appropriate steps for the given input.

Input adverb 59 36 salient 81 idioms bakery 14 launch 47 umpire 62

32. Which elements come exactly between ‘59’ and ‘bakery’ in step II of the given input?

(a)  Only launch

(b)  Only 62

(c)  Only idioms

(d)  Both 81 and salient

(e)  Both adverb and 36

Answer: (a)

33. Which of the following combinations represents the first two and the last two element of the step VI of the given input?

(a)  47, 36 launch, bakery

(b)  81, 62 salient, launch

(c)  50, 47, bakery, salient

(d)  62, 14, idioms, umpire

(e)  62, 81, umpire, launch

Answer: (a)

34. If in the step III, ‘idioms’ interchanges its position with ’81’ and ‘salient’ also interchanges its position with ‘14’, then which element will be third to the right of ‘47’?

(a)  14

(b)  salient

(c)  adverb

(d)  idioms

(e)  81

Answer: (d)

35. Which step are the elements ‘bakery launch 47’ found in the same order?

(a)  Sixth

(b)  Third

(c)  Fourth

(d)  The given order of elements is not found in any step.

(e)  Fifth

Answer: (c)

36. This question consist of information and two statements numbered I and II given below it. You have to decide which of the given statements weakens or strengthens the information and decide the appropriate answer.

Information Due to increased cases of kidnapping in its vicinity, school M has made it compulsory for parents or legal guardians of the students to give a duly signed authority letter to the person picking up the students from school.

I. Most working parents rely on their domestic help, for picking up their children from school, who can easily coerced into forging or misusing the authority letter for monetary plans.

II. There is no photograph on the authority letter making it difficult to identify the person who comes to pick up the child.

(a)  Both statements I and II strengthens the information.

(b)  Statement I strengthens the information while statement II weakens the information.

(c)  Both statements I and II weakens the information.

(d)  Both statements I and II are neutral statements.

(e)  Statements I weakens the information while statement II strengthens the information.

Answer: (b)

37. How many such pairs of letters are there in the word ‘PAINTED’ each of the which has as many letters between them in the word in both forward and backward directions, as there are between them in the English alphabetical series?

(a)  More than three

(b)  Two

(c)  Three

(d)  None

(e)  One

Answer: (c)

38. If A = 2, B = 4, then MBA = ?

(a)  24

(b)  30

(c)  32

(d)  34

(e)  38

Answer: (c)

Directions (Q. Nos. 39-43) Study the given information carefully to answer the given questions.

In a certain code language,

‘urban people prefers cars’ is written as ‘ve fm ab eg’

‘profit for urban areas’ is written as ‘ab ep zi so’

‘people demand for hike’ is written as ‘zi qr cd ve’

‘hike in profit margin’ is written as ‘al jn ep cd’

(All codes are two-letter codes only.)

39. What will be the possible code for ‘urban food demand’ in the given code language?

(a)  qr ab nj

(b)  qr cr ab

(c)  nj qr cd

(d)  zi ab nj

(e)  zi ve nj

Answer: (b)

40. What is the code for ‘areas’ in the given code language?

(a)  ab

(b)  zi

(c)  ep

(d)  qr

(e)  so

Answer: (e)

41. What does the code ‘jn’ stand for in the given code language?

(a)  Either ‘hike’ or ‘people’

(b)  Either ‘in’ or ‘margin’

(c)  profit

(d)  hike

(e)  demand

Answer: (b)

42. In the given code language, if ‘small’ is coded as ‘wy’, then how will ‘prefer small cars’ be coded as?

(a)  wy eg ab

(b)  fm ve wy

(c)  eg wy fm

(d)  a beg fm

(e)  ab wy eg

Answer: (c)

43. What will be the code for ‘hike’ in the given code language?

(a)  xl

(b)  zi

(c)  Other than those given as options

(d)  qr

(e)  jn

Answer: (c)

44. Read the given information and answer the question.

Long-term usage of antibiotics causes the disease. ‘Cretosis’ as it decreases the secretion of hormone X. While body can endure the level of hormone X dropping to half the required number of micrograms in levels dropping to 23-microgrms needs immediate medical attention.

Which of the following can be concluded from the given statement?

(a)  A patient can be said to have Cretosis only if his/her hormone ‘X’ levels are 23 micrograms or low.

(b)  Usage of antibiotics on a short-term cannot cause ‘Cretosis’ ever to a minor extent.

(c)  All micrograms is exactly half of the amount of hormone X required daily by the body.

(d)  The normal numbers of micrograms of hormone X is more than all micrograms.

(e)  In a patient with hormone X level of 21 microgram, if administered another 23 micrograms would bring the level to absolute normal.

Answer: (a)

Directions (Q. Nos. 45-49) Read the given information carefully to answer the given questions.

Eight people K, L, M, N, O, P, Q and R live on eight different floors of a building but not necessarily in the same order.

The lowermost floor of the building is numbered one, the one above that is numbered two and so on till the topmost floor is numbered eight.

P lives on an even numbered floor but not on the topmost floor. Only three people live between L and P. L lives immediately below M. Only two people live between R and M. K lives above Q any of the floor. There are as many people between Q and R as are there between R and O.

45. If L and P interchanges their position and so do K and M, then who will live between M and L, as per the new arrangement?

(a)  Q

(b)  O

(c)  N

(d)  R

(e)  Other than those given as options

Answer: (a)

46. Who amongst the following lives on the floor numbered six?

(a)  M

(b)  P

(c)  Q

(d)  R

(e)  L

Answer: (e)

47. On which of the following floor numbers does O live?

(a)  Eight

(b)  Six

(c)  Three

(d)  Seven

(e)  Two

Answer: (c)

48. As per the given arrangement, four of the following five are like in a certain way and so from a group. Which one of the following does not belong to the group?

(a)  MR

(b)  RP

(c)  QL

(d)  LO

(e)  NK

Answer: (b)

49. Who amongst the following live exactly between K and R?

(a)  L, P

(b)  P, Q

(c)  Q, O

(d)  R, L

(e)  Q, R

Answer: (b)

50. Study the given information carefully to answer the given question.

The following are two findings of a one year long survey conducted on the employees of company K.

(A) Every time, an employee is rewarded for his/her work, he/she has performed better for at least next two years.

(B) The performance of none of the employees of company K has improved in the past ten months.

Which of the following can b e inferred from the given information?

(Note An inference is something by which you can logically deduce something to be true based on the known promises.)

(a)  More than 90% of company K’s employees work to their highest potential only when they are rewarded.

(b)  All the employees of company K  have been rewarded at least once in their work span.

(c)  The employees receive heavy incentives apart from being rewarded which drive them to perform better.

(d)  There are factors other than being rewarded which affect the performance of the employees.

(e)  No employee has been rewarded for his/her work in the last two quarters.

Answer: (c)

Part II Quantitative Aptitude

51. At present, Ron is eight years younger to Emma. Harry is two years younger to Emma. If the respective ratio between the present age of Ron and that of Harry is 3 : 4. What is Harry’s present age?

(a)  20 yr

(b)  8 yr

(c)  12 yr

(d)  24 yr

(e)  18 yr

Answer: (d)

52. The average of 12 observations is 8, later it was observed that one observation 10 is wrongly written as 13. The correct average of observations is

(a)  7

(b)  17.5

(c)  7.75

(d)  8

(e)  5

Answer: (c)

53. In March, 2014. Rashmi paid EMI which was 30% of her monthly salary. The remaining salary she spent on shopping of groceries and clothes in the respecting ratio of 4 : 3. She spent Rs 15000 on shopping of clothes. If in April, 2014 her salary increasing by 12%, what was her salary in April?

(a)  Rs 48000

(b)  Rs 50000

(c)  Rs 56000

(d)  Rs 66000

(e)  Rs 55000

Answer: (c)

Directions (Q. Nos. 54-58) In the following question two equations numbered I and II are given. Solve both the equations and

Give answer

(a) if x > y

(b) if x ≥ y

(c) if x < y

(d) if x ≤ y

(e) if x = y or the relationship cannot be established

54. I. 6x2 – 14x + 8 = 0 II. 3y2 – 13y + 12 = 0

Answer: (d)

55. I. 4x2 – 11x – 15 = 0           II. 8y2 – 2y – 6 = 0

Answer: (e)

56. I. 30x2 + 36x + 6 = 0 II. 30y2 + 11y + 1 = 0

Answer: (d)

57. I. 3x2 – 19x + 20 = 0 II.16y2 – 28y – 8 = 0

Answer: (a)

58. I. x2 – 8x + 15 = 0 II. 2y2 + 9y + 10 = 0

Answer: (a)

Directions (Q. Nos. 59-63) Each of the question below consists of a question and two statements number I and II given below it. You have to decide whether the data provided in the statements are sufficient to answer the question. Read both statements and

Give answer

(a) if the data in statement I alone are sufficient to answer the question, while the data in statement II alone are not sufficient to answer the question.

(b) if the data in statement II alone are sufficient to answer the question, while the data in statement I alone are not sufficient to answer the question

(c) if the data in statement I alone or in statement II alone are sufficient to answer the question

(d) if the data in both the statements I and II are not sufficient to answer the question

(e) if the data in both the statements I and II together are necessary to answer the question

59. What was the annual salary of the Mr. Murthy in 2013?

(I)  Out of the annual salary in 2012, Mr. Murthy invested 18% in Mutual Funds and 22% in Stocks. He spent 45% of his annual salary on household expenses and saved an amount of Rs 4.5 lakh at the end of the year.

(II) Annual salary of Mr. Murthy increased to 12% from 2012 to 2013 and by this he got a raise of Rs 3.6 lakh in his annual salary.

Answer: (a)

60. What is Rasika’s present age?

(I)  Rasika’s age four years hence will b e three times Manisha’s age that time.

(II) Rasika’s age two y ears ago was five times Manisha’s age that time.

Answer: (e)

61. What is the area of the square?

(I)  Area of the largest circle that can be inscribed in the given square is 616 cm2.

(II) Area of the smallest circle in which the given square can be inscribed is 1212 cm2.

Answer: (a)

62. What is the cost of painting four walls of the rectangular hall wall at Rs. 135 per m2, the ball has a door measuring 3.5 ×5 m and no windows?

(I)  Perimeter of the floor of the hall is equal to the perimeter of a square field having side 12 m. Length and breadth of the hall are in the ratio of 5 : 1 respectively.

(II) Perimeter of a smaller wall is 15 m.

Answer: (e)

63. What was the initial quantity of juice in the mixture of juice and water?

(I)  Initial quantity of the mixture was 25 L.

(II) Initially the quantity of juice in the mixture was 70%, after adding 10 L of water, it became 50%.

Answer: (b)

64. A started a business with an investment of Rs 5000. After 2 months, B and C joined with Rs 2500 and Rs 3500 respectively. If total annual profit was Rs 4800, what was B’s share in the annual profit?

(a)  Rs 1150

(b)  Rs 1000

(c)  Rs 1050

(d)  Rs 1820

(e)  Rs 1200

Answer: (b)

Directions (Q. Nos. 65-70) Refer to the pie-chart and answer the given questions.

65. In 2013, the number of female Associate Professors in university B was double the number of male Associate Professors in the same university. If in university B, the number of female Associate Professors is same as that in university D, what is the number of male associate professors in university B?

(a)  24

(b)  26

(c)  18

(d)  25

(e)  20

Answer: (c)

66. In 2014, equal number of Associate Professors merged from universities B and D. If the resultant respective ratio between the number of Associate Professors in university B and that the university D is 8 : 7, what is the number of Associate Professors who resigned from university B in 2014?

(a)  10

(b)  24

(c)  12

(d)  5

(e)  16

Answer: (b)

67. What is the central angle corresponding to the number of Associate Professors in university F?

(a)  16°

(b)  19.6°

(c)  18°

(d)  20°

(e)  17.4°

Answer: (c)

68. In January, 2013, 24% of the Assistant Professors in university E were promoted to Associate Professors. If university E had 54 professors in December, 2012, what was the number of Assistant Professors in December, 2012?

(Note : No Associated Professor was recruited and no Assistant Professors left university E in the same time.)

(a)  150

(b)  50

(c)  126

(d)  100

(e)  75

Answer: (c)

69. What is the average number of Associate Professors in universities A, C and E?

(a)  61

(b)  63

(c)  60

(d)  65

(e)  66

Answer: (c)

70. The total number of Associate Professors in universities C and F together is what percent m ore than the number of Associate Professors in university A?

(a)  70%

(b)  65%

(c)  80%

(d)  60%

(e)  93%

Answer: (e)

71. The speed of a boat in still water is 15 km/h and the speed of the current is 2 km/h. The time taken by the boat to travel from point A to point B downstream is 1 hour less than the time taken to the same boat to travel from point B to point C upstream. If the distance between points A and B is 4 km less than that between points B and C, what is the distance between points Band C?

(a)  30 km

(b)  40 km

(c)  45 km

(d)  36 km

(e)  42 km

Answer: (d)

72. 10 men can complete a piece of work in 6 days and 6 women can complete the same piece of work in 12 days. In how many days will 15 men and 10 women together complete the work?

(a) 

(b) 

(c) 

(d) 

(e)  3 days

Answer: (b)

Directions (Q. Nos. 73-78) Refer to the graph and answer the given questions.

73. On December 25, the number of gift articles sold by shop B was twice of what the same shop sold on December 24. How many gift articles did shop B sell on December 25?

(a)  300

(b)  440

(c)  540

(d)  560

(e)  400

Answer: (c)

74. Number of gift articles sold by shop A on December 22 is what percent more than that sold by shop B on the same day?

(a)  20%

(b)  15%

(c)  50%

(d)  30%

(e)  25%

Answer: (a)

75. What is the difference between total number of gift articles sold by both the shops together on December 21 and that sold by both the shops together on December 23?

(a)  230

(b)  240

(c)  170

(d)  180

(e)  270

Answer: (a)

76. Number of gifts articles sold by shop B on December 20 is what percent of number of articles sold by the same shop on December 24?

(a)  25%

(b) 

(c) 

(d) 

(e)  14%

Answer: (d)

77. What is the average number of gift articles sold by shop B on December 20 and 23?

(a)  70

(b)  65

(c)  80

(d)  60

(e)  130

Answer: (e)

78. What is the respective ratio between the total number of gift articles sold by shop A on December 23 and 24 together and by the same shop on December 21 and 22 together?

(a)  9 : 4

(b)  9 : 5

(c)  11 : 7

(d)  11 : 9

(e)  9 : 7

Answer: (b)

79. A mobile phone and a tablet were sold at a profit of 10% and at a loss of 8% respectively. If the cost price of the mobile is 1.5 times that of the tablet, what is the overall profit percentage earned by selling both the articles?

(a)  3.2%

(b)  2.8%

(c)  5%

(d)  4%

(e)  1.6%

Answer: (b)

80. The interest earned on Rs 4000 when invested in scheme A for two years at 7% per annum simple interest is half of the interest earned when Rs ‘X’ is invested for five years in the same scheme at the same rate of interest. What is the value of ‘X’?

(a)  Rs 2000

(b)  Rs 3000

(c)  Rs 3600

(d)  Rs 2400

(e)  Rs 3200

Answer: (e)

81. A jar contains a mixture of milk and water in the respective ratio of 3 : 1. When 4 L of the mixture is taken out and thereafter 3 L of milk is added to the remaining mixture. The respective ratio of milk and water in the resultant mixture thus formed is 4 : 1. What was the initial quantity of water in the mixture?

(a)  1 L

(b)  6 L

(c)  4 L

(d)  2 L

(e)  3 L

Answer: (c)

Directions (Q. Nos. 82-86) What approximate value will come in place of question marks in the given questions? (You are not expected to calculate the exact value.)

82. 

(a)  15

(b)  90

(c)  50

(d)  60

(e)  30

Answer: (e)

83. (24.99% of 900.911) ÷ (10.30% of 25.011) = ?4

(a)  9

(b)  30

(c)  27

(d)  3

(e)  6

Answer: (d)

84. 

(a)  40

(b)  93

(c)  60

(d)  58

(e)  49

Answer: (d)

85. 

(a)  40

(b)  8

(c)  24

(d)  16

(e)  2

Answer: (e)

86. 3481/3 ×001 × (27.998)2 ÷ (1.997)3 = 2? × 74

(a)  4

(b)  6

(c)  5

(d)  3

(e)  2

Answer: (e)

87. The side of a square is equal to the length of a rectangle; also the side of the square is twice the breadth of the rectangle. If the sum of the areas of the square and rectangle is 48 cm2, what is the length of the rectangle?

(a)  5 cm

(b)  3 cm

(c)  6 cm

(d)  9 cm

(e)  4 cm

Answer: (c)

Directions (Q. Nos. 88-93) Study the table and answer the given questions.

88. The number of illiterate in village C are what percent less than that in village D?

(a) 

(b)  52%

(c) 

(d) 

(e)  60%

Answer: (d)

89. What is the respective ratio between number of literates in village A and that in village C?

(a)  4 : 3

(b)  5 : 3

(c)  7 : 4

(d)  8 : 5

(e)  9 : 4

Answer: (d)

90. If in village B, two-third of the literates (males & females) are males, what percent of total number of females in village B are literate?

(a)  15%

(b)  35%

(c)  30%

(d)  20%

(e)  25%

Answer: (e)

91. Total population of village B is what percent more than that of total population of village C?

(a)  100%

(b)  150%

(c)  180%

(d)  200%

(e)  125%

Answer: (b)

92. What is the difference between total number of males in village C and D together and the total number of females in the same villages together?

(a)  140

(b)  100

(c)  200

(d)  210

(e)  120

Answer: (a)

93. What is the average number of females in village A and B?

(a)  110

(b)  121

(c)  120

(d)  75

(e)  175

Answer: (e)

Directions (Q. Nos. 94-99) What will come in place of question marks in the given number series?

94. 120   24      6        2        ?        1

(a)  2

(b)  0.5

(c)  1.5

(d)  1

(e)  2.5

Answer: (d)

95. 90   91      98      115    149    ?

(a)  274

(b)  240

(c)  209

(d)  252

(e)  196

Answer: (c)

96. 6    7        16      51      ?        1045

(a)  257

(b)  194

(c)  139

(d)  153

(e)  208

Answer: (e)

97. 124     122    127    117    134    ?

(a)  112

(b)  108

(c)  106

(d)  116

(e)  90

Answer: (b)

98. 6    5        6        10      ?

(a)  20.5

(b)  26

(c)  19

(d)  29

(e)  40

Answer: (c)

99. 274     136    66      30      11      ?

(a)  5.5

(b)  0.5

(c)  5

(d)  1

(e)  2

Answer: (b)

Directions (Q. Nos. 101-109) Which of the  phrases given against the sentence should replace the word/phrase given in bold in the sentence to make it grammatically correct? If the sentence is correct as it is given and no correction is required. Mark ‘No correction required’ as the answer.

100. Sia gave one-fourth of the money she had with her to her brother. Her brother. Her brother, from the money he received from Sia, spent one-eighth on his bus fare, one-fourth on tuition fees. After the mentioned expenses, Sia’s brother had Rs 125 remaining with him. How much money did Sia have initially?

(a)  Rs 750

(b)  Rs 800

(c)  Rs 780

(d)  Rs 790

(e)  Rs 770

Answer: (b)

Part III English Language

Directions (Q. Nos. 101-109) Which of the phrases given against the sentence should replace the word/phrase given in bold in the sentence to make it grammatically correct? If the sentences is correct as it is given and no correction is required. Mark ‘No correction required’ as the answer.

101. Ultimately, the only way to sustained a competitive advantage is to upgrade it.

(a)  ways to sustain

(b)  sustainable ways

(c)  way to sustainable

(d)  way to sustain

(e)  No correction required

Answer: (d)

102. What exactly have the managers being doing wrong?

(a)  doing wrong been

(b)  been doing wrong

(c)  been wrong doing

(d)  wrongly being doing

(e)  No correction required

Answer: (b)

103. Success in trade is the result on patent and meticulous preparations.

(a)  are result of

(b)  is resulting of

(c)  is the result of

(d)  results of the

(e)  No correction required

Answer: (c)

104. Competitors will eventually and inevitably overtake any companies that steps improving and innovating.

(a)  any company that

(b)  any companies who

(c)  any company

(d)  many company that

(e)  No correction required

Answer: (a)

105. The giant search engine has been flirting with virtual reality but has never quite full dived into it until now.

(a)  yet never fully quite

(b)  but has never quite fully

(c)  but ever fully

(d)  never has but full quietly

(e)  No correction required

Answer: (b)

106. Under the agreement, the government of Japan is committed to provide a soft loan of $ 19864 billion to its neighbouring country.

(a)  committing to provide

(b)  provides to commit

(c)  provides committing

(d)  commitment to provide

(e)  No correction required

Answer: (e)

107. By its very nature, innovative design is initially destructive of capital- either in the form for labour skills or capital equipment.

(a)  either in the form and

(b)  in either forming of

(c)  neither form on

(d)  either in the form of

(e)  No correction required

Answer: (d)

108. A teenager has work out how germs travel on airplanes and what can be done to stop them.

(a)  has worked out

(b)  have worked on

(c)  worked in

(d)  has been worked out

(e)  No correction required

Answer: (a)

109. Ordinary salary is just one factor to consider when it come to choosing a university, exclaimed the Director of Civic University.

(a)  when it come to

(b)  when it comes to

(c)  when that comes to

(d)  when it coming to

(e)  No correction required

Answer: (b)

110. Whenever he comes here, he brought down a gift.

(a)  brought up

(b)  brings up

(c)  brings down

(d)  brings

(e)  No correction required

Answer: (d)

Directions (Q. Nos. 111-120) Read the following passage carefully and answer the questions. Certain words/phrases are given in bold to help you locate them while answering some of the questions.

Today, the discipline of science that Sir Isaac Newton helped found in the second half of the 17th century has extended humanity’s horizons to a degree he could scarcely have envisaged. Even though Pluto was reclassified as a dwarf planet in 2006, with the discovery of other similarly sized bodies nearby, the latest mission of America’s space agency NASA to Pluto is expected to produce plenty of data for planetary scientists to pore over. But then the stream of missions to the outer planets (namely Jupiter, Saturn, Uranus and Neptune) turns into a trickle. At the same time, Cassini was launched in 1997 to explore Saturn and its Moons but by 2017 its propellant will be depleted and provided it survives a series of fly-bye through Saturn’s rings- It will burn up as it plunges through the planet’s thick atmosphere. Sometime, before 2025 even the stalwart voyage probes, both launched in 1977, will lack the power continue sending back data. Voyager-1, now in interstellar space, is the most distant man-made object in the Universe, and Voyage-2 is not far behind.

The upshot is that for a decade or so, discoveries will come mostly from objects closer to Earth, regular excursions to Mars are planned. There will also be plenty of instruments launched to look at Earth itself. The hiatus might n ot end until two proposed space missions are launched in the early 2020s. It seems an abrupt slowdown after a golden age of missions by NASA and European Space Agency (ESA). But, building a space probe in both complicated an expensive, it takes years of planning and jostling for funds as well as hefty dose of lack to ensure that complex equipment works well.

We are travelling today from some good science and good funding in the 1990s. And money has become much scarcer in recent years. In 1981, the recent high-water mark for NASA, the agency received $ 25 billion. Its budget fell to a low of $ 16.9 billion in 2013. Some of NASA’s cash has been shifted to other projects. NASA’s cooperation with ESA on future missions has also been scaled back as a result of budget cuts. The Europeans, by contrast, have kept their funding fairly steady. But, ESA’s budget is just £ 4.4 billion ($ 4.9 billion). Other countries are interested in space and have missions under way or in the making, including China, Japan and India. But, so far they have no ambitions to venture beyond Mars.

 Does the coming gap in planetary exploration matter? Studying the geology, atmosphere and evolution of plants, and comets provides valuable science. Others have loftier ambitions-keeping planetary science going is critical to the long-term survival of the species on this plant. Because space missions have such long lead times, the looming run of years will have deleterious effects even if budgets start to rise again. The concern is that when funding does get back, there will be a missing generation of valuable knowledge almost. It’s really difficult to go through boom and bust cycles since you’ve got to keep the scientific community and the engineers ticking over to maintain the expertise will have in outer solar-system exploration.

We are travelling today from some good science and good funding in the 1990s. And money has become much scarcer in recent years. In 1981, the recent high-water mark for NASA, the agency received $ 25 billion. Its budget fell to a low of $ 16.9 billion in 2013. Some of NASA’s cash has been shifted to other projects. NASA’s cooperation with ESA on future missions has also been scaled back as a result of budget cuts. The Europeans, by contrast, have kept their funding fairly steady. But, ESA’s budget is Just £4.4 billion ($ 4.9 billion). Other countries are interested in space and have missions under way or in the making, including China, Japan and India. But, so far they have no ambitions to venture beyond Mars.

Does the coming gap in planetary exploration matter? Studying the geology, atmosphere and evolution of plants, and comets provides valuable science. Others have loftier ambitions-keeping planter science going is critical to the long-term survival of the species on this planet. Because space missions have such long lead times, the looming run of years will have deleterious effects even if budgets start to rise again. The concern is that when funding does get  back, there will be a missing generation of valuable knowledge almost. It’s really difficult to go through boom and bust cycles since you’ve got to keep the scientific community and the engineers ticking over to maintain the expertise will have in outer solar-system exploration.

111. Which of the following is the central idea of the passage?

(a)  Space travel is exclusive to developed counties and this is unlikely to change.

(b)  We are too focused on studying the universe and proving the existence of extraterrestrial life.

(c)  There has been a decline in the quality of scientific discovery in recent times.

(d)  Despite huge leaps in planetary science in the past, exploration is unfortunately likely to dwindle now.

(e)  Though we have physically explored various planets we are unable to make them habitable.

Answer: (d)

112. Which of the following can be said about the voyage probes?

(a)  These have been obsolete for a long time and should be called as soon as possible.

(b)  These probes have been damaged and are responsible for polluting the galaxy.

(c)  These have travelled the furthest and provided invaluable insights in the field of planetary science.

(d)  Scientists have lost contact with these and worryingly cannot prepare their exact location.

(e)  Too many resources are diverted to maintaining these outdated probes.

Answer: (a)

113. Which of the following is/are (a) factor (s) which affect space missions today?

(A) Funding from NASA and ESA to space programme in developing countries.

(B) Scarcity of engineers in the field.

(C) Budgets and advance planning of projects.

(a)  Only A

(b)  Only C

(c)  A, B and C

(d)  Only B

(e)  A and C

Answer: (c)

114. What does the phrase ‘It’s really difficult to go through boom-and-bust cycle’ convey?

(a)  Some economics are still trying to recover from exorbitant space funding in the year 199s.

(b)  Space exploration missions have had more failures than successes.

(c)  Global economic mission since 2000 has resulted in lack of employment for engineers.

(d)  Fluctuations in space exploration funding are not desirable.

(e)  The durations for space missions should be more optimally planned to avoid wastage of resources.

Answer: (d)

115. Choose the word which is most nearly the same in meaning to the word ‘Matter’ given in bold as used in the passage?

(a)  Substance

(b)  Theme

(c)  Suitable

(d)  Count

(e)  Question

Answer: (c)

116. What does the author suggest regarding countries such as China, Japan and India?

(A) These countries should increase their budgets for space travel.

(B) These should enter into agreements with developed countries for space exploration.

(C) Their ides of making Mars habitable are too lofty.

(a)  Only A

(b)  Only B

(c)  B and C

(d)  A and C

(e)  None of these

Answer: (e)

117. What is the author’s view regarding reduction in funds for space exploration?

(a)  It is an appropriate step as the planet face more pressing problems.

(b)  It will be detrimental as it will hamper scientific discovery and knowledge.

(c)  It is desirable as we have adequate evidence that sustaining life in some planets is too costly.

(d)  It is an unwelcome move as several space missions are scheduled this year.

(e)  Other than those given as options

Answer: (b)

118. What do the statistics on space agency budgets cited in the passage indicate?

(a)  Space agency budgets vary across countries and within a country over time.

(b)  Building spacecrafts is becoming more expensive over time.

(c)  Despite increasing space budgets, there is a lack of innovation in space exploration

(d)  There have been many expensive failures in space missions in recent times.

(e)  Asia’s space budget is fast catching up to Europe’s and America’s till present.

Answer: (a)

119. Choose the word which is opposite in meaning to the word ‘Extended’ given in bold as used in the passage?

(a)  Postponed

(b)  Delayed

(c)  Amplified

(d)  Curtailed

(e)  Relaxed

Answer: (d)

120. Which of the following is true in the context of the passage?

(a)  There is conflict among scientists about the classification of planetary bodies.

(b)  Collaborations among nations for space exploration has not really worked.

(c)  Studying outer space has implications for life on Earth.

(d)  The number of space scientists has fallen since the 1990s.

(e)  None of the given options is true in the context of the passage.

Answer: (c)

Directions (Q. Nos. 121-130) Read each sentence of find out whether there is any error in it the error, if any, will be in one part of the sentence. The number of that part is the answer. If there ‘No error’, then answer is (e).

121. The next time you/ are at the city airport,/ apart of shopping for the usual,/ you can also purchase a piece of art.

(a)  The next time you

(b)  are at the city airport,

(c)  apart of shopping for the usual,

(d)  you can also purchase a piece of art

(e)  No error

Answer: (c)

122. Despite being laid low by illness/ in the run-up to the event, / the sportsman intend to give his best/ on the upcoming championship.

(a)  Despite being laid low by illness

(b)  in the run-up to the event,

(c)  the sportsman intend to give his best

(d)  on the upcoming championship

(e)  No error

Answer: (c)

123. After staying together / for several years, the actress/ finally separated from her husband/ for good in 2004.

(a)  After staying together

(b)  for several years, the actress

(c)  finally separated from her husband

(d)  for good in 2004

(e)  No error

Answer: (e)

124. The city’s young women/ are going out and buying/ diamonds themselves, as by gifted/ diamonds by men is such

(a)  The city’s young women

(b)  are doing out and buying

(c)  diamonds themselves, as by gifted

(d)  diamonds by men is such passed

(e)  No error

Answer: (c)

125. After swung between playing / positive and negative characters, / the actor is set to attempt comedy / for the first time on small screen.

(a)  After swung between playing

(b)  positive and negative characters,

(c)  the actor is set to attempt comedy

(d)  for the first time on small screen

(e)  No error

Answer: (a)

126. Research’s show that people / who are able to responding/ more quickly to questions are / perceived as more charismatic.

(a)  Research’s show that people

(b)  who are able to responding

(c)  more quickly to questions are

(d)  perceived as more charismatic

(e)  No error

Answer: (b)

127. His wife’s untimely death / in a plane crash and him/ to the supportive actress, who/ lend him a shoulder to cry on.

(a)  His wife’s untimely death

(b)  in a plane crash and him

(c)  to the supportive actress, who

(d)  lend him a shoulder to cry on

(e)  No error

Answer: (b)

128. Staying healthy / and high spirited/ is not / very difficult.

(a)  Staying healthy

(b)  and high spirited

(c)  is not

(d)  very difficult

(e)  No error

Answer: (e)

129. Like against/ a fixed interest rate loan, / a floating interest rate loan offers/ flexibility to borrowers.

(a)  Like against

(b)  a fixed interest rate loan,

(c)  a floating interest rate loan offers

(d)  flexibility to borrowers

(e)  No error

Answer: (a)

130. The director refused/ to meet his critics/ and did not respond to/ any of their letters.

(a)  The director refused

(b)  to meet his critics

(c)  and did not respond to

(d)  any of their letters

(e)  No error

Answer: (e)

Directions (Q. Nos. 131-135) Each sentence has two blanks which indicated that something has been omitted. Choose the words that best fit the meaning of the sentences as a whole.

131. Findings ………… that social intelligence is more than just ……. the right thing to do.

(a)  depict; making

(b)  state; letting

(c)  suggest; ascertaining

(d)  show; knowing

(e)  illustrate; allowing

Answer: (d)

132. Saunas are not only a good way to detox, but they also help you get ………… a cold quicker by opening up your sinus passage, and helping you ……….. easily.

(a)  over; breathe

(b)  past; air

(c)  better; smell

(d)  leave; oxygenate

(e)  arrest; vacate

Answer: (a)

133. A fixed rate home loan is …………… for those who want to have a …………. monthly repayment schedule.

(a)  good; high

(b)  kept; limited

(c)  ideal; predictable

(d)  prime; logical

(e)  best; annual

Answer: (c)

134. Paintings are generally quite ……………, but by expanding art to forms and objects beyond paintings, we will make it ……… to the common man.

(a)  costly; limited

(b)  cheap; available

(c)  expensive; accessible

(d)  reasonable; pricey

(e)  steep; exorbitant

Answer: (c)

135. No ………… how big or small piece of jewerllery, it is the fact that you bought it with your own money that makes it

(a)  matter; special

(b)  doubt; unique

(c)  problem; stand

(d)  way; small

(e)  issue; dear

Answer: (a)

Directions (Q. Nos. 136-140) Rearrange the following six sentences A, B, C, D, E and F in a proper sequence to form a meaningful paragraph, then answer the given questions.

(A) At the same time; allowing restaurant drivers to take leftovers home in a ‘doggy bag’ is a common phenomenon in the US, but the practice is frowned upon in some EU countries.

(B) An approach to train waste-minimizing habits is through cooking classes, for example, the local authority of Burssels train 1900 people in 2009 on how to minimize waste.

(C) Caterers can minimize waste by anticipating demand, informed by reservations and customer feedback surveys.

(D) There are similar education opportunities in the hospitality industry as well.

(E) Societal efforts are needed to banish this embarrassment.

(F) The European Parliament has recommended that this practical training be incorporated in school curricula.

136. Which of the following should be the THIRD sentence after the rearrangement?

(a)  A

(b)  B

(c)  F

(d)  D

(e)  E

Answer: (c)

137. Which of the following should be the FOURTH sentence after the rearrangement?

(a)  A

(b)  B

(c)  C

(d)  E

(e)  D

Answer: (e)

138. Which of the following should be the LAST (SIXTH) sentence after the rearrangement?

(a)  A

(b)  E

(c)  D

(d)  C

(e)  B

Answer: (b)

139. Which of the following should be the FIRST sentence after the rearrangement?

(a)  A

(b)  B

(c)  C

(d)  D

(e)  E

Answer: (b)

140. Which of the following should be the FIFTH sentence after the rearrangement?

(a)  A

(b)  B

(c)  C

(d)  D

(e)  F

Answer: (a)

Directions (Q. Nos. 141-150) In the following passage, there are blanks, each of which has been numbered.

Against each, five words are suggested, one of which fits the blank appropriately. Find out the appropriate word in each case.

Around world, companies that have achieved international leadership employ strategies that (141) from each other in every respect. But, when every successful company will employ its own particular strategy, the (142) mode of operation-the character and trajectory of all successful companies is fundamentally the same.

Companies (143) competitive advantage through acts of innovation. They approach innovation in its broadest sense, including both new technologies and new ways of doing things. They perceive a new basis for competing or better means to competing in old ways. Innovations can be (144) in a new product design, a new production process, a new marketing approach or a new way of conducting training. Much innovation is mundane and incremental, depending more on accumulation of small insights and advances (145) on a single, major technological breakthrough. It often involves ideas that are not even ‘new’ ideas that have been (146), but never vigorously pursued. It always involves investments in skill and knowledge, as well as in physical assets and brand reputations.

Some innovations create competitive advantage by perceiving an entirely new market opportunity or by serving a market segment that offers have (147). When competitors are slow to respond, such innovation (148) competitive advantage. For instance, in industries such as autos and home electronics, Japanese companies (149) this initial advantages by (150) on smaller more compact, lower capacity models that foreign competitors declared as less profitable, less important and less attractive.

141.

(a)  offer

(b)  deter

(c)  after

(d)  contrast

(e)  vary

Answer: (e)

142.

(a)  underlying

(b)  basis

(c)  prima

(d)  element

(e)  routed

Answer: (a)

143.

(a)  compete

(b)  work

(c)  follow

(d)  achieve

(e)  dispatch

Answer: (d)

144.

(a)  obviously

(b)  seen

(c)  attached

(d)  attested

(e)  noticeable

Answer: (b)

145.

(a)  there

(b)  the

(c)  that

(d)  therefore

(e)  than

Answer: (d)

146.

(a)  existed

(b)  over

(c)  around

(d)  all

(e)  universally

Answer: (a)

147.

(a)  divergent

(b)  noticed

(c)  ignored

(d)  shut

(e)  detoured

Answer: (a)

148.

(a)  profits

(b)  yields

(c)  return

(d)  felicitates

(e)  turns on

Answer: (e)

149.

(a)  strengthens

(b)  proceed

(c)  allowed

(d)  gained

(e)  prove

Answer: (d)

150.

(a)  heading

(b)  indicating

(c)  touching

(d)  focusing

(e)  hitting

Answer: (d)

IBPS Bank Specialist Officers (Marketing) Scale-I Common Written Examination-2015 Quantitative Aptitude Question Paper With Answer Key

IBPS Bank Specialist Officers (Marketing) Scale-I Common Written Examination-2015 Quantitative Aptitude
IBPS Bank Specialist Officers (Marketing) Scale-I Common Written Examination-2015 Quantitative Aptitude Question Paper With Answer Key

IBPS Bank Specialist Officers (Marketing) Scale-I Common Written Examination-2015

Quantitative Aptitude

Directions-(Q. 1-10) What should come in place of question mark (?) in the following question ?

1. −84 × 29 + 365 = ?

(A)  2436

(B)  2801

(C)  −2801

(D)  −2071

(E)  None of these

Answer: (D)

2. (21.69)2 – √324 = ?

(A)  440.4615

(B)  425.4561

(C)  452.4561

(D)  442.4651

(E)  None of these

Answer: (C)

3. (800 ÷64) × 1296 ÷ 36 = ?

(A)  420

(B)  460

(C)  500

(D)  540

(E)  None of these

Answer: (E)

4. (42 × 229) ÷ (9261)1/3 = ?

(A)  452

(B)  460

(C)  500

(D)  540

(E)  None of these

Answer: (E)

5. (35423 + 7164 + 41720) – (317 × 89) = ?

(A)  28213

(B)  84307

(C)  50694

(D)  54096

(E)  None of these

Answer: (E)

6. (9% of 386) × (6.5% × 144) = ?

(A)  1340.1664

(B)  325.1664

(C)  333.3333

(D)  328.0065

(E)  None of these

Answer: (B)

7. 4500 × ? = 3375

(A)  2/5

(B)  3/4

(C)  1/4

(D)  3/5

(E)  None of these

Answer: (B)

8. 4 + 4.44 + 44.4 + 4.04 + 444 = ?

(A)  500.88

(B)  572.2

(C)  495.22

(D)  472.88

(E)  None of these

Answer: (A)

9. ∛? = (36 × 24) ÷ 9

(A)  884736

(B)  804036

(C)  854734

(D)  814736

(E)  None of these

Answer: (A)

10. (43% of 2750) – (38% of 2990) = ?

(A)  49.3

(B)  44.7

(C)  43.6

(D)  46.3

(E)  None of these

Answer: (D)

Directions- (Q. 11-15) What approximate value should come in place of question-mark (?) in the following question ? (You are not expected to calculate the exact value).

11. (421% of 738) ÷ 517 = ?

(A)  6

(B)  8

(C)  10

(D)  4

(E)  12

Answer: (A)

12. 5248 ÷ 2 ÷ 3.5 = ?

(A)  747

(B)  752

(C)  744

(D)  756

(E)  750

Answer: ()

13. (1.65% of 8471) – (0.61% of 9326)

(A)  85

(B)  81

(C)  87

(D)  83

(E)  89

Answer: (D)

14. 

(A)  50

(B)  49

(C)  43

(D)  38

(E)  40

Answer: (C)

15. (5995 + 4874 + 3333) ÷ (713 + 509 + 326) = ?

(A)  11

(B)  9

(C)  11.5

(D)  7.5

(E)  12

Answer: (B)

Directions- (Q. 16-19) Study the following table and answer the given question :

16. If family ‘C’ has saved 30% of his annual income in 2008 then how much was his annual income in that year (in thousand)?

(A)  Rs 520

(B)  Rs 730

(C)  Rs 600

(D)  Rs 580

(E)  Rs 850

Answer: (C)

17. How much percentage increased in expenditure (in thousand) of family A between 2008 to 2012 ?

(A)  11

(B)  4

(C)  10

(D)  13

(E)  7

Answer: (E)

18. What was the combined savings of all family in 2011 if the income of all families was Rs 51,20,000 ?

(A)  Rs 23,20,000

(B)  Rs 25,10,000

(C)  Rs 25,50,000

(D)  Rs 27,10,000

(E)  Rs 21,60,000

Answer: (C)

19. What was the respective ratio of annual savings of family E and D in 2011 if the annual income of each family was Rs 5,20,000 ?

(A)  6 : 21

(B)  9 : 19

(C)  7 : 26

(D)  5 : 17

(E)  7 : 30

Answer: (B)

Directions- (Q. 20-24) What will come in place of question mark (?) in the given number ?

20. 6, 9, 15, 27, 51, ?

(A)  84

(B)  99

(C)  123

(D)  75

(E)  97

Answer: (B)

21. 19, 10, 11, 18, ?, 97.5

(A)  42

(B)  38

(C)  32

(D)  30

(E)  36

Answer: (B)

22. 13, 10, 14, 14, 33, ?, 585

(A)  130

(B)  120

(C)  115

(D)  145

(E)  150

Answer: (B)

23. 9, 62, ?, 1854, 7415, 22244

(A)  433

(B)  406

(C)  309

(D)  371

(E)  417

Answer: (D)

24. 7, 8, 18, ?, 232, 1165

(A)  84

(B)  42

(C)  36

(D)  48

(E)  57

Answer: (E)

Directions- (Q. 25-28) Read the following information and answer the given questions.

25. If bakery A and B bear the cost Rs 250 and Rs 350 for the purchasing material for each cake, then what is the respective ratio between the total expenditures for the purchasing of material by bakery A in month January and the total expenditure for the purchasing material by bakery B in month April?

(A)  5 : 9

(B)  2 : 9

(C)  9 : 10

(D)  7 : 9

(E)  3 : 10

Answer: (A)

26. What is the difference in numbers between the combined number of the baked cake by bakery A during March and April and the combined number of the baked cake by bakery B during January and May ?

(A)  180

(B)  170

(C)  200

(D)  360

(E)  220

Answer: (C)

27. By what per cent is the number of baked cake by bakery B in March more than the number of baked cake by bakery A in January ? (rounded off of two digits after decimal)

(A)  78.57

(B)  81.24

(C)  66.71

(D)  92.57

(E)  71.37

Answer: (A)

28. What is the average number cakes baked by bakery B February, March, April and May together ?

(A)  450

(B)  385

(C)  410

(D)  425

(E)  370

Answer: (B)

Directions- (Q. 29-33) Study the following Pie Graph carefully and answer the question given below-

29. What is the budget estimated by the family in clothing and Grocery together?

(A)  Rs 8960

(B)  Rs 8550

(C)  Rs 8780

(D)  Rs 8690

(E)  Rs 8850

Answer: (A)

30. Due to a sudden marriage the family incurs miscellaneous expenditure of Rs 3040 in total how much is the increase in the around under this head from that budgeted ?

(A)  Rs 1738

(B)  Rs 304

(C)  Rs 800

(D)  Rs 224

(E)  None of these

Answer: (C)

31. The family actually paid Rs 4672 on Grocery. What is the difference in amount budgeted and spent on the grocery ?

(A)  Rs 1738

(B)  Rs 1672

(C)  Rs 467

(D)  Rs 1038

(E)  None of these

Answer: (E)

32. What is the difference in the amount estimated by the family on Electricity and cell bill?

(A)  Rs 1920

(B)  Rs 4160

(C)  Rs 6080

(D)  Rs 8000

(E)  Rs 4480

Answer: (B)

33. The family saved Rs 1920 on their electricity bill as it was less than the estimated budget. What is the percentage of the Electricity Bill amount on the total salary?

(A)  10.5

(B)  12

(C)  24.5

(D)  13

(E)  16

Answer: (D)

Directions- (Q. 34-38) Study the following Table carefully and answer the questions given below-

34. What is the average number of failed students from class VII for the given years ?

(A)  29.5

(B)  27

(C)  26.5

(D)  25

(E)  24.5

Answer: (E)

35. What is the ratio between total number of passed students and total number of failed students for the year 2011?

(A)  2 : 1

(B)  56 : 23

(C)  69 : 13

(D)  69: 35

(E)  315 : 105

Answer: (E)

36. Which of the following classes has the maximum number of p assed students, as compared to the total number of students of that class, over the years?

(A)  VI

(B)  VII

(C)  VIII

(D)  IX

(E)  X

Answer: (B)

37. What is the number of passed students, for all the classes together, in the year 2009 ?

(A)  277

(B)  297

(C)  315

(D)  357

(E)  377

Answer: (A)

38. What is the total percentage of passed students of class VI from all the years together ?

(A)  50

(B)  82

(C)  70

(D)  77

(E)  90

Answer: (C)

39. Rohit had a certain amount. He invests 3/5 part of the total amount in scheme A for 8 years and the balance amount invests in scheme B for two y ears Scheme A gives 18% per annum as a simple interest and scheme B gives 10% per annum as compound interest (yearly). If Rohit gets Rs 1422 as total interest, then how much money did he invest in scheme A & B?

(A)  Rs 1026

(B)  Rs 1800

(C)  Rs 1500

(D)  Rs 900

(E)  Rs 1296

Answer: (C)

40. A sum of money is divided among A, B, C and D in the ratio of 3 : 5 : 8 : 9 respectively. If the share of D is Rs 1,872 more than the share of A, then what is the total amount of money of B and C together ?

(A)  Rs 4,156

(B)  Rs 4,165

(C)  Rs 4,056

(D)  Rs 4,065

(E)  None of these

Answer: (C)

41. Arun started a business investing Rs 38,000. After 5 months Bakul joined him with a capital of Rs 55,000. At the end of the year the total profit was Rs 22,000. What is the approximate difference between the share of profits of Arun and Bakul?

(A)  Rs 1,192

(B)  Rs 1,856

(C)  Rs 1,007

(D)  Rs 1,928

(E)  Rs 1,568

Answer: (B)

42. What is the 40% of 50% of 3/4th of 3200?

(A)  480

(B)  560

(C)  420

(D)  600

(E)  None of these

Answer: (A)

43. One-fourth of a number is 17. What will 45% of that number be ?

(A)  51

(B)  45.4

(C)  37.4

(D)  30.6

(E)  None of these

Answer: (D)

44. Ishan spent Rs 35,645 on buying a bike, Rs 24,355 on buying a television and the remaining 20% of the total amount he had as cash with him. What was the total amount ?

(A)  Rs 60,000

(B)  Rs 72,000

(C)  Rs 75,000

(D)  Rs 80,000

(E)  None of these

Answer: (C)

45. A, B, C and D are four consecutive even numbers respectively and their average is 65. What is the product of A and D ?

(A)  3968

(B)  4216

(C)  4092

(D)  4352

(E)  None of these

Answer: (B)

46. The sum of the five numbers is 555. The average of first two numbers is 75 and the third number is 115. What is the average of last two numbers ?

(A)  145

(B)  290

(C)  265

(D)  150

(E)  None of these

Answer: (A)

47. A car covers the first 35 km of its journey in 45 minutes and covers the remaining 69 km in 75 minutes. What is the average speed of the car ?

(A)  42 kms/hr.

(B)  50 kms/hr.

(C)  52 kms/hr.

(D)  60 kms/hr.

(E)  None of these

Answer: (C)

48. In how many different ways can the letters of the word ‘PRIDE’ be arranged ?

(A)  60

(B)  120

(C)  15

(D)  360

(E)  None of these

Answer: (B)

49. If the numerator of a fraction is increased by 250% and the denominator is increased by 400%. The resultant fraction is 7/19. What is the original fraction ?

(A)  10/9

(B)  5/9

(C)  9/5

(D)  19/7

(E)  None of these

Answer: (A)

50. The ages of Samir and Tanuj are in the ratio of 8 : 15 years respectively. After 9 years the ratio of their ages will be 11 : 18. What is the difference in years between their ages ?

(A)  24 years

(B)  20 years

(C)  33 years

(D)  21 years

(E)  None of these

Answer: (D)

IBPS Regional Rural Bank Officers Examination Held on 12-9-2015 Quantitative Aptitude Question Paper With Answer Key

IBPS Regional Rural Bank Officers Examination Held on 12-9-2015 Quantitative Aptitude
IBPS Regional Rural Bank Officers Examination Held on 12-9-2015 Quantitative Aptitude Question Paper With Answer Key

IBPS Regional Rural Bank Officers Examination Held on 12-9-2015

Quantitative Aptitude

1. Four vehicles having equal radii are drawn with centres at the four corners of a square. Each circle touches the other two adjacent circles. If the remaining area of the square is 168 cm2, what is the size of the radius of the circle? (in centimeters)

(A)  14

(B)  1.4

(C)  35

(D)  21

(E)  3.5

Answer: (A)

Directions-(Q. 2-6) What will come in place of question-mark (?) in the given number series ?

2. 158, 78, 38, 18, 8, ?

(A)  3

(B)  5

(C)  2

(D)  7

(E)  6

Answer: (A)

3. 16, 19, 24, 33, 50, ?

(A)  83

(B)  66

(C)  99

(D)  74

(E)  102

Answer: (A)

4. 402, 400, 388, 358, 302, ?

(A)  212

(B)  236

(C)  190

(D)  182

(E)  210

Answer: (A)

5. 31, 15, 21, 50, ?, 767.25

(A)  160.5

(B)  171.5

(C)  156.5

(D)  122.5

(E)  143.5

Answer: (B)

6. 8, 5.5, 8.5, 23, 89.5, ?

(A)  455

(B)  420.5

(C)  445

(D)  415.5

(E)  433

Answer: (C)

7. A vessel contains a mixture of milk and water in the respective ratio of 14 : 3. 25.5 litres of the mixture is taken out from the vessel and 2.5 litres of pure water and 5 litres of pure milk is added to the mixture. If the resultant mixture contains 20% water, what was the initial quantity of mixture in the vessel before the replacement ? (in litres)

(A)  51

(B)  102

(C)  68

(D)  85

(E)  34

Answer: (C)

8. A, B and C started a business by investing Rs 20,000, Rs 28,000 and Rs 36,000 respectively. After 6 months, A and B withdrew an amount of Rs 8,000 each and C invested an additional amount of Rs 8,000. All of them invested for equal period of time. If at the end of the year C, got Rs 12,550 as his share of profit, what was the total profit earned?

(A)  Rs 25,100

(B)  Rs 26,600

(C)  Rs 24,300

(D)  Rs 22,960

(E)  Rs 21,440

Answer: (A)

Directions-(Q. 9-13) Refer to the pie charts carefully and answer the given question.

9. Number of cellular phones (both Nokia and Samsung) sold by store B is what per cent m ore than the number of Nokia cellular phones sold by store A?

(A) 

(B) 

(C) 

(D) 

(E) 

Answer: (B)

10. What is the central angle corresponding to number of cellular phones (both Nokia and Samsung) sold by store D?

(A)  124.2°

(B)  112.4°

(C)  115.2°

(D)  120.8°

(E)  118. 8°

Answer: (E)

11. What is the average number of Samsung cellular phones sold by stores B, C and D?

(A)  796

(B)  792

(C)  780

(D)  812

(E)  784

Answer: (B)

12. What is the difference between number of cellular phones (both Nokia and Samsung) sold by store A and Total number of Nokia cellular phones sold by stores B and E together?

(A)  442

(B)  436

(C)  432

(D)  428

(E)  426

Answer: (B)

13. Number of Samsung cellular phones sold by store E is what per cent of the number of phones (both Nokia and Samsung) sold by store C?

(A)  55

(B)  45

(C)  60

(D)  65

(E)  50

Answer: (A)

14. There are three positive numbers, 1/3rd of average of all the three numbers is 8 less than the value of the highest number. Average of the lowest and the second lowest number is 8. Which is the highest number?

(A)  11

(B)  14

(C)  10

(D)  9

(E)  13

Answer: (A)

15. A sum of money was invested for 14 years in Scheme A which offers simple interest at a rate of 8% p.a. The amount received from Scheme A after 14 years was then invested for two years in Scheme B which offers compound interest (compounded annually) at a rate of 10% p.a. If the interest received from Scheme B was Rs 6,678, what was the sum invested in Scheme A?

(A)  Rs 15,500

(B)  Rs 14,500

(C)  Rs 16,000

(D)  Rs 12,500

(E)  Rs 15,000

Answer: (E)

16. 4 years ago, the respective ratio between 1/2 of A’s age at that time and four times of B’s age at that time was 5 : 12. Eight years hence, 1/2 of A’s age at that time will be less than B’s age at that time by 2 years. What is B’s present age ?

(A)  10 years

(B)  14 years

(C)  12 years

(D)  5 years

(E)  8 years

Answer: (A)

Directions-(Q. 17 to 21) Study the table and answer the given question.

Note:

1. Total deduction = Provident Fund Deduction (which is 10% of basic salary) + Other deduction.

2. Net Salary = Basic Salary + Total allowance – Total Deduction.

3. Few values are missing in the table (indicated by…). A candidate is expected to calculate the missing value, if it is required to answer the given question, on the basis of the given data and information.

17. If other deduction of P was Rs 4,720 what was his net salary?

(A)  Rs 42,500

(B)  Rs 43,500

(C)  Rs 43,000

(D)  Rs 41,500

(E)  Rs 42,000

Answer: (B)

18. If Q’s total allowance was Rs 3,000 more than his basic salary, what was his total allowance?

(A)  Rs 17,000

(B)  Rs 17,500

(C)  Rs 16,000

(D)  Rs 16,600

(E)  Rs 15,500

Answer: (D)

19. If the respective ratio of Provident Fund deduction and other deduction of S was 7 : 13, what was S’s other deduction?

(A)  Rs 2,160

(B)  Rs 2,080

(C)  Rs 2,120

(D)  Rs 2,040

(E)  Rs 1,948

Answer: (B)

20. Basic salary of S is what percent more than the basic salary of R?

(A) 

(B) 

(C) 

(D) 

(E) 

Answer: (E)

21. If other deduction of T was Rs 4,000 what was his Net Salary ?

(A)  Rs 32,500

(B)  Rs 31,900

(C)  Rs 32,700

(D)  Rs 31,700

(E)  Rs 32,900

Answer: (E)

22. I. x2 – 3x – 88 = 0

II. y2 + 8y – 48 = 0

(A)  x ≥ y

(B)  x ≤ y

(C)  x < y

(D)  x > y

(E)  Relationship between x and y cannot be determined

Answer: (D)

23. I. 5x2 + 29x + 20 = 0

II. 25y2 + 25y + 6 = 0

(A)  x ≥ y

(B)  x ≤ y

(C)  x < y

(D)  x > y

(E)  Relationship between x and y cannot be determined

Answer: (C)

24. I. 2x2 – 11x + 12 = 0

II. 2y2 – 19y + 44 = 0

(A)  x ≥ y

(B)  x ≤ y

(C)  x < y

(D)  x > y

(E)  Relationship between x and y cannot be determined

Answer: (B)

25.I. 3x2 + 10x + 8= 0

II. 3y2 + 7y + 4 = 0

(A)  x ≥ y

(B)  x ≤ y

(C)  x < y

(D)  x > y

(E)  Relationship between x and y cannot be determined

Answer: (B)

26. I. 2x2 + 21x + 10 = 0

II. 3y2 + 13y + 14 = 0

(A)  x ≥ y

(B)  x ≤ y

(C)  x < y

(D)  x > y

(E)  Relationship between x and y cannot be determined

Answer: (E)

27. I. x2 – x – 12 = 0

II. y2 + 5y + 6 = 0

(A)  x ≥ y

(B)  x ≤ y

(C)  x < y

(D)  x > y

(E)  Relationship between x and y cannot be determined

Answer: (A)

Directions-(Q. 28-31) This data is regarding total number of employees working in Administration (Admin), Operations(Ops.( and other departments of corporate divisions of Companies A and B.

    The total number of employees working in both the companies together is 4800. The respective ratio of number of employees in Companies A and B is 5 : 7. Each employee works in only one of the mentioned departments.

   In company A, 70% of the total employees are males. 60% of the total male employees work in ‘Ops’. Out of the remaining male employees. 1/8th work in ‘Admin’. Out of the total female employees, 24% work in. ‘Admin’ and 5/8th of the remaining female employees work in ‘Ops’.

In company B, 80% of the total employees are males. 65% of the total male employees work in ‘Ops’. Number of male employees who work in ‘other departments’ in Company B is 20% more than the male employees work in ‘Other departments’ in company A. Number of female employees who works in Ops in Company B are less than number of male employees who work for ‘Ops’ in the same company by 75%. Out of the remaining female employees, 1/4 work in ‘Admin’.

28. What per cent of the total number of male employees in company A work in other departments ?

(A)  45

(B)  25

(C)  30

(D)  35

(E)  40

Answer: (D)

29. What is the total number of female employees who work in Ops in Company A and B together?

(A)  681

(B)  781

(C)  689

(D)  649

(E)  788

Answer: (D)

30. What is the difference between the average number of males working in ‘Admin’ in both the Companies together and average number of females working in ‘Other Departments’ in both the companies together?

(A)  26

(B)  36

(C)  16

(D)  24

(E)  14

Answer: (A)

31. In company B, what is the respective ratio between the total number of employees (both male and female) who work in ‘Admin’ and the total number of employees (both male and female) who work in ‘Other department’s in the same company?

(A)  2 : 3

(B)  1 : 3

(C)  1 : 4

(D)  3 : 5

(E)  1 : 5

Answer: (B)

32. Monthly salaries of Pia and Som are in the respective ratio of 5 : 4. Pia, from her monthly salary, gives 3/5th to her mother. 15% towards her sister’s tuition fees, 18% towards a loan and she shops with the remaining amount which was Rs 2,100. What is the monthly salary of Som?

(A)  Rs 25,000

(B)  Rs 30,000

(C)  Rs 15,000

(D)  Rs 20,000

(E)  Rs 24,000

Answer: (E)

33. 12 men can finish a project in 20 days. 18 women can finish the same project in 16 days and 24 children can finish it in 18 days. 8 women and 16 children worked for 9 days and then left. In how many days will 10 men complete the remaining project?

(A) 

(B)  10

(C)  9

(D) 

(E) 

Answer: (B)

34. An item was bought at Rs X and sold at Rs Y, thereby earning a profit of 20%. Had the value of X been 15% less and the value of Y been Rs 76 less, a profit of 30% would have been earned. What was the value of ‘X’?

(A)  Rs 640

(B)  Rs 400

(C)  Rs 600

(D)  Rs 800

(E)  Rs 840

Answer: (D)

Directions-(Q. 35 to 39) Refer to the graph and answer the given question-

35. Out of the total number of projects handled by company A in 2005 and 2006 together, 20% were Governmental projects. What was the total number of governmental projects handled by company A in 2005 and 2006 together ?

(A)  108

(B)  132

(C)  128

(D)  116

(E)  122

Answer: (E)

36. The project handled by company B can be broadly classified into types : Governmental projects and Non-governmental projects. If the average number of non-governmental projects handled by company B in 2003 and 2004 is 127. what is the total number of governmental projects handled by the same company in 2003 and 2004 together?

(A)  204

(B)  188

(C)  192

(D)  196

(E)  212

Answer: (D)

37. Number of projects handled by company B decreased by what per cent from 2004 to 2006 ?

(A) 

(B) 

(C) 

(D) 

(E) 

Answer: (E)

38. If the number of projects handled by company A increased by 20% from 2007 to 2008 and by 5% from 2008 to 2009, what was the number of projects handled by company A in 2009?

(A)  378

(B)  372

(C)  384

(D)  396

(E)  368

Answer: (A)

39. What is the difference between the total number of projects handled by company A in 2003 and 2004 together and total number of projects handled by company B in 2005 and 2007 together?

(A)  120

(B)  150

(C)  130

(D)  180

(E)  170

Answer: (B)

40. The speed of the boat in still water is 24 kmph and the speed of the stream is 4 km/h. The item taken by the boat to travel from A to B downstream is 36 minutes less than the time taken by the same boat to travel from B to C upstream. If the distance between A and B is 4 km more than the distance between B and C, what is the distance between A and B?

(A)  112 km

(B)  140 km

(C)  56 km

(D)  84 km

(E)  28 km

Answer: (C)

IBPS Agriculture Officers Examination-2016 Held on 29-1-2017 Quantitative Aptitude Question Paper With Answer Key

IBPS Agriculture Officers Examination-2016 Held on 29-1-2017 Quantitative Aptitude
IBPS Agriculture Officers Examination-2016 Held on 29-1-2017 Quantitative Aptitude Question Paper With Answer Key

IBPS Agriculture Officers Examination-2016 Held on 29-1-2017

Quantitative Aptitude

Directions – (Q. 1-5) The question consists of a question and two statements I and II given below it. You have to decide whether the data provided in the statements are sufficient to answer the question. Read the both the statements and choose the appropriate option.

(A) The data in both the statements I and II together are necessary to answer the question.

(B) The data either in statement I alone or in statement II alone are sufficient to answer the question.

(C) The data in both the statements I and II together are not sufficient to answer the question.

(D) The data in statement II alone are sufficient to answer the question, while the data in statement I alone are not sufficient to answer the question.

(E) The data in statement I alone are sufficient to answer the question, while the data in statement II alone are not sufficient to answer the question.

1. What is the cost price of the article ? (in Rs)

I. 8% of the selling price of the article is equal to the 12% of the cost price.

II. 15% of the cost price of the article is Rs 6 more than 9% of the selling price of the article.

Answer: (A)

2. In how many days A, B and C together can finish a piece of work?

I. A is twice as efficient as B.

II. A and B together can finish the piece of work in  days. B and C together can finish the piece of work in  17 A and C together can finish the same piece of work in   

Answer: (D)

3. What distance (in km) the boat can travel downstream in 20 minutes?

I. The distance travelled by the boat upstream in 48 minutes is equal to the distance travelled by the boat downstream in 36 minutes.

II. The respective ratio between speed of the boat in still water and speed of the water currents is 7 : 1.

Answer: (C)

4. What is M’s mother’s present age ? (in years)

I. The respective ratio between M’s father present age and M’s present age is 7 : 2.

II. The combined present age of M’s father and M is 6 years more than the combined present age of M’s mother and M. 12 years ago M’s father age that time was  M’s mother’s age that time.

Answer: (D)

5. Side of a square is equal to the length of a rectangle. What is the area of the square ? (in m2)

I. The difference between area of the rectangle and that of the square is 196 m2.

II. The difference between perimeter of the square and that of the rectangle is 14 m.

Answer: (A)

Directions – (Q. 6-11) Study the table and answer the given question.

   The given table shows the attendance at a stadium (having a seating capacity of 40,000) during all the days of a 5-day test match.

Note- Everyone left the stadium at the end of play. End of play occurs at the end of every day.

6. What was the average number of people who left the stadium before the end of play on Thursday, Friday and Sunday?

(A)  3700

(B)  3900

(C)  3300

(D)  3100

(E)  3500

Answer: (E)

7. What is respective ratio between the total number of people who left the stadium before the end of play on Friday and Saturday together and that who left the stadium before the end of play on Thursday and Monday together?

(A)  3 : 5

(B)  4 : 7

(C)  5 : 7

(D)  5 : 8

(E)  2 : 3

Answer: (C)

8. Out of the number of people who left the stadium before the end of play on Monday, the respective ratio between the number of children, adults and senior citizens was 4 : 3 : 1. What was the number of adults who left the stadium before the end of play on Monday?

(A)  840

(B)  750

(C)  1200

(D)  900

(E)  1020

Answer: (B)

9. What per cent the total seats of the stadium was sold for Saturday, if 1,080 people did not turn up for that day’s play ?

(A)  81.2

(B)  78.7

(C)  82.7

(D)  94.8

(E)  83.2

Answer: (C)

10. If males comprised 65% and 60% of those who attended the stadium on Friday and Sunday respectively, what was the difference between the number of males who attended the stadium on Friday and Sunday?

(A)  1800

(B)  3400

(C)  3200

(D)  5100

(E)  2400

Answer: (E)

11. The number of people who stayed back till the end of p lay on Friday was what per cent more than that on Thursday?

(A)  50

(B)  60

(C)  40

(D)  25

(E)  30

Answer: (A)

Directions – (Q. 12-16) What will come in place of question-mark (?) in the given number series ?

12. 5   388    387    383    359    ?

(A)  167

(B)  154

(C)  181

(D)  199

(E)  173

Answer: (A)

13. 350   410    466    513    544    ?

(A)  585

(B)  560

(C)  572

(D)  595

(E)  550

Answer: (E)

14. 2   3        8        27      ?        565

(A)  174

(B)  132

(C)  166

(D)  148

(E)  112

Answer: (E)

15. 23   11      10      5   25      ?

(A)  90

(B)  45

(C)  50

(D)  75

(E)  60

Answer: (E)

16. 86   86      79      105    42      ?

(A)  112

(B)  166

(C)  188

(D)  148

(E)  128

Answer: (B)

17. Ram and Ali were travelling on their motorbikes from point P to Q. Ram, travelling at a speed of 60 km/hr leaves from point P at 6 am. Ali travelling at a speed of 90 km/hr leaves point P at 10 am on the same day as Ram. At what distance from point P, will Ram and Ali meet?

(A)  400  km

(B)  720 km

(C)  420 km

(D)  640 km

(E)  480 km

Answer: (B)

18. A started a business with an investment of Rs 12,000. At the end of six months from the start of the business, A withdrew half of his initial investment and B and C invested in the ratio of 10 : 9 respectively. If A’s share in annual profit of Rs 14,000 was Rs 4,500, what was the investment made by B?

(A)  Rs 20,000

(B)  Rs 25,000

(C)  Rs 10,000

(D)  Rs 12,000

(E)  Rs 12,500

Answer: (A)

Directions – (Q. 19-24) What approximate value will come in place of question mark in the given question ?

(You are not expected to calculate the exact value).

19. 85 + 12.91 × 2.04 = 112.98 × ?

(A)  2

(B)  9

(C)  4

(D)  5

(E)  3

Answer: (D)

20. (419.89 + 59.99) = ? ×899

(A)  18

(B)  12

(C)  15

(D)  6

(E)  8

Answer: (B)

21. 

(A)  9

(B)  11

(C)  13

(D)  7

(E)  15

Answer: (B)

22. ? % of (144.98 ÷06) + 27.01 = 50.04.

(A)  80

(B)  30

(C)  65

(D)  90

(E)  75

Answer: (C)

23. √145 + ?2 ×98 = 44

(A)  2

(B)  1

(C)  8

(D)  4

(E)  6

Answer: (D)

24. 

(A)  1

(B)  6

(C)  3

(D)  2

(E)  4

Answer: (C)

Directions – (Q. 25-30) – Refer to the pie charts ad answer the given question.

25. Combining stores D and E, only 40% of bags (of domestic and international brands) sold was of brand ‘X’. Among those, only 50% was leather bags. What is the total number of leather bags of brand ‘X’ sold by stores D and E together?

(A)  249

(B)  236

(C)  247

(D)  243

(E)  239

Answer: (D)

26. Approximately, what per cent of bags sold by store D was of domestic brands?

(A)  40

(B)  52

(C)  58

(D)  45

(E)  35

Answer: (D)

27. Number of bags of international brands sold by stores B and E increased by 20% and 25% respectively from 2010 to 2011. In 2011, if the respective ratio between total number of bags of international brands sold by stores B and E together and that of domestic brands sold by same stores together was 13 : 21 what was the total number of bags of domestic brands sold by stores B and E together in 2011?

(A)  588

(B)  609

(C)  567

(D)  667

(E)  547

Answer: (C)

28. What is the difference between total number of bags of domestic brands sold by stores A and B together and that sold by stores D and E together?

(A)  275

(B)  279

(C)  283

(D)  287

(E)  297

Answer: (B)

29. What is the average number of bags (of domestic and international brands) sold by stores A, B and D?

(A)  528

(B)  522

(C)  518

(D)  520

(E)  532

Answer: (B)

30. What is the central angle corresponding to number of bags of international brands sold by store C? (in degrees)

(A)  78.2

(B)  86.4

(C)  93.2

(D)  84.8

(E)  85.4

Answer: (B)

31. Ria invests Rs P in scheme A which offers simple interest @ 12% p.a. and Rs P + 2,520 in scheme B which offers compound interest (compounded annually) @ 20% p.a. The respective ratio between the amount received from scheme A at the end of 5 years and that received from scheme B at the end of 2 years was 4 : 5. What is the value of P?

(A)  6,720

(B)  6,500

(C)  6,480

(D)  8,000

(E)  6,000

Answer: (C)

32. Cost price of article B is 20% more than that of article A. Article B was marked 20% above its cost price. The difference (in Rs)between the cost price and the marked price is equal that of article A. Both the articles were sold at a discount of 10%. If the selling price of article B is Rs 45 more than that of article A, what is the cost price of article A?

(A)  Rs 250

(B)  Rs 300

(C)  Rs 280

(D)  Rs 225

(E)  Rs 240

Answer: (A)

Directions- (Q. 33-38) Study the following information carefully to answer the question.

   A bakery sells three baked products-Macaroons, cupcakes and brownies. On January 1, 2017, the total number of customers who bought products from the bakery was ‘X’. The respective ratio between male customers and female customers was 4 : 9.

   Out of the total number of male customers, 1/5th bought only macaroons 3/6th bought only cupcakes and 30% bought only brownies. The number of customers who bought only macaroons and cupcakes both was equal to the number of customers who bought only cupcakes and brownies both. The number of customers who bought only macaroons and brownies was 6 less than the number of customers who bought only macaroons and cupcakes both. 8 customers bought all the three baked products and 48 customers bought only brownies.

   Out of the total number of female customers, 1/8th bought only macaroons, 3/8th bought only brownies, 10% bought only macaroons and cupcakes and 5% bought only cupcakes and brownies. The number of customers who bought only cupcake was twice the number of customers who bought only macaroons. 16 customers bought only macaroons and brownies and the remaining bought all three baked products.

33. What is t he respective ratio between the total number of male customers who purchased both cupcakes and brownies and the total number of female customers who purchased the same?

(A)  15 : 17

(B)  14 : 23

(C)  15 : 19

(D)  12 : 19

(E)  12 : 17

Answer: (D)

34. The total number of customers (both male and female) who bought only brownies was what per cent more than that who bought only cupcakes?

(A)  55.5

(B)  52.5

(C)  54

(D)  58

(E)  51.5

Answer: (B)

35. Out of the male customers who bought macaroons, 1/3rd bought 4 macaroons each and remaining bought 2 macaroons each. How many total macaroons were sold to male customers?

(A)  176

(B)  181

(C)  153

(D)  167

(E)  168

Answer: (A)

36. What is the difference between the number of males and females customers who bought only cupcakes?

(A)  30

(B)  50

(C)  60

(D)  40

(E)  70

Answer: (C)

37. On January 2, 2017 the total number of customers who purchased from the bakery decreased by 10% as compared to the previous day. If the respective ratio between male customers and female customers was 4 : 5, what was the number of male customers?

(A)  236

(B)  232

(C)  216

(D)  208

(E)  220

Answer: (D)

38. What is the difference between the total number of customers (both male and female) who bought all the three baked products and that who bought only two of the baked products?

(A)  84

(B)  74

(C)  53

(D)  58

(E)  48

Answer: (A)

39. A jar had 120 litre mixture of milk and water in the respective ratio of 5 : 1. 30 litres of this mixture is taken out and ‘X’ litre of each milk and water is added to the jar. The respective ratio between milk and water in the jar was 4 : 1 respectively, what was the total quantity of both milk and water added to jar ? (in litres)

(A)  12

(B)  10

(C)  16

(D)  20

(E)  18

Answer: (B)

Directions – (Q. 40-45) In the following questions two equations numbered I and II are given you have to solve both the equations and give answer, if-

(A) x > y       (B) x ≥ y

(C) x < y       (D) x ≤ y

(E) x = y or relationship cannot be established.

40. I. 4x2 + 12x + 5 = 0

II. y2 + 7y + 12 = 0

Answer: (A)

41. I. 3x2 – 10x + 8 = 0

II. 3y2 – 9y + 6 = 0

Answer: (E)

42. I. 2x2 + 3x + 1 = 0

II. 2y2 + 9y + 9 = 0

Answer: (A)

43. I. x2 – 8x + 15 = 0

II. y2 – 11y + 28 = 0

Answer: (E)

44. I. 2x2 + 13x + 21 = 0

II. 2y2 + 9y + 10 = 0

Answer: (C)

45. I. x2 = 64

II. y2 – 15y + 56 = 0

Answer: (E)

46. The present age of A’s father is equal to six times of A’s age two years hence. If A’s mother is eight years younger than A’s father and the respective ratio between present age of A’s mother and father is 7 : 9, what is A’s present age ? (in years)

(A)  12

(B)  6

(C)  5

(D)  4

(E)  8

Answer: (D)

47. If the circumference of a circle is 36 cm more than the perimeter of a square and the radius of t he circle is 3 cm less than the side of the square. What is the measure of the area of square? (in cm2)

(A)  567

(B)  676

(C)  476

(D)  576

(E)  675

Answer: (D)

48. A project can be completed by 20 men and 36 women together in  28 men can complete the same project in 15 days. How much work will be left unfinished if 40 women together work for 7 days?

(A)  1/3

(B)  1/5

(C)  2/5

(D)  2/3

(E) 

Answer: (E)

49. In village A and B, out of their respective population (male + female), 2/5th and 3/8th are females. The numbers of males in village B are 25% more than those in village A. The number of females in village A and B together is 2125, what is the total population (male + female) of village A?

(A)  3600

(B)  2500

(C)  3000

(D)  2000

(E)  3500

Answer: (B)

50. In the month of July, Ramu spent Rs 3,225 on paying electricity bill, Rs. 1,290 on paying water bill, Rs 5,160 on paying telephone bill and Rs 645 on paying cable bill. After paying these bills he is left with 84% of his monthly salary. Out of the remaining salary he spends 20% on buying groceries and the remaining he invests in mutual funds and VPF in the respective ratio of 5 : 3. How much does he invests in mutual funds every month?

(A)  Rs 29,400

(B)  Rs 28,500

(C)  Rs 30,650

(D)  Rs 26,550

(E)  Rs 27,090

Answer: (E)

© Copyright Entrance India - Engineering and Medical Entrance Exams in India | Website Maintained by Firewall Firm - IT Monteur